Bac – Spécialité mathématiques – Centres étrangers – sujet 2 – 14 mars 2023

Centre étrangers – 14 mars 2023

Spécialité maths – Sujet 2 – Correction

L’énoncé de ce sujet de bac est disponible ici.

Ex 1

Exercice 1

  1. On considère la fonction $F$ définie sur $\R$ par $F(x)=(x-1)\e^x$.
    Elle est dérivable sur $\R$ en tant que produit de fonctions dérivables sur $\R$.
    Pour tout réel $x$ on a :
    $\begin{align*} F'(x)&=1\times \e^x+(x-1)\e^x \\
    &=(1+x-1)\e^x \\
    &=x\e^x \\
    &=f(x)\end{align*}$
    Réponse B
    $\quad$
  2. $f$ est convexe sur $[5;+\infty[$ donc $f\dsec(x)\pg 0$ sur cet intervalle.
    Il semblerait que $f(x)\pg 0$ sur cet intervalle également.
    Réponse D
    $\quad$
  3. $g(0)=2\ssi \dfrac{a}{b+1}=2$
    $\lim\limits_{t\to +\infty} \e^{-t}=0$ donc $\lim\limits_{t\to +\infty} g(t)=\dfrac{a}{b}$. Par conséquent $\dfrac{a}{b}=3$.
    On résout donc le système :
    $\begin{align*}\begin{cases} \dfrac{a}{b+1}=2\\[2mm] \dfrac{a}{b}=3 \end{cases} &\ssi \begin{cases} a=2b+2\\a=3b\end{cases} \\
    &\ssi \begin{cases} a=3b\\3b=2b+2\end{cases} \\
    &\ssi \begin{cases} b=2\\a=6\end{cases}\end{align*}$
    Réponse D
    $\quad$
  4. On appelle $A$ l’événement “choisir l’urne A”, $B$ l’événement “choisir l’urne B” et $V$ l’événement “obtenir une boule verte”
    $(A,B)$ forme un système complet d’événements fini. D’après la formule des probabilités totales :
    $\begin{align*} p(V)&=p(A\cap V)+p(B\cap V) \\
    &=p(A)p_A(V)+p(B)p_B(V) \\
    &=\dfrac{1}{2}\times \dfrac{1}{2}+\dfrac{1}{2}\times \dfrac{3}{4} \\
    &=\dfrac{5}{8}\end{align*}$
    On veut calculer :
    $\begin{align*} p_V(B)&=\dfrac{p(V\cap B)}{p(V)} \\
    &=\dfrac{p(B)p_B(V)}{p(V)} \\
    &=\dfrac{\dfrac{1}{2}\times \dfrac{3}{4}}{\dfrac{5}{8}} \\
    &=\dfrac{3}{5}\end{align*}$
    Réponse C
    $\quad$
  5. Dans le script A, on calcule tous les termes de la somme mais on ne les ajoute pas.
    Dans le script C, la boucle while s’arrête si la somme est supérieure à $100$ ce qui ne correspond pas à ce qu’on veut calculer.
    Dans le script D, la boucle while est infinie puisque $k$ ne change pas de valeur.
    Réponse B
    $\quad$

 

Ex 2

Exercice 2

Partie A : Étude d’une fonction auxiliaire

  1. $\lim\limits_{x\to -1,5^+} 2x+3=0^+$ et $\lim\limits_{X\to 0^+} \ln(X)=-\infty$ donc $\lim\limits_{x\to -1,5^+} f(x)=-\infty$.
    Par conséquent $\lim\limits_{x\to -1,5^+} f(x)-x=-\infty$.
    Donc $\lim\limits_{x\to -1,5^+} g(x)=-\infty$.
    $\quad$
  2. La fonction $g$ est dérivable sur $]-1,5;+\infty[$ en tant que somme et composée de fonctions dérivables.
    Pour tout $x>-1,5$
    $\begin{align*} g'(x)&=\dfrac{2}{2x+3}-1 \\
    &=\dfrac{2-2x-3}{2x+3} \\
    &=\dfrac{-2x-1}{2x+3}\\
    &=-\dfrac{2x+1}{2x+3}\end{align*}$.
    Sur $]-1,5;+\infty[$ on a $2x+3>0$.
    $g'(x)$ est donc du signe de $-(2x+1)$.
    Or $2x+1=0 \ssi 2x=-1 \ssi x=-\dfrac{1}{2}$ et $2x+1>0\ssi 2x>-1\ssi x>-\dfrac{1}{2}$.
    La fonction $g$ est donc strictement croissante sur $]-1,5;-0,5]$ et strictement décroissante sur $[-0,5;+\infty[$.
    $\quad$
  3. a. La fonction $g$ est continue (car dérivable) et strictement décroissante sur $]-0,5;+\infty[$.
    $g(-0,5)=\ln(2)-0,5\approx 0,19>0$.
    $\lim\limits_{x\to +\infty} g(x)=-\infty$.
    D’après le théorème de la bijection (ou corollaire du théorème des valeurs intermédiaires) l’équation $g(x)=0$ admet une unique solution $\alpha$ sur $]-0,5;+\infty[$.
    $\quad$
    b. D’après la calculatrice $\alpha \approx 0,256$ donc $0,25\pp \alpha \pp 0,26$.
    $\quad$

Partie B : Étude de la suite $\boldsymbol{\left(u_n\right)}$

  1. $g(\alpha)=0 \ssi f(\alpha)-\alpha=0\ssi f(\alpha)=\alpha$.
    $f(-1)=-1$.
    La fonction $f$ est strictement croissante sur $]-1,5;+\infty[$.
    Or $[-1;\alpha] \subset ]-1,5;+\infty[$.
    Donc, pour tout $x\in [-1;\alpha]$, $f(x)\in \left[f(-1);f(\alpha)\right]$ soit $f(x)\in [-1;\alpha]$.
    $\quad$
  2. a. Pour tout $n\in \N$ on pose $P(n):~-1 \pp u_n \pp u_{n+1} \pp \alpha$.
    Initialisation : $u_0=0$, $u_1=f(0)=\ln(3)-1\approx 0,1$.
    Par conséquent $0 \pp u_0 \pp u_1 \pp \alpha$ puisque $\alpha \approx 0,256$.
    La propriété est vraie au rang $0$.
    $\quad$
    Hérédité : Soit $n\in \N$. On suppose $P(n)$ vraie.
    $-1\pp u_n \pp u_{n+1} \pp \alpha$.
    La fonction $f$ est strictement croissante sur $]-1,5;+\infty[$.
    Donc $f\left(u_n\right) \pp f\left(u_{n+1}\right)$ soit $u_{n+1} \pp u_{n+2}$.
    De plus d’après la question B.1, pour tout $x\in [-1;\alpha]$, $f(x)\in [-1;\alpha]$.
    Par conséquent, pour tout $u_{n+1}$ et $u_{n+2}$ appartiennent à $[-1;\alpha]$.
    Ainsi $-1 \pp u_{n+1} \pp u_{n+2} \pp \alpha$ et $P(n+1)$ est vraie.
    $\quad$
    Conclusion : La propriété est vraie au rang $0$ et est héréditaire. Donc, pour tout entier naturel $n$ on a $-1\pp u_n\pp u_{n+1} \pp \alpha$.
    $\quad$
    b. La suite $\left(u_n\right)$ est croissante et majorée par $\alpha$. Elle converge donc.
    $\quad$

Ex 3

Exercice 3

  1. a. On a $H(0;2;2)$, $M(3,0,1)$ et $N(3,1,1)$.
    $\quad$
    b. $\vect{HM}\begin{pmatrix} 3\\-2\\-1\end{pmatrix}$
    Une représentation paramétrique de la droite $(HM)$ est donc $\begin{cases} x=3t\\y=2-2t\\z=2-t\end{cases} \quad \forall t\in \R$.
    $\quad$
  2. Les abscisses des points $B$, $C$ et $F$ sont toutes égales à $2$.
    Une équation du plan $(BCF)$ est donc $x=2$.
    On appelle $P’$ le point de coordonnées $\left(2;\dfrac{2}{3};\dfrac{4}{3}\right)$.
    L’abscisse de $P’$ est $2$ donc $P’$ appartient donc à $(BCF)$.
    Prenons $t=\dfrac{2}{3}$ dans la représentation paramétrique de $(HM)$.
    On obtient $x=2$, $y=2-\dfrac{4}{3}=\dfrac{2}{3}$ et $z=2-\dfrac{2}{3}=\dfrac{4}{3}$. Donc $P’$ appartient à $(HM)$.
    La droite $(HM)$ n’est pas incluse dans le plan $(BCF)$ puisque $H$ n’appartient pas à $(BCF)$.
    Ainsi le point d’intersection de $(HM)$ et de $(BCF)$ est $P’$ de coordonnées $\left(2;\dfrac{2}{3};\dfrac{4}{3}\right)$.
    $\quad$
  3. a. $\vect{PM}\begin{pmatrix}1\\-\dfrac{2}{3}\\[2mm]-\dfrac{1}{3}\end{pmatrix}$ et $\vect{PN}\begin{pmatrix}1\\\dfrac{1}{3}\\[2mm]-\dfrac{1}{3}\end{pmatrix}$
    Par conséquent
    $\begin{align*} \vect{PM}.\vect{PN}&=1-\dfrac{2}{9}+\dfrac{1}{9} \\
    &=\dfrac{8}{9}\end{align*}$
    $\quad$
    b.
    $\begin{align*} PM&=\sqrt{1^2+\left(-\dfrac{2}{3}\right)^2+\left(-\dfrac{1}{3}\right)^2} \\
    &=\sqrt{1+\dfrac{4}{9}+\dfrac{1}{9}} \\
    &=\sqrt{\dfrac{14}{9}}\\
    &=\dfrac{\sqrt{14}}{3}\end{align*}$
    $\quad$
    c. D’une part $ \vect{PM}.\vect{PN}=\dfrac{8}{9}$
    D’autre part $ \vect{PM}.\vect{PN}=PM\times PN\times \cos\left(\widehat{MPN}\right)$.
    Par conséquent
    $\cos\left(\widehat{MPN}\right)=\dfrac{\dfrac{8}{9}}{PM\times PN}$
    soit $\cos\left(\widehat{MPN}\right)=\dfrac{\dfrac{8}{9}}{\dfrac{\sqrt{14}}{3}\times \dfrac{\sqrt{11}}{3}}$
    Ainsi $\widehat{MPN} \approx 49,86$°$<55$°.
    Le toît pourra être construit.
    $\quad$
  4. $E(0;0;2)$ donc $\vect{EN}\begin{pmatrix}3\\1\\-1\end{pmatrix}$.
    Méthode 1 : 
    Une représentation paramétrique de la droite $(EN)$ est $\begin{cases} x=3k\\y=k\\z=2-k\end{cases} \forall k\in \R$.
    Résolvons le système :
    $\begin{align*} \begin{cases} x=3k\\y=k\\z=2-k\\x=3t\\y=2-2t\\z=2-t\end{cases}&\ssi \begin{cases} x=3k\\y=k\\z=2-k\\k=t\\2-2t=k\\2-t=2-k\end{cases} \\
    &\ssi \begin{cases} x=3k\\y=k\\z=2-k\\k=t\\2-2t=t\end{cases} \\
    &\ssi \begin{cases} x=3k\\y=k\\z=2-k\\k=t\\3t=2\end{cases} \\
    &\ssi \begin{cases} t=k=\dfrac{2}{3}\\x=2\\y=\dfrac{2}{3}\\z=\dfrac{4}{3}\end{cases}\end{align*}$.
    Les droites $(HM)$ et $(EN)$ sont donc sécantes en $P$.
    $\quad$
    Méthode 2 :
    $E$ et $M$ ont la même ordonnée ce qui n’est pas le cas de $M$ et $H$. Ainsi $E$, $H$ et $M$ ne sont pas alignés.
    Les droites $(HM)$ et $(EN)$ sont donc soit sécantes soit non coplanaires.
    $\vect{EN}\begin{pmatrix}3\\1\\-1\end{pmatrix}$ et $\vect{PN}\begin{pmatrix}1\\\dfrac{1}{3}\\[2mm]-\dfrac{1}{3}\end{pmatrix}$.
    Donc $\vect{EN}=-3\vect{PN}$.
    Ces deux vecteurs sont colinéaires. Par conséquent $E$, $P$ et $N$ sont alignés.
    Par construction, $P$ appartient à $(HM)$.
    Ainsi $(HM)$ et $(EN)$ sont sécantes en $P$.
    $\quad$

Ex 4

Exercice 4

On appelle $X$ la variable aléatoire comptant le nombre de candidats se qualifiant lors de la première phase.
On répète de manière indépendante $4$ fois la même expérience de Bernoulli de paramètre $0,6$.
$X$ suit donc la loi binomiale de paramètre $n=4$ et $p=0,6$.
$\begin{align*}
P(X\pg 2)&=1-P(X\pp 1) \\
&=1-P(X=0)-P(X=1) \\
&=1-0,4^4-\dbinom{4}{1}0,6^1\times 0,4^3\\
&\approx 0,821\end{align*}$.
Par conséquent $P(X\pg 2) \pg 0,8$ : la première condition est vérifiée.

La durée moyenne est égale à
$\begin{align*} d_m&=5P(X=2)+9P(X=3)+11P(X=4)
&\approx 6,3\\
&>6\end{align*}$
La condition 2 n’est pas vérifiée.

Le jeu ne pourra pas être retenu.

$\quad$

Énoncé

Exercice 1 (QCM)       5 points

Cet exercice est un questionnaire à choix multiples. Pour chaque question, une seule des quatre propositions est exacte. Indiquer sur la copie le numéro de la question et la lettre de la proposition choisie. Aucune justification n’est demandée.

Pour chaque question, une réponse exacte rapporte un point. Une réponse fausse, une réponse multiple ou l’absence de réponse ne rapporte ni n’enlève de point.

  1. Question1:
    Soit $f$ la fonction définie sur $\R$ par $f(x)=x\e^x$. Une primitive $F$ sur $\R$ de la fonction $f$ est définie par :
    A. $F(x)=\dfrac{x^2}{2}\e^x$
    B. $F(x)=(x-1)\e^x$
    C. $F(x)=(x+1)\e^x$
    D. $F(x)=\dfrac{1}{2}x\e^{x^2}$
    $\quad$
  2. La courbe $C$ ci-dessous représente une fonction $f$ définie et deux fois dérivable sur $]0; +\infty[$.
    On sait que :
    $\bullet$ le maximum de la fonction $f$ est atteint au point d’abscisse $3$ ;
    $\bullet$ le point $P$ d’abscisse $5$ est l’unique point d’inflexion de la courbe $C$.
    $\quad$

    $\quad$
    A. pour tout $x\in ]0;5[$, $f(x)$ et $f'(x)$ sont de même signe ;
    B. pour tout $x\in ]5;+\infty[$, $f(x)$ et $f'(x)$ sont de même signe ;
    C. pour tout $x\in ]0;5[$, $f'(x)$ et $f\dsec(x)$ sont de même signe ;
    D. pour tout $x\in ]5;+\infty[$, $f(x)$ et $f\dsec(x)$ sont de même signe .
    $\quad$
  3. Question 3 :
    On considère la fonction $g$ définie sur $[0; +\infty[$ par $g(t)=\dfrac{a}{b+\e^{-t}}$ où $a$ et $b$ sont deux nombres réels.
    On sait que $g(0) = 2$ et $\lim\limits_{t\to +\infty} g(t)=3$. Les valeurs de $a$ et $b$ sont :
    A. $a=2$ et $b=3$
    B. $a=4$ et $b=\dfrac{1}{3}$
    C. $a=4$ et $b=1$
    D. $a=6$ et $b=2$
    $\quad$
  4. Question 4 :
    Alice dispose de deux urnes A et B contenant chacune quatre boules indiscernables au toucher. L’urne A contient deux boules vertes et deux boules rouges. L’urne B contient trois boules vertes et une boule rouge.
    Alice choisit au hasard une urne puis une boule dans cette urne. Elle obtient une boule verte. La probabilité qu’elle ait choisi l’urne B est :
    A. $\dfrac{3}{8}$
    B. $\dfrac{1}{2}$
    C. $\dfrac{3}{5}$
    D. $\dfrac{5}{8}$
    $\quad$
  5. Question 5 :
    On pose $S = 1+\dfrac{1}{2}+\dfrac{1}{3}+\dfrac{1}{4}+\ldots+\dfrac{1}{100}$.
    Parmi les scripts Python ci-dessous, celui qui permet de calculer la somme $S$ est :
    $$\begin{array}{lll}
    \begin{array}{l}
    \textbf{a.}\\
    \texttt{def somme_a() :}\\
    \hspace{0.8cm}\texttt{S = 0}\\
    \hspace{0.8cm}\texttt{for k in range(100) :}\\
    \hspace{1.6cm}\texttt{S=1/(k+1)}\\
    \hspace{0.8cm}\texttt{return S}\end{array}
    &\phantom{1234}
    &\begin{array}{l}
    \textbf{b.}\\
    \texttt{def somme_b() :}\\
    \hspace{0.8cm}\texttt{S = 0}\\
    \hspace{0.8cm}\texttt{for k in range(100) :}\\
    \hspace{1.6cm}\texttt{S = S + 1/(k + 1)}\\
    \hspace{0.8cm}\texttt{return S}\end{array} \\\\
    \begin{array}{l}
    \textbf{c.}\\
    \texttt{def somme_c() :}\\
    \hspace{0.8cm}\texttt{k = 0}\\
    \hspace{0.8cm}\texttt{while S < 100}\\
    \hspace{1.6cm}\texttt{S=S + 1/(k+1)}\\
    \hspace{0.8cm}\texttt{return S}\end{array}
    &\phantom{1234}
    &\begin{array}{l}
    \textbf{d.}\\
    \texttt{def somme_d() :}\\
    \hspace{0.8cm}\texttt{k = 0}\\
    \hspace{0.8cm}\texttt{while k < 100}\\
    \hspace{1.6cm}\texttt{S = S + 1/(k + 1)}\\
    \hspace{0.8cm}\texttt{return S}\end{array} \\
    \end{array}$$
    $\quad$

$\quad$

Exercice 2       6 points

On considère la fonction $f$ définie sur $]-1,5;+\infty[$ par $f(x)=\ln(2x+3)-1$.

Le but de cet exercice est d’étudier la convergence de la suite $\left(u_n\right)$ définie par $$u_0=0 \text{ et } u_{n+1}=f\left(u_n\right) \text{ pour tout entier naturel $n$.}$$

Partie A : Étude d’une fonction auxiliaire

On considère la fonction $g$ définie sur $]-1,5;+\infty[$ par $g(x)=f(x)-x$.

  1. Déterminer la limite de la fonction $g$ en $-1,5$.
    $\quad$

On admet que la limite de la fonction $g$ en $+\infty$ est $-\infty$.

  1. Étudier les variations de la fonction $g$ sur $]-1,5 ; +∞[$.
    $\quad$
  2. a. Démontrer que, dans l’intervalle $]-0,5 ; +\infty[$, l’équation $g(x)=0$ admet une unique solution $\alpha$.
    $\quad$
    b. Déterminer un encadrement de $\alpha$ d’amplitude $10^{-2}$.
    $\quad$

Partie B : Étude de la suite $\boldsymbol{\left(u_n\right)}$

On admet que la fonction $f$ est strictement croissante sur $]-1,5 ; +\infty[$.

  1. Soit $x$ un nombre réel. Montrer que si $x\in [-1; \alpha]$ alors $f(x)\in [-1;\alpha]$.
    $\quad$
  2. a. Démontrer par récurrence que pour tout entier naturel $n$ :
    $$-1\pp u_n \pp u_{n+1} \pp \alpha$$
    $\quad$
    b. En déduire que la suite $\left(u_n\right)$ converge.
    $\quad$

$\quad$

Exercice 3       6 points

La figure ci-dessous correspond à la maquette d’un projet architectural.
Il s’agit d’une maison de forme cubique $(ABCDEFGH)$ accolée à un garage de forme cubique $(BIJKLMNO)$ où $L$ est le milieu du segment $[BF]$ et $K$ est le milieu du segment $[BC]$.
Le garage est surmonté d’un toit de forme pyramidale $(LMNOP)$ de base carrée $LMNO$ et de sommet $P$ positionné sur la façade de la maison.

On munit l’espace du repère orthonormé $\left(A;\vec{i},\vec{j},\vec{k}\right)$ avec $\vec{i}=\dfrac{1}{2}\vect{AB}$, $\vec{j}=\dfrac{1}{2}\vect{AD}$ et $\vec{k}=\dfrac{1}{2}\vect{AE}$.

  1. a. Par lecture graphique, donner les coordonnées des points $H$, $M$ et $N$.
    $\quad$
    b. Déterminer une représentation paramétrique de la droite $(HM)$.
    $\quad$
  2. L’architecte place le point $P$ à l’intersection de la droite $(HM)$ et du plan $(BCF)$.
    Montrer que les coordonnées de $P$ sont $\left(2;\dfrac{2}{3};\dfrac{4}{3}\right)$.
    $\quad$
  3. a. Calculer le produit scalaire $\vect{PM}.\vect{PN}$.
    $\quad$
    b. Calculer la distance $PM$.
    $\quad$
    On admet que la distance $PN$ est égale à $\dfrac{\sqrt{11}}{3}$
    $\quad$
    c. Pour satisfaire à des contraintes techniques, le toit ne peut être construit que si l’angle $\widehat{MPN}$ ne dépasse pas $55$°. Le toit pourra-t-il être construit ?
    $\quad$
  4. Justifier que les droites $(HM)$ et $(EN)$ sont sécantes. Quel est leur point d’intersection ?
    $\quad$

$\quad$

Exercice 4       3 points

Une société de production s’interroge sur l’opportunité de programmer un jeu télévisé. Ce jeu réunit quatre candidats et se déroule en deux phases.

La première phase est une phase de qualification. Cette phase ne dépend que du hasard.
Pour chaque candidat, la probabilité de se qualifier est $0,6$ .

La deuxième phase est une compétition entre les candidats qualifiés. Elle n’a lieu que si deux candidats au moins sont qualifiés. Sa durée dépend du nombre de candidats qualifiés comme l’indique le tableau ci-dessous (lorsqu’il n’y a pas de deuxième phase, on considère que sa durée est nulle).

$$\begin{array}{|l|c|c|c|c|c|}
\hline
\begin{array}{l}\text{Nombre de}\\
\text{candidats qualifiés}\\
\text{pour la deuxième}\\
\text{phase}\end{array}&\phantom{123}0\phantom{123}&\phantom{123}1\phantom{123}&\phantom{123}2\phantom{123}&\phantom{123}3\phantom{123}&\phantom{123}4\phantom{123}\\
\hline
\begin{array}{l}\text{Durée de la}\\
\text{deuxième phase en}\\
\text{minutes}\end{array}&0&0&5&9&11\\
\hline
\end{array}$$

Pour que la société décide de retenir ce jeu, il faut que les deux conditions suivantes soient vérifiées :

Condition n°1 : La deuxième phase doit avoir lieu dans au moins $80\%$ des cas.

Condition n°2 : La durée moyenne de la deuxième phase ne doit pas excéder $6$ minutes.

$\quad$

Le jeu peut-il être retenu ?

$\quad$

$\quad$

 

Bac – Spécialité mathématiques – Centres étrangers – sujet 1 – 13 mars 2023

Centre étrangers – 13 mars 2023

Spécialité maths – Sujet 1 – Correction

L’énoncé de ce sujet de bac est disponible ici.

Ex 1

Exercice 1

  1. Pour tout $n\in \N$ on a:
    $\begin{align*} u_n&=\dfrac{1+2^n}{3+5^n} \\
    &=\dfrac{2^n\left(\dfrac{1}{2^n}+1\right)}{5^n\left(\dfrac{3}{5^n}+1\right)} \\
    &=\left(\dfrac{2}{5}\right)^n \times \dfrac{\dfrac{1}{2^n}+1}{\dfrac{3}{5^n}+1}\end{align*}$
    $-1<\dfrac{2}{5}<1$ donc $\lim\limits_{n\to +\infty} \left(\dfrac{2}{5}\right)^n=0$.
    De plus $\lim\limits_{n\to +\infty} \dfrac{1}{2^n}=0$ et $\lim\limits_{n\to +\infty} \dfrac{3}{5^n}=0$
    Donc $\lim\limits_{n\to +\infty} u_n=0$.
    Réponse c
    $\quad$
  2. $f$ est dérivable sur $]0;+\infty[$ en tant que produit de fonctions dérivables sur cet intervalle.
    Pour tout $x>0$ on a
    $\begin{align*} f'(x)&=2x\ln(x)+x^2\times \dfrac{1}{x} \\
    &=2x\ln(x)+x \\
    &=x\left(2\ln(x)+1\right)\end{align*}$
    Réponse b
    $\quad$
  3. $h(x)\pp 0$ sur $]-\infty;1]$. Par conséquent, $H$ est décroissante sur $]-\infty;1]$ et donc sur $]-\infty;0]$.
    Or $H(0)=0$.
    Par conséquent, pour tout $x\pp 0$, $H(x)\pg H(0)$ soit $H(x)\pg 0$.
    Réponse a
    $\quad$
  4. Il s’agit d’un algorithme de dichotomie qui est mis en place.
    Dans la boucle while, il faut que l’écart soit supérieur à $0,001$ pour continuer cette boucle. On exclut donc la proposition c.
    La fonction est croissante sur $[a;b]$. Par conséquent si $f(m)<0$ alors $a$ prend la valeur de $m$. On exclut la proposition a.
    Il faut recalculer la variable $m$ à chaque tour de boucle : on exclut la proposition b.
    Réponse d
    $\quad$
  5. On choisit $2$ boules vertes parmi les $3$ boules vertes de l’urne. La probabilité de tirer une boule bleue est égale à $\dfrac{7}{10}$ et celle de tirer une boule verte est égale à $\dfrac{3}{10}$.
    On répète $3$ fois de façon indépendante la même expérience de Bernoulli de paramètre $\dfrac{7}{10}$.
    La variable aléatoire égale au nombre de boules vertes tirées suit la loi binomiale de paramètres $n=3$ et $p=\dfrac{7}{10}$.
    La probabilité de tirer exactement deux boules vertes est égale à $\dbinom{3}{2}\left(\dfrac{7}{10}\right)\left(\dfrac{3}{10}\right)^2$.
    Réponse d
    $\quad$

 

Ex 2

Exercice 2

Partie A

  1. La trottinette est en bon état lors de sa mise en service.
    Ainsi
    $\begin{align*} p_1&=p\left(B_1\right) \\
    &=p\left(B_0\right)p_{B_0}\left(B_1\right)\\
    &=1\times 0,9\\
    &=0,9\end{align*}$
    $\quad$
    $\left(B_1,\conj{B_1}\right)$ est un système complet d’événements fini. D’après la formule des probabilités totales.
    $\begin{align*} p_2&=p\left(B_1\cap B_2\right)+p\left(\conj{B_1}\cap B_2\right) \\
    &=p\left(B_1\right)p_{B_1}\left(B_2\right)+p\left(\conj{B_1}\right)p_{\conj{B_1}}\left(B_1\right) \\
    &=0,9\times 0,9+0,1\times 0,4 \\
    &=0,81+0,04\\
    &=0,85\end{align*}$
    $\quad$
  2. On obtient l’arbre pondéré suivant :
    $\quad$
    $\quad$
  3. $\left(B_n,\conj{B_n}\right)$ est un système complet d’événements fini. D’après la formule des probabilités totales.
    $\begin{align*} p_{n+1}&=p\left(B_n\cap B_{n+1}\right)+p\left(\conj{B_n}\cap B_{n+1}\right) \\
    &=p\left(B_n\right)p_{B_n}\left(B_{n+1}\right)+p\left(\conj{B_n}\right)p_{\conj{B_n}}\left(B_{n+1}\right) \\
    &=0,9\times p_n+0,4\left(1-p_n\right) \\
    &=0,9p_n+0,4-0,4p_n\\
    &=0,5p_n+0,4\end{align*}$
    $\quad$
  4. a. Pour tout $n$ on pose $R(n):~p_n\pg 0,8$.
    Initialisation : $p_0=1\pg 0,8$ donc $R(0)$ est vraie.
    $\quad$
    Hérédité : Soit $n\in \N$. On suppose $R(n)$ vraie.
    $\begin{align*} p_n\pg 0,8& \ssi 0,5p_n\pg 0,4 \\
    &\ssi 0,5p_n +0,4\pg 0,8 \\
    &\ssi p_{n+1} \pg 0,8\end{align*}$
    Donc $R(n+1)$ est vraie.
    $\quad$
    Conclusion : La propriété $R$ est vraie au rang $0$ et est héréditaire. Par conséquent, pour tout entier naturel $n$, $p_n \pg 0,8$.
    $\quad$
    b. L’entreprise peut annoncer qu’au moins $80\%$ du parc de trottinette est bon état à tout moment.
    $\quad$
  5. a. Soit $n\in \N$.
    $\begin{align*} u_{n+1}&=p_{n+1}-0,8 \\
    &=0,5p_n+0,4-0,8 \\
    &=0,5p_n-0,4 \\
    &=0,5\left(p_n-0,8\right) \\
    &=0,5u_n\end{align*}$.
    La suite $\left(u_n\right)$ est donc géométrique de raison $0,5$ et de premier terme $u_0=p_0-0,8=0,2$.
    $\quad$
    b. Pour tout $n\in \N$ on a donc $u_n=0,2\times 0,5^n$.
    Ainsi $p_n=0,8+u_n=0,8+0,2\times 0,5^n$.
    $\quad$
    c. $0<0,5<1$ donc $\lim\limits_{n\to +\infty} 0,5^n=0$.
    Par conséquent $\lim\limits_{n\to +\infty} p_n=0,8$.
    $\quad$

Partie B

  1. On répète $15$ fois de manière indépendante la même expérience de Bernoulli de paramètre $0,8$.
    $X$ suit donc la loi binomiale de paramètres $n=15$ et $p=0,8$.
    $\quad$
  2. On veut calculer :
    $\begin{align*} P(X=15)&=0,8^{15} \\
    &\approx 0,035\end{align*}$
    La probabilité que les $15$ trottinettes soient en bon état est égale à $0,8^{15} \approx 0,035$.
    $\quad$
  3. On veut calculer :
    $\begin{align*} p(X\pg 10)&=1-p(X\pp 9) \\
    &\approx 0,939\end{align*}$
    La probabilité qu’au moins $10$ trottinettes soient en bon état dans un lot de $15$ est environ égale à $0,939$.
    $\quad$
  4. Cela signifie qu’en moyenne, dans un lot de $15$ trottinettes, $12$ sont en bon état.
    $\quad$

 

Ex 3

Exercice 3

  1. $\vect{BF}=\dfrac{1}{2}\vect{AE}$ et $\vec{k}=\dfrac{1}{8}\vect{AE}$.
    Donc $E$ a pour coordonnées $(0,0,8)$, $F$ a pour coordonnées $(4,0,4)$
    Ainsi $I$, milieu de $[EF]$ a pour coordonnées $(2,0,6)$.
    $J$ est le milieu de $[AE]$ donc $J$ a pour coordonnées $(0,0,4)$.
    $\quad$
  2. a. On a $\vect{IJ}\begin{pmatrix} -2\\0\\-2\end{pmatrix}$.
    $G$ a pour coordonnées $(4,4,4)$ donc $\vect{IG}$ a pour coordonnées $\begin{pmatrix} 2\\4\\-2\end{pmatrix}$.
    Ces deux vecteurs ne sont pas colinéaires puisqu’ils n’ont pas la même coordonnée nulle.
    D’une part $\vec{n}.\vect{IJ}=2+0-2=0$
    D’autre part $\vec{n}.\vect{IG}=-2+4-2=0$
    $\vec{n}$ est donc orthogonal à deux vecteurs non colinéaires du plan $(IGJ)$.
    $\vec{n}$ est un vecteur normal au plan $(IGJ)$.
    $\quad$
    b. Une équation cartésienne du plan $(IGJ)$ est alors de la forme $-x+y+z+d=0$.
    $I(2,0,6)$ appartient à ce plan. Donc $-2+0+6+d=0 \ssi d=-4$.
    Une équation cartésienne du plan $(IGJ)$ est donc $-x+y+z-4=0$.
    $\quad$
  3. $H$ a pour coordonnées $(0,4,8)$.
    Une représentation paramétrique de la droite $d$ est $\begin{cases} x=-t\\y=4+t\\z=8+t\end{cases}, \quad \forall t\in \R$.
    $\quad$
  4. Montrons que le point $L’\left(\dfrac{8}{3};\dfrac{4}{3};\dfrac{16}{3}\right)$ appartient à la droite et au plan.
    $-\dfrac{8}{3}+\dfrac{4}{3}+\dfrac{16}{3}-4= \dfrac{12}{3}-4  =0$. $L’$ appartient au plan $(IGJ)$.
    Prenons $t=-\dfrac{8}{3}$ dans la représentation paramétrique de $d$.
    On obtient alors $x=\dfrac{8}{3}$, $y=4-\dfrac{8}{3}=\dfrac{4}{3}$ et $z=8-\dfrac{8}{3}=\dfrac{16}{3}$.
    $L’$ appartient alors également à $d$.
    Ainsi $L’$ appartient à la fois à $d$ et au plan $(IGJ)$. La droite $d$ est normale au plan $(IGJ)$; elle n’est donc pas incluse dedans.
    Par conséquent les coordonnées du point $L$ sont bien $\left(\dfrac{8}{3};\dfrac{4}{3};\dfrac{16}{3}\right)$.
    $\quad$
  5. La distance cherchée est $HL$.
    $\vect{HL}\begin{pmatrix} \dfrac{8}{3}\\[2mm]-\dfrac{8}{3}\\[2mm]-\dfrac{8}{3}\end{pmatrix}$.
    Ainsi :
    $\begin{align*} HL&=\sqrt{\left(\dfrac{8}{3}\right)^2+\left(-\dfrac{8}{3}\right)^2+\left(-\dfrac{8}{3}\right)^2} \\
    &=\sqrt{\dfrac{64}{3}} \\
    &=\dfrac{8}{\sqrt{3}}\end{align*}$
    $\quad$
  6. $\quad$
    $\begin{align*} \vect{IG}.\vect{IJ}&=-4+0+4 \\
    &=0\end{align*}$
    Les vecteurs $\vect{IJ}$ et $\vect{IG}$ sont orthogonaux et le triangle $IGJ$ est rectangle en $I$.
    $\quad$
  7. On a :
    $\begin{align*} IJ&=\sqrt{(-2)^2+0^2+(-2)^2} \\
    &=\sqrt{8}\end{align*}$
    $\begin{align*} IG&=\sqrt{2^2+4^2+(-2)^2} \\
    &=\sqrt{24}\end{align*}$Le volume du tétraèdre $IGJH$ est :
    $\begin{align*} V&=\dfrac{1}{3}\times \text{Aire}_{IGJ}\times HL \\
    &=\dfrac{1}{3}\times \dfrac{IJ\times IG}{2}\times HL \\
    &=\dfrac{1}{3}\times \dfrac{\sqrt{8}\times \sqrt{24}}{2}\times \dfrac{8}{\sqrt{3}} \\
    &=\dfrac{32}{3}\end{align*}$
    $\quad$

 

Ex 4

Exercice 4

  1. La fonction $f$ est dérivable sur $[0;+\infty[$ en tant que composée de fonctions dérivables.
    Pour tout $t\pg 0$ on a :
    $\begin{align*} f'(t)&=-(-t+1)\e^{-0,5t^2+t+2} \\
    &=(t-1)\e^{-0,5t^2+t+2}\end{align*}$
    La fonction exponentielle est strictement positive sur $\R$. Par conséquent $f'(t)$ est du signe de $t-1$.
    Or $t-1<0 \ssi t<1$ et $t-1=0 \ssi t=1$.
    Ainsi la fonction $f$ est strictement décroissante sur $[0;1[$.
    Affirmation 1 fausse
    $\quad$
  2. La fonction $f$ est croissante sur $[1;+\infty[$ d’après la question précédente.
    $\lim\limits_{t\to +\infty} (-0,5t^2t+2) = \lim\limits_{t\to +\infty} -0,5t^2=-\infty$ (limite des termes de plus haut degré).
    Or $\lim\limits_{X\to -\infty} \e^X=0$ donc $\lim\limits_{t\to +\infty} \e^{-0,5t^2+t+2}=0$ et $\lim\limits_{t\to +\infty} f(t)=\e^3$.
    Or $\e^3 \approx 20,086$.
    Ainsi, après $1$ heure, la population de bactéries va croître jusqu’à environ $20~086<21~000$ entités.
    Affirmation 2 fausse
    $\quad$
  3. La fonction $f$ est continue (car dérivable) et strictement décroissante sur $[0;1]$.
    $f(0)=\e^3-\e^2\approx 12,7>10$
    $f(1)\approx 7,9<10$
    D’après le théorème de la bijection (ou corollaire du théorème des valeurs intermédiaires) l’équation $f(t)=10$ admet une unique solution sur $[0;1]$.
    $\quad$
    La fonction $f$ est continue (car dérivable) et strictement croissante sur $[1;+\infty[$.
    $f(1)\approx 7,9<10$
    $\lim\limits_{t\to +\infty} f(t)=\e^3>10$
    D’après le théorème de la bijection (ou corollaire du théorème des valeurs intermédiaires) l’équation $f(t)=10$ admet une unique solution sur $[1;+\infty[$.
    $\quad$
    Finalement l’équation $f(t)=10$ admet exactement deux solutions sur $[0;+\infty[$.
    La population de bactéries aura un effectif de $10~000$ à deux reprises au cours du temps.
    Affirmation 3 vraie.
    $\quad$

 

Énoncé

Exercice 1  (QCM)     5 points

Cet exercice est un questionnaire à choix multiples. Pour chaque question, une seule des quatre propositions est exacte. Indiquer sur la copie le numéro de la question et la lettre de la proposition choisie. Aucune justification n’est demandée.

Pour chaque question, une réponse exacte rapporte un point. Une réponse fausse, une réponse multiple ou l’absence de réponse ne rapporte ni n’enlève de point.

  1. Question 1 :
    On considère la suite numérique $\left(u_n\right)$ définie pour tout $n$ entier naturel par $u_n=\dfrac{1+2^n}{3+5^n}$.
    Cette suite :
    a. diverge vers $+\infty$
    b. converge vers $\dfrac{2}{5}$
    c. converge vers $0$
    d. converge vers $\dfrac{1}{3}$
    $\quad$
  2. Question 2 :
    Soit $f$ la fonction définie sur $]0; +\infty[$ par $f(x)= x^2\ln(x)$. L’expression de la fonction dérivée de $f$ est :
    a. $f'(x)=2x\ln(x)$
    b. $f'(x)=x\left(2\ln(x)+1\right)$
    c. $f'(x)=2$
    d. $f'(x)=x$
    $\quad$
  3. Question 3 :
    On considère une fonction ℎ définie et continue sur $\R$ dont le tableau de variation est donné ci-dessous :
    $\quad$

    $\quad$
    On note $H$ la primitive de $h$ définie sur $\R$ qui s’annule en $0$.
    Elle vérifie la propriété :
    a. $H$ positive sur $]-\infty ; 0]$.
    b. $H$ négative sur $]-\infty ; 1]$.
    c. $H$ croissante sur $]-\infty ; 1]$.
    d. $H$ croissante sur $\R$.
    $\quad$
  4. Question 4 :
    Soit deux réels $a$ et $b$ avec $a < b$.
    On considère une fonction $f$ définie, continue, strictement croissante sur l’intervalle $[a ; b]$ et qui s’annule en un réel $\alpha$.
    Parmi les propositions suivantes, la fonction en langage Python qui permet de donner une valeur approchée de $\alpha$ à $0,001$ est :
    $$\begin{array}{lll}
    \begin{array}{l}
    \textbf{a.}\\
    \textcolor{blue}{\text{def }} \textbf{racine(a,b) :}\\
    \hspace{0.8cm} \textcolor{blue}{\text{while }} \textcolor{violet}{\text{abs}}\text{(b − a) >= } \textcolor{brown}{ 0.001 } \text{:}\\
    \hspace{1,6cm}\text{m = (a +b)/}\textcolor{brown}{\text{2}}\\
    \hspace{1,6cm}\textcolor{blue}{\text{if }} \text{f(m) <} \textcolor{brown}{\text{0}} \text{:}\\
    \hspace{2,4cm}\text{b = m}\\
    \hspace{1,6cm}\textcolor{blue}{\text{else }} \text{:}\\
    \hspace{2,4cm}\text{a = m}\\
    \hspace{0,8cm}\textcolor{blue}{\text{return }} \text{m}\\
    \end{array}
    &\phantom{1234}&
    \begin{array}{l}
    \textbf{c.}\\
    \textcolor{blue}{\text{def }} \textbf{racine(a,b) :}\\
    \hspace{0,8cm}\text{m = (a +b)/}\textcolor{brown}{\text{2}}\\
    \hspace{0.8cm} \textcolor{blue}{\text{while }} \textcolor{violet}{\text{abs}}\text{(b − a) <= } \textcolor{brown}{ 0.001 } \text{:}\\
    \hspace{1,6cm}\textcolor{blue}{\text{if }} \text{f(m) <} \textcolor{brown}{\text{0}} \text{:}\\
    \hspace{2,4cm}\text{a = m}\\
    \hspace{1,6cm}\textcolor{blue}{\text{else }} \text{:}\\
    \hspace{2,4cm}\text{b = m}\\
    \hspace{0,8cm}\textcolor{blue}{\text{return }} \text{m}\\
    \end{array} \\
    \begin{array}{l}
    \textbf{b.}\\
    \textcolor{blue}{\text{def }} \textbf{racine(a,b) :}\\
    \hspace{0,8cm}\text{m = (a +b)/}\textcolor{brown}{\text{2}}\\
    \hspace{0.8cm} \textcolor{blue}{\text{while }} \textcolor{violet}{\text{abs}}\text{(b − a) >= } \textcolor{brown}{ 0.001 } \text{:}\\
    \hspace{1,6cm}\textcolor{blue}{\text{if }} \text{f(m) <} \textcolor{brown}{\text{0}} \text{:}\\
    \hspace{2,4cm}\text{a = m}\\
    \hspace{1,6cm}\textcolor{blue}{\text{else }} \text{:}\\
    \hspace{2,4cm}\text{b = m}\\
    \hspace{0,8cm}\textcolor{blue}{\text{return }} \text{m}\\
    \end{array}
    &\phantom{1234}&
    \begin{array}{l}
    \textbf{d.}\\
    \textcolor{blue}{\text{def }} \textbf{racine(a,b) :}\\
    \hspace{0.8cm} \textcolor{blue}{\text{while }} \textcolor{violet}{\text{abs}}\text{(b − a) >= } \textcolor{brown}{ 0.001 } \text{:}\\
    \hspace{1,6cm}\text{m = (a +b)/}\textcolor{brown}{\text{2}}\\
    \hspace{1,6cm}\textcolor{blue}{\text{if }} \text{f(m) <} \textcolor{brown}{\text{0}} \text{:}\\
    \hspace{2,4cm}\text{a = m}\\
    \hspace{1,6cm}\textcolor{blue}{\text{else }} \text{:}\\
    \hspace{2,4cm}\text{b = m}\\
    \hspace{0,8cm}\textcolor{blue}{\text{return }} \text{m}\\
    \end{array}\end{array}$$
    $\quad$
  5. Question 5 :
    Une urne contient $10$ boules indiscernables au toucher dont $7$ sont bleues et les autres vertes. On effectue trois tirages successifs avec remise. La probabilité d’obtenir exactement deux boules vertes est :
    a. $\left(\dfrac{7}{10}\right)^2\times \dfrac{3}{10}$
    b. $\left(\dfrac{3}{10}\right)^2$
    c. $\dbinom{10}{2}\left(\dfrac{7}{10}\right)\left(\dfrac{3}{10}\right)^2$
    d. $\dbinom{3}{2}\left(\dfrac{7}{10}\right)\left(\dfrac{3}{10}\right)^2$
    $\quad$

$\quad$

Exercice 2       6 points

Les deux parties de cet exercice sont indépendantes.
Dans une grande ville française, des trottinettes électriques sont mises à disposition des usagers. Une entreprise, chargée de l’entretien du parc de trottinettes, contrôle leur état chaque lundi.

$\quad$

Partie A

On estime que :

  • lorsqu’une trottinette est en bon état un lundi, la probabilité qu’elle soit encore en bon état le lundi suivant est $0,9$ ;
  • lorsqu’une trottinette est en mauvais état un lundi, la probabilité qu’elle soit en bon état le lundi suivant est $0,4$.

On s’intéresse à l’état d’une trottinette lors des phases de contrôle.
Soit $n$ un entier naturel. On note $B_n$ l’événement « la trottinette est en bon état $n$ semaines après sa mise en service » et $p_n$ la probabilité de $B_n$.
Lors de sa mise en service, la trottinette est en bon état. On a donc $p_0=1$.

  1. Donner $p_1$ et montrer que $p_2 = 0,85$. On pourra s’appuyer sur un arbre pondéré.
    $\quad$
  2. Recopier et compléter l’arbre pondéré ci-dessous :
    $\quad$

    $\quad$
  3. En déduire que, pour tout entier naturel $n$, $p_{n+1} = 0,5p_n + 0,4$.
    $\quad$
  4. a. Démontrer par récurrence que pour tout entier naturel $n$, $p_n\pg 0,8$.
    $\quad$
    b. À partir de ce résultat, quelle communication l’entreprise peut-elle envisager pour valoriser la fiabilité du parc ?
    $\quad$
  5. a. On considère la suite $\left(u_n\right)$ définie pour tout entier naturel $n$ par $u_n=p_n-0,8$.
    Montrer que $\left(u_n\right)$ est une suite géométrique dont on donnera le premier terme et la raison.
    $\quad$
    b. En déduire l’expression de $u_n$ puis de $p_n$ en fonction de $n$.
    $\quad$
    c. En déduire la limite de la suite $\left(p_n\right)$.
    $\quad$

Partie B
Dans cette partie, on modélise la situation de la façon suivante :

  • l’état d’une trottinette est indépendant de celui des autres ;
  • la probabilité qu’une trottinette soit en bon état est égale à $0,8$.

On note $X$ la variable aléatoire qui, à un lot de $15$ trottinettes, associe le nombre de trottinettes en bon état. Le nombre de trottinettes du parc étant très important, le prélèvement de $15$ trottinettes peut être assimilé à un tirage avec remise.

  1. Justifier que $X$ suit une loi binomiale et préciser les paramètres de cette loi.
    $\quad$
  2. Calculer la probabilité que les $15$ trottinettes soient en bon état.
    $\quad$
  3. Calculer la probabilité qu’au moins $10$ trottinettes soient en bon état dans un lot de $15$.
    $\quad$
  4. On admet que $E(X) = 12$. Interpréter le résultat.
    $\quad$

$\quad$

Exercice 3       6 points

On considère le prisme droit $ABFEDCGH$, de base $ABFE$, trapèze rectangle en $A$.
On associe à ce prisme le repère orthonormé $\left(A;\vec{i},\vec{j},\vec{k}\right)$ tel que :
$\vec{i}=\dfrac{1}{4}\vect{AB}$ ; $\vec{j}=\dfrac{1}{4}\vect{AD}$ ; $\vec{k}=\dfrac{1}{8}\vect{AE}$.
De plus on a $\vect{BF}=\dfrac{1}{2}\vect{AE}$.
On note $I$ le milieu du segment $[EF]$.
On note $J$ le milieu du segment $[AE]$.

 

  1. Donner les coordonnées des points $I$ et $J$.
    $\quad$
  2. Soit $\vec{n}$ le vecteur de coordonnées $\begin{pmatrix}-1\\1\\1\end{pmatrix}$.
    a. Montrer que le vecteur $\vec{n}$ est normal au plan $(IGJ)$.
    $\quad$
    b. Déterminer une équation cartésienne du plan $(IGJ)$.
    $\quad$
  3. Déterminer une représentation paramétrique de la droite $d$, perpendiculaire au plan $(IGJ)$ et passant par $H$.
    $\quad$
  4. On note $L$ le projeté orthogonal du point $H$ sur le plan $(IGJ)$.
    Montrer que les coordonnées de $L$ sont $\left(\dfrac{8}{3};\dfrac{4}{3};\dfrac{16}{3}\right)$.
    $\quad$
  5. Calculer la distance du point $H$ au plan $(IGJ)$.
    $\quad$
  6. Montrer que le triangle $IGJ$ est rectangle en $I$.
    $\quad$
  7. En déduire le volume du tétraèdre $IGJH$.
    On rappelle que le volume $V$ d’un tétraèdre est donné par la formule :
    $$V=\dfrac{1}{3}\times (\textit{aire de la base}) \times \textit{hauteur}$$
    $\quad$

$\quad$

Exercice 4       3 points

Un biologiste a modélisé l’évolution d’une population de bactéries (en milliers d’entités) par la fonction $f$ définie sur $[0; +\infty[$ par $f(t) = \e^3-\e^{-0,5t^2+t+2}$ où $t$ désigne le temps en heures depuis le début de l’expérience.

À partir de cette modélisation, il propose les trois affirmations ci-dessous. Pour chacune d’elles, indiquer, en justifiant, si elle est vraie ou fausse.

  • Affirmation 1 : « La population augmente en permanence ».
    $\quad$
  • Affirmation 2 : « À très long terme, la population dépassera $21~000$ bactéries ».
    $\quad$
  • Affirmation 3 : « La population de bactéries aura un effectif de $10~000 $ à deux reprises au cours du temps ».
    $\quad$

$\quad$

 

Bac – Spécialité mathématiques – Nouvelle Calédonie- sujet 2 – 27 octobre 2022

Nouvelle Calédonie – 27 octobre 2022

Spécialité maths – Sujet 2 – Correction

L’énoncé de ce sujet de bac est disponible ici.

Ex 1

Exercice 1

  1. a. On obtient l’arbre pondéré suivant :
    $\quad$

    $\quad$
    b. On a
    $\begin{align*} p\left(\conj{D}\cap R\right)&=p\left(\conj{D}\right)\times p_{\conj{D}}(R) \\
    &=\dfrac{3}{4}\times 0,35 \\
    &=0,262~5\end{align*}$
    $\quad$
    c. $\left(D,\conj{D}\right)$ forme un système complet d’événements fini.
    D’après la formule des probabilités totales on a :
    $\begin{align*} p(R)&=p(R\cap D)+ p\left(\conj{D}\cap R\right) \\
    &=p(D)\times p_D(R)+0,262~5 \\
    &=\dfrac{1}{4}\times 0,6+0,262~5 \\
    &=0,412~5\end{align*}$
    La probabilité que Stéphanie réussisse un tir est bien égale à $0,412~5$.
    $\quad$
    d. On veut calculer :
    $\begin{align*} p_R\left(\conj{D}\right)&=\dfrac{p\left(R\cap \conj{D}\right)}{p(R)} \\
    &=\dfrac{0,262~5}{0,412~5} \\
    &\approx 0,64\end{align*}$
    La probabilité qu’il s’agisse d’un tir à trois points si Stéphanie réussit un tir est environ égale à $0,64$.
    $\quad$
  2. a. On répète $10$ fois de façon indépendantes la même expérience de Bernoulli de paramètre $0,35$.
    $X$ suit donc la loi binomiale de paramètres $n=10$ et $p=0,35$.
    $\quad$
    b. L’espérance de $X$ est :
    $\begin{align*} E(X)&=10\times 0,35 \\
    &=3,5\end{align*}$
    Sur $100$ tirs à trois points elle en réussit donc en moyenne $35$.
    $\quad$
    c. On veut calculer $P(X\pp 6)\approx 0,97$.
    La probabilité que Stéphanie rate $4$ tirs ou plus est environ égale à $0,97$.
    $\quad$
    d. On veut calculer $P(X\pg 6)=1-P(X\pp 5)\approx 0,09$.
    La probabilité que Stéphanie rate au plus $4$ tirs est environ égale à $0,09$.
    $\quad$
  3. On note $Y$ la variable aléatoire qui compte le nombre de tirs réussis.
    On répète $n$ fois de façon indépendantes la même expérience de Bernoulli de paramètre $0,35$.
    $Y$ suit donc la loi binomiale de paramètres $n$ et $p=0,35$
    On veut déterminer le plus plus petit entier naturel $n$ tel que :
    $\begin{align*} p(Y\pg 1)\pg 0,99 &\ssi 1-P(X=0)\pg 0,99 \\
    &\ssi P(X=0) \pp 0,01 \\
    &\ssi 0,65^n \pp 0,01 \\
    &\ssi n\ln(0,65) \pp \ln(0,01) \\
    &\ssi n\pg \dfrac{\ln(0,01)}{\ln(0,65)}\quad \text{car } \ln(0,65)>0\end{align*}$
    Or $\dfrac{\ln(0,01)}{\ln(0,65)}\approx 10,69$.
    La plus petite valeur de $n$ telle que la probabilité que Stéphanie réussisse au moins un tir parmi les $n$ tirs soit supérieure ou égale à $0,99$ est donc $11$.
    $\quad$

 

Ex 2

Exercice 2

  1. a. La fonction $f$ est dérivable sur $]0;+\infty[$ par hypothèse.
    Pour tout $x>0$ on a
    $\begin{align*} f'(x)&=\ln(x)+x\times \dfrac{1}{x}-1 \\
    &=\ln(x)+1-1\\
    &=\ln(x)\end{align*}$
    $\quad$
    b. On a $f(\e)=-2$ et $f'(\e)=1$.
    Une équation de la tangente $T$ est donc $y=1\times (x-\e)-2$ soit $y=x-\e-2$.
    $\quad$
    c. Par hypothèse la fonction $f$ est deux fois dérivables sur $]0;+\infty[$.
    Par conséquent, pour tout réel $x>0$ on a $f\dsec(x)=\dfrac{1}{x}>0$.
    La fonction $f$ est donc convexe sur $]0;+\infty[$.
    $\quad$
    d. La fonction $f$ est convexe sur $]0;+\infty[$. La courbe $\mathscr{C}_f$ est donc au-dessus de toutes ses tangentes.
    Ainsi $\mathscr{C}_f$ est au-dessus de $T$.
    $\quad$
  2. a. Par croissances comparées $\lim\limits_{x\to 0} x\ln(x)=0$. Donc $\lim\limits_{x\to 0} f(x)=-2$.
    $\quad$
    b. Pour tout réel $x>0$ on a $f(x)=x\left(\ln(x)-1-\dfrac{2}{x}\right)$.
    Or $\lim\limits_{x\to +\infty} \ln(x)=+\infty$ et $\lim\limits_{x\to +\infty} \dfrac{1}{x}=0$.
    Donc $\lim\limits_{x\to +\infty} f(x)=+\infty$.
    $\quad$
  3. $\ln(x)=0\ssi x=1$ et $\ln(x)>0 \ssi x>1$
    On obtient donc le tableau de variations suivant :
    $\quad$

    $\quad$
  4. a. Pour tout réel $x\in ]0;1]$ on a, d’après la question précédente, $f(x)<-2$. L’équation $f(x)=0$ n’admet donc aucune solution sur l’intervalle $]0;1]$.
    La fonction $f$ est continue (car dérivable) et strictement croissante sur l’intervalle $[1;+\infty[$.
    $f(1)=-3<0$ et $\lim\limits_{x\to +\infty} f(x)=+\infty$.
    D’après le théorème de la bijection (ou corollaire du théorème des valeurs intermédiaires) l’équation $f(x)=0$ admet une unique solution sur l’intervalle $[1;+\infty[$.
    Ainsi l’équation $f(x)=0$ admet une unique solution sur $]0;+\infty[$.
    $\quad$
    b. $f(4,3)\approx -0,03<0$ et $f(4,4)\approx 0,12>0$.
    Donc $f(4,3)<f(\alpha)<f(4,4)$.
    La fonction $f$ est strictement croissante sur l’intervalle $[4,3;4;4]$.
    Par conséquent $4,3<\alpha<4,4$.
    Ainsi $\alpha\in ]4,3;4,4[$.
    $\quad$
    c. D’après les questions précédentes :
    $\bullet$ $f(x)<0$ sur $]0;\alpha[$;
    $\bullet$ $f(\alpha)=0$;
    $\bullet$ $f(x)>0$ sur $]\alpha;+\infty[$.
    $\quad$
  5. $\texttt{seuil(0.01)}$ renvoie la valeur $4,32$.
    Il s’agit d’une valeur approchée à $10^{-2}$ près de $\alpha$.
    $\quad$

 

Ex 3

Exercice 3

  1. On a $B(6;4;0)$, $E(0;4;4)$, $F(6;4;4)$ et $G(6;0;4)$.
    $\quad$
  2. Le volume du toit est
    $\begin{align*}V_{pyramide}&=\dfrac{1}{3}\times 6\times 4\times (6-4) \\
    &=16\end{align*}$
    Le volume de $EFGHS$ est donc égale à $16$ u.v.
    Le volume du parallélépipède est :
    $\begin{align*} V_{parallélépipède}&=6\times 4\times 4\\
    &=96\end{align*}$
    Le volume de la maison est donc $V=16+96=112$ u.v.
    $\dfrac{16}{112}=\dfrac{1}{7}$
    Le volume de la pyramide $EFGHS$ représente bien le septième du volume total de la maison.
    $\quad$
  3. a. On a $\vect{EF}\begin{pmatrix} 6\\0\\0\end{pmatrix}$ et $\vect{ES}\begin{pmatrix}3\\-2\\2\end{pmatrix}$.
    Ces deux vecteurs sont clairement non colinéaires.
    Ainsi $\vec{n}.\vect{EF}=0+0+0=0$ et $\vec{n}.\vect{ES}=0-2+2=0$.
    Le vecteur $\vec{n}$ est donc orthogonal à deux vecteurs non colinéaires du plan $(EFS)$. Il est, par conséquent, normal au plan $(EFS)$.
    $\quad$
    b. Une équation cartésienne du plan $(EFS)$ est donc de la forme $y+z+d=0$.
    Le point $E(0;4;4)$ appartient au plan $(EFS)$.
    Donc $4+4+d=0 \ssi d=-8$.
    Une équation cartésienne du plan $(EFS)$ est donc $y+z-8=0$.
    $\quad$
  4. a. La droite $(PQ)$ est dirigée par $\vec{k}$ et passe par $Q(2;3;5,5)$.
    Une représentation paramétrique de la droite $(PQ)$ est donc $$\begin{cases} x=2\\y=3\\z=5,5+t\end{cases} \qquad t\in \R$$
    $\quad$
    b. Le point $P$ est le point d’intersection de la droite $(PQ)$ et du plan $(EFS)$. Déterminons les coordonnées de ce point à l’aide du système :
    $\begin{align*}\begin{cases} y+z-8=0 \\x=2\\y=3\\z=5,5+t\end{cases} &\ssi \begin{cases}x=2\\y=3\\z=5,5+t\\3+5,5+t-8=0\end{cases} \\
    &\ssi \begin{cases} x=2\\y=3\\t=-0,5\\z=5\end{cases}\end{align*}$
    Ainsi $P$ a pour coordonnées $(2;3;5)$.
    $\quad$
    c. On a alors $\vect{PQ}\begin{pmatrix}0\\0\\0,5\end{pmatrix}$.
    Ainsi $PQ=0,5$.
    $\quad$
  5. Un vecteur directeur de $\Delta$ est $\vec{u}\begin{pmatrix} 6\\-4\\4\end{pmatrix}$
    $\vec{k}$ et $\vec{u}$ ne sont pas colinéaires. Les droites $(PQ)$ et $\Delta$ ne sont donc pas parallèles.
    Déterminons si elles sont sécantes.
    $\begin{align*} \begin{cases} x=2\\y=3\\z=5,5+t\\x=-4+6s\\y=7-4s\\z=2+4s\end{cases}&\ssi \begin{cases} x=2\\y=3\\z=5,5+t\\-4+6s=2\\7-4s=3\\z=2+4s\end{cases} \\
    &\ssi \begin{cases} x=2\\y=3\\s=1\\z=2+4s\\z=5,5+t \end{cases} \\
    &\ssi \begin{cases} x=2\\y=3\\z=6\\s=1\\t=0,5\end{cases}\end{align*}$
    Les droites $(PQ)$ et $\Delta$ sont donc sécantes. Leur point d’intersection a pour coordonnées $(2;3;6)$.
    L’oiseau passe donc $0,5$ unité au-dessus de l’antenne. Par conséquent, il ne la percute pas.
    $\quad$

 

Ex 4

Exercice 4

  1. Pour tout $n\in \N$ on a $-1\pp (-1)^n \pp 1$ donc $-\dfrac{1}{n+1}\pp u_n \pp \dfrac{1}{n+1}$.
    Or $\lim\limits_{n\to +\infty} \dfrac{1}{n+1}=0$.
    D’après le théorème des gendarmes $\lim\limits_{n\to +\infty} u_n=0$.
    Réponse D
    $\quad$
  2. On a :
    $\begin{align*} w_0&=\e^{-2\ln(a)}+2 \\
    &=a^{-2}+2 \\
    &=\dfrac{1}{a^2}+2\end{align*}$
    Réponse A
    $\quad$
  3. La suite $\left(v_n\right)$ est décroissante.
    Pour tout $n\in \N$
    $\begin{align*} v_n\pp v_{n+1} &\ssi -2v_n\pg -2v_{n+1} \\
    &\ssi \e^{-2v_n}\pg \e^{-2v_{n+1}} \\
    &\ssi w_n\pg w_{n+1}\end{align*}$
    La suite $\left(w_n\right)$ est donc décroissante.
    La fonction exponentielle est strictement positive. Par conséquent, pour tout $n\in \N$, $\e^{-2v_n}>0$ et $w_n>2$.
    Réponse B
    $\quad$
  4. Montrons que la bonne réponse est la B.
    Il suffisait ici de calculer les premiers termes de chacune des $5$ suites pour déterminer que seule la proposition convenait.
    $-\dfrac{2}{3^0}+4=2$ ce qui correspond bien à $a_0=2$.
    $\begin{align*} -\dfrac{2}{3^{n+1}}+4&=\dfrac{1}{3}\times \dfrac{-2}{3^n}+4 \\
    &=\dfrac{1}{3}\left(-\dfrac{2}{3^n}+4-4\right)+4 \\
    &=\dfrac{1}{3}\left(-\dfrac{2}{3^n}+4\right)-\dfrac{4}{3}+4 \\
    &=\dfrac{1}{3}\left(-\dfrac{2}{3^n}+4\right)+\dfrac{8}{3}\end{align*}$
    On retrouve bien la relation de récurrence $a_{n+1}=\dfrac{1}{3}a_n+\dfrac{8}{3}$.
    Réponse B
    $\quad$
  5. Pour tout $n\in \N$ on a $b_{n+1}-b_n=\ln\left(\dfrac{2}{\left(b_n\right)^2+3}\right)$.
    Or $\left(b_n\right)^2+3>2$ donc $\ln\left(\dfrac{2}{\left(b_n\right)^2+3}\right)<0$.
    La suite $\left(b_n\right)$ est par conséquent décroissante.
    Réponse B
    $\quad$
  6. $\lim\limits_{x\to 0^+} \dfrac{1}{x}=+\infty$ donc $\lim\limits_{x\to 0^+} g(x)=+\infty$.
    La droite d’équation $x=0$ est asymptote à la courbe $\mathscr{C}_g$.
    Par croissances comparées, $\lim\limits_{x\to +\infty} g(x)=+\infty$.
    La courbe $\mathscr{C}_g$ ne possède pas d’asymptote horizontale.
    Réponse B
    $\quad$
  7. On considère la fonction $F$ définie sur $\R$ par $F(x)=\dfrac{1}{2}\e^{x^2+1}$
    $F$ est dérivable sur $\R$ en tant que composée de fonctions dérivables sur $\R$.
    Pour tout réel $x$ on a
    $\begin{align*} F'(x)&=\dfrac{1}{2}\times 2x\e^{x^2+1} \\
    &=f(x)\end{align*}$
    Réponse D
    $\quad$
    Remarque : On pouvait également déterminer, à vue, une primitive de $f$. En effet, pour tout réel $x$, on a
    $\begin{align*}f(x)&=x\e^{x^2+1} \\
    &=\dfrac{1}{2}\times 2x\e^{x^2+1}\end{align*}$
    Ainsi $f(x)$ est de la forme $\dfrac{1}{2}u'(x)\e^{u(x)}$ où $u(x)=x^2+1$.
    Une primitive de $f$ est donc la fonction $F$ définie sur $\R$ par $F(x)=\dfrac{1}{2}\e^{u(x)}$ soit $F(x)=\dfrac{1}{2}\e^{x^2+1}$.
    $\quad$

 

Énoncé

Le sujet propose 4 exercices.
Le candidat choisit 3 exercices parmi les 4 et ne doit traiter que ces 3 exercices.

Exercice 1     7 points

Principaux domaines abordés : probabilités

Au basket-ball, il existe deux sortes de tir :

  • les tirs à deux points.
    Ils sont réalisés près du panier et rapportent deux points s’ils sont réussis.
  • les tirs à trois points.
    Ils sont réalisés loin du panier et rapportent trois points s’ils sont réussis.

Stéphanie s’entraîne au tir. On dispose des données suivantes :

  • Un quart de ses tirs sont des tirs à deux points. Parmi eux, $60 \%$ sont réussis.
  • Trois quarts de ses tirs sont des tirs à trois points. Parmi eux, $35\%$ sont réussis.
  1. Stéphanie réalise un tir.
    On considère les évènements suivants :
    $D$ : « Il s’agit d’un tir à deux points ».
    $R$ : « le tir est réussi ».
    a. Représenter la situation à l’aide d’un arbre de probabilités.
    $\quad$
    b. Calculer la probabilité $p\left(\conj{D} \cap R\right)$.
    $\quad$
    c. Démontrer que la probabilité que Stéphanie réussisse un tir est égale à $0,412~5$.
    $\quad$
    d. Stéphanie réussit un tir. Calculer la probabilité qu’il s’agisse d’un tir à trois points.
    Arrondir le résultat au centième.
    $\quad$
  2. Stéphanie réalise à présent une série de $10$ tirs à trois points.
    On note $X$ la variable aléatoire qui compte le nombre de tirs réussis.
    On considère que les tirs sont indépendants. On rappelle que la probabilité que Stéphanie réussisse un tir à trois points est égale à $0,35$.
    a. Justifier que $X$ suit une loi binomiale. Préciser ses paramètres.
    $\quad$
    b. Calculer l’espérance de $X$. Interpréter le résultat dans le contexte de l’exercice.
    $\quad$
    c. Déterminer la probabilité que Stéphanie rate $4$ tirs ou plus. Arrondir le résultat au centième.
    $\quad$
    d. Déterminer la probabilité que Stéphanie rate au plus $4$ tirs. Arrondir le résultat au centième.
    $\quad$
  3. Soit $n$ un entier naturel non nul.
    Stéphanie souhaite réaliser une série de $n$ tirs à trois points.
    On considère que les tirs sont indépendants. On rappelle que la probabilité qu’elle réussisse un tir à trois points est égale à $0,35$.
    Déterminer la valeur minimale de $n$ pour que la probabilité que Stéphanie réussisse au moins un tir parmi les n tirs soit supérieure ou égale à $0,99$.
    $\quad$

$\quad$

Exercice 2     7 points

Principaux domaines abordés : fonctions, fonction logarithme.

Soit $f$ la fonction définie sur l’intervalle $]0 ; +\infty[$ par :
$$f(x) = x\ln(x)-x-2$$
On admet que la fonction $f$ est deux fois dérivable sur $]0 ; +\infty[$.
On note $f’$ sa dérivée, $f\dsec$ sa dérivée seconde et $\mathscr{C}_f$ sa courbe représentative dans un repère.

  1. a. Démontrer que, pour tout $x$ appartenant à $]0 ; +\infty[$, on a $f'(x) = \ln(x)$.
    $\quad$
    b. Déterminer une équation de la tangente $T$ à la courbe $\mathscr{C}_f$ au point d’abscisse $x =\e$.
    $\quad$
    c. Justifier que la fonction $f$ est convexe sur l’intervalle $]0 ; +\infty[$.
    $\quad$
    d. En déduire la position relative de la courbe $\mathscr{C}_f$ et de la tangente $T$.
    $\quad$
  2. a. Calculer la limite de la fonction $f$ en $0$.
    $\quad$
    b. Démontrer que la limite de la fonction $f$ en $+\infty$ est égale à $+\infty$.
    $\quad$
  3. Dresser le tableau de variations de la fonction $f$ sur l’intervalle $]0 ; +\infty[$.
    $\quad$
  4. a. Démontrer que l’équation $f (x) = 0$ admet une unique solution dans l’intervalle $]0 ; +\infty[$. On note $\alpha$ cette solution.
    $\quad$
    b. Justifier que le réel $\alpha$ appartient à l’intervalle $]4,3; 4,4[$.
    $\quad$
    c. En déduire le signe de la fonction $f$ sur l’intervalle $]0 ; +\infty[$.
    $\quad$
  5. On considère la fonction $\texttt{seuil}$ suivante écrite dans le langage Python :
    On rappelle que la fonction $\texttt{log}$ du module $\texttt{math}$ (que l’on suppose importé) désigne
    la fonction logarithme népérien $\ln$.$$\begin{array}{|l|}
    \hline
    \text{def seuil(pas) :}\\
    \quad  \text{x=4.3}\\
    \quad  \text{while x*log (x) – x – 2 < 0:}\\
    \qquad  \text{x=x+pas}\\
    \quad  \text{return x}\\
    \hline
    \end{array}$$
    Quelle est la valeur renvoyée à l’appel de la fonction $\texttt{seuil(0.01)}$?
    Interpréter ce résultat dans le contexte de l’exercice.
    $\quad$

$\quad$

Exercice 3     7 points

Principaux domaines abordés : géométrie dans l’espace

Une maison est modélisée par un parallélépipède rectangle $ABCDEFGH$ surmonté d’une pyramide $EFGHS$.
On a $DC = 6$, $DA = DH = 4$.
Soit les points $I$, $J$ et $K$ tels que $\vect{DI}=\dfrac{1}{6}\vect{DC}$, $\vect{DJ}=\dfrac{1}{4}\vect{DA}$, $\vect{DK}=\dfrac{1}{4}\vect{DH}$.
On note $\vec{i}=\vect{DI}$, $\vec{j}=\vect{DJ}$, $\vec{k}=\vect{DK}$.
On se place dans le repère orthonormé $\left(D;\vec{i},\vec{j},\vec{k}\right)$.
On admet que le point $S$ a pour coordonnées $(3; 2; 6)$.

  1. Donner, sans justifier, les coordonnées des points $B$, $E$, $F$ et $G$.
    $\quad$
  2. Démontrer que le volume de la pyramide $EFGHS$ représente le septième du volume total de la maison.
    On rappelle que le volume $V$ d’un tétraèdre est donné par la formule : $$V =\dfrac{1}{3}\times \text{(aire de la base)}\times \text{hauteur}$$
    $\quad$
  3. a. Démontrer que le vecteur $\vec{n}$ de coordonnées $\begin{pmatrix}0\\1\\1\end{pmatrix}$ est normal au plan $(EFS)$.
    $\quad$
    b. En déduire qu’une équation cartésienne du plan $(EFS)$ est $y +z-8 = 0$.
    $\quad$
  4. On installe une antenne sur le toit, représentée par le  segment $[PQ]$. On dispose des
    données suivantes :
    $\bullet$ le point $P$ appartient au plan $(EFS)$;
    $\bullet$ le point $Q$ a pour coordonnées $(2; 3; 5,5)$;
    $\bullet$ la droite $(PQ)$ est dirigée par le vecteur $\vec{k}$.
    a. Justifier qu’une représentation paramétrique de la droite $(PQ)$ est :
    $$\begin{cases}x=2\\y = 3\\z = 5,5+t\end{cases} \quad (t \in \R)$$
    b. En déduire les coordonnées du point $P$.
    $\quad$
    c. En déduire la longueur $PQ$ de l’antenne.
    $\quad$
  5. Un oiseau vole en suivant une trajectoire modélisée par la droite $\Delta$ dont une représentation paramétrique est : $$\begin{cases} x=-4+6s\\y=7-4s\\z=2+4s\end{cases} \quad (s\in \R)$$
    Déterminer la position relative des droites $(PQ)$ et $\Delta$.
    L’oiseau va-t-il percuter l’antenne représentée par le segment $[PQ]$?
    $\quad$

$\quad$

Exercice 4     7 points

Principaux domaines abordés : : suites, fonctions, primitives

Cet exercice est un questionnaire à choix multiples.
Pour chacune des questions suivantes, une seule des quatre réponses proposées est exacte.
Une réponse fausse, une réponse multiple ou l’absence de réponse à une question ne rapporte ni n’enlève de point.
Pour répondre, indiquer sur la copie le numéro de la question et la lettre de la réponse choisie.
Aucune justification n’est demandée

  1. On considère la suite $\left(u_n\right)$ définie pour tout entier naturel $n$ par $$u_n=\dfrac{(-1)^n}{n+1}$$
    On peut affirmer que :
    a. la suite $\left(u_n\right)$ diverge vers $+\infty$.
    b. la suite $\left(u_n\right)$ diverge vers $-\infty$.
    c. la suite $\left(u_n\right)$ n’a pas de limite.
    d. la suite $\left(u_n\right)$ converge.
    $\quad$

Dans les questions 2 et 3, on considère deux suites $\left(v_n\right)$ et $\left(w_n\right)$ vérifiant la relation : $$w_n=\e^{-2v_n}+2$$

  1. . Soit $a$ un nombre réel strictement positif. On a $v_0 = \ln(a)$.
    a. $w_0=\dfrac{1}{a^2}+2$
    b. $w_0=\dfrac{1}{a^2+2}$
    c. $w_0=-2a+2$
    d. $w_0=\dfrac{1}{-2a}+2$
    $\quad$
  2. On sait que la suite $\left(v_n\right)$ est croissante. On peut affirmer que la suite $\left(w_n\right)$ est :
    a. décroissante et majorée par $3$.
    b. décroissante et minorée par $2$.
    c. croissante et majorée par $3$.
    d. croissante et minorée par $2$.
    $\quad$
  3. On considère la suite $\left(a_n\right)$ ainsi définie : $$a_0=2 \text{ et, pour tout entier naturel }n,~~a_{n+1}=\dfrac{1}{3}a_n+\dfrac{8}{3}$$
    Pour tout entier naturel $n$, on a :
    a. $a_n=4\times \left(\dfrac{1}{3}\right)^n-2$
    b. $a_n=-\dfrac{2}{3^n}+4$
    c. $a_n=4-\left(\dfrac{1}{3}\right)^n$
    d. $a_n=2\times \left(\dfrac{1}{3}\right)^n+\dfrac{8n}{3}$
    $\quad$
  4. On considère une suite $\left(b_n\right)$ telle que, pour tout entier naturel $n$, on a : $$b_{n+1}=b_n+\ln\left(\dfrac{2}{\left(b_n\right)^2+3}\right)$$
    On peut affirmer que :
    a. la suite $\left(b_n\right)$ est croissante.
    b. la suite $\left(b_n\right)$ est décroissante.
    c. la suite $\left(b_n\right)$ n’est pas monotone.
    d. le sens de variation de la suite $\left(b_n\right)$ dépend de $b_0$.
    $\quad$
  5. On considère la fonction $g$ définie sur l’intervalle $]0 ; +\infty[$ par : $$g(x)=\dfrac{\e^x}{x}$$
    On note $\mathscr{C}_g$ la courbe représentative de la fonction $g$ dans un repère orthogonal.
    La courbe $\mathscr{C}_g$ admet :
    a. une asymptote verticale et une asymptote horizontale.
    b. une asymptote verticale et aucune asymptote horizontale.
    c. aucune asymptote verticale et une asymptote horizontale.
    d. aucune asymptote verticale et aucune asymptote horizontale.
    $\quad$
  6. Soit $f$ la fonction définie sur $\R$ par $$f(x)=x\e^{x^2+1}$$
    Soit $F$ une primitive sur $\R$ de la fonction $f$. Pour tout réel $x$, on a :
    a. $F(x)=\dfrac{1}{2}x^2\e^{x^2+1}$
    b. $F(x)=\left(1+2x^2\right)\e^{x^2+1}$
    c. $F(x)=\e^{x^2+1}$
    d. $F(x)=\dfrac{1}{2}\e^{x^2+1}$
    $\quad$

$\quad$

 

Bac – Spécialité mathématiques – Nouvelle Calédonie- sujet 1 – 26 octobre 2022

Nouvelle Calédonie – 26 octobre 2022

Spécialité maths – Sujet 1 – Correction

L’énoncé de ce sujet de bac est disponible ici.

Ex 1

Exercice 1

  1. a.$\lim\limits_{x\to 0} x^2-6x=0$ et $\lim\limits_{x\to 0} \ln(x)=-\infty$ donc $\lim\limits_{x\to 0} f(x)=-\infty$.
    La droite d’équation $x=0$ est donc asymptote à la courbe $\mathscr{C}_f$.
    $\quad$
    b. Pour tout réel $x$ on a $f(x)=x^2\left(1-\dfrac{6}{x}+\dfrac{4\ln(x)}{x^2}\right)$.
    Or $\lim\limits_{x\to +\infty} \dfrac{1}{x}=0$ et, par croissances comparées, $\lim\limits_{x\to +\infty} \dfrac{\ln(x)}{x^2}=0$.
    De plus $\lim\limits_{x\to +\infty} x^2=+\infty$.
    Donc $\lim\limits_{x\to +\infty} f(x)=+\infty$.
    $\quad$
  2. a. La fonction $f$ est dérivable sur $]0;+\infty[$ par hypothèse.
    Pour tout réel $x>0$ on a:
    $\begin{align*} f'(x)&=2x-6+\dfrac{4}{x} \\
    &=\dfrac{2x^2-6x+4}{x} \\
    &=\dfrac{2\left(x^2-3x+2\right)}{x}\end{align*}$
    $\quad$
    b. Le signe de $f'(x)$ ne dépend que de celui-ci de $x^2-3x+2$.
    Il s’agit d’un polynôme du second degré dont le discriminant est $\Delta=1>0$.
    Les racines de ce polynômes sont :
    $x_1=\dfrac{3+\sqrt{1}}{2}=2$ et $x_2=\dfrac{3-\sqrt{1}}{2}=1$.
    Le coefficient principal du polynôme est $a=1>0$.
    Ainsi :
    $\bullet$ $f'(x)>0$ sur $]0;1[$;
    $\bullet$ $f'(1)=0$;
    $\bullet$ $f'(x)<0$ sur $]1;2[$;
    $\bullet$ $f'(2)=0$;
    $\bullet$ $f'(x)>0$ sur $]2;+\infty[$.
    On obtient donc le tableau de variations suivant :
    $\quad$

    $\quad$
    $f(2)=-8+4\ln(2)$
    $\quad$
  3. La fonction $f$ est continue (car dérivable) et strictement croissante sur l’intervalle $[4;5]$.
    De plus $f(4)\approx -2,45<0$ et $f(5)\approx 1,44>0$
    D’après le théorème de la bijection (ou corollaire du théorème des valeurs intermédiaires) l’équation $f(x)=0$ admet une unique solution sur $[4;5]$.
    $\quad$
  4. a. Pour tout $x>0$
    $\begin{align*} f\dsec(x)>0 &\ssi 2x^2-4>0 \\
    &\ssi x^2>2 \\
    &\ssi x>\sqrt{2}\end{align*}$
    La fonction $f$ est donc concave sur $\left]0;\sqrt{2}\right]$ et convexe sur $\left[\sqrt{2};+\infty\right[$.
    De plus $f\dsec\left(\sqrt{2}\right)=0$ et $f\left(\sqrt{2}\right)=2-6\sqrt{2}+2\ln(2)$.
    Ainsi, $\mathscr{C}_f$ admet un unique point d’inflexion de coordonnées $\left(\sqrt{2};2-6\sqrt{2}+2\ln(2)\right)$.
    $\quad$
    b. La fonction $f$ est concave sur $\left]0;\sqrt{2}\right]$. La courbe $\mathscr{C}_f$ est donc au-dessus de ses cordes sur cet intervalle.
    La fonction $f$ est convexe sur $\left[\sqrt{2};+\infty\right[$. La courbe $\mathscr{C}_f$ est donc au-dessous de ses cordes sur cet intervalle.
    Ainsi :
    – $\mathscr{C}_f$ est au-dessus de $[AM]$ sur $\left]0;\sqrt{2}\right[$.
    – $\mathscr{C}_f$ est au-dessous de $[AM]$ sur $\left]\sqrt{2};+\infty[\right[$.
    $\quad$

 

 

Ex 2

Exercice 2

  1. a. $u_1=-\e^{-1}\approx -0,368$ et $u_2=-\e^{-3-\e^{-1}}\approx -0,034$.
    $\quad$
    b. $\texttt{fonc(2)}$ renvoie la valeur de $u_2$ c’est-à-dire environ $0,034$.
    $\quad$
  2. a. Par hypothèse $f$ est dérivable sur $\R$.
    Pour tout réel $x$ on a
    $\begin{align*} f'(x)&=3x^2\e^x+x^3\e^x \\
    &=x^2\e^x(3+x)\end{align*}$
    $\quad$
    b. La fonction exponentielle est strictement positive sur $\R$.
    Le signe de $f'(x)$ ne dépend donc que de celui de $x+3$.
    Or $x+3=0 \ssi x=-3$ et $x+3>0 \ssi x>-3$.
    La fonction $f$ est donc strictement décroissante sur $]-\infty;-3]$ et strictement croissante sur $[-3;+\infty[$.
    De plus, par croissances comparées $\lim\limits_{x\to -\infty} x^3\e^{-x}=0$.
    $\lim\limits_{x\to +\infty} x^3=+\infty$ et $\lim\limits_{x\to +\infty} \e^x=+\infty$ donc $\lim\limits_{x\to +\infty} f(x)=+\infty$.
    $\begin{align*} f(-3)&=(-3)^3\e^{-3} \\
    &=-27\e^{-3}\end{align*}$
    On a ainsi justifié chacun des éléments du tableau de variations.
    $\quad$
    c. Pour tout $n\in \N$ on pose $P(n):~-1\pp u_n\pp u_{n+1} \pp 0$.
    Initialisation : $u_0=-1$ et $u_1\approx -0,368$.
    On a donc bien $-1\pp u_0\pp u_1 \pp 0$ et $P(0)$ est vraie.
    $\quad$
    Hérédité : Soit $n\in \N$. On suppose que $P(n)$ est vraie.
    $-1\pp u_n \pp u_{n+1} \pp 0$.
    La fonction $f$ est strictement croissante sur $[-1;0]$.
    Par conséquent $f(-1) \pp f\left(u_n\right)\pp f\left(u_{n+1}\right) \pp f(0)$
    Or $f(-1) \approx -0,368$ et $f(0)=0$.
    Ainsi $-1\pp u_{n+1} \pp u_{n+2}\pp 0$.
    $P(n+1)$ est donc vraie.
    $\quad$
    Conclusion : D’après le principe de récurrence, pour tout $n\in \N$ on a $-1\pp u_n\pp u_{n+1} \pp 0$.
    $\quad$
    d. La suite $\left(u_n\right)$ est croissante et majorée par $0$. Elle converge donc vers un réel $\ell$.
    $\quad$
    e. On a $f(x)=x\ssi x^3\e^x=x \ssi x\left(x^2\e^x-1\right)=0 \ssi x=0$ ou $x^2\e^x-1=0$.
    Or l’équation $x^2\e^x-1=0$ possède une unique solution supérieure à $\dfrac{1}{2}$ et on sait que $-1\pp \ell \pp 0$.
    Ainsi $\ell=0$.
    $\quad$

 

Ex 3

Exercice 3

  1. Le point $G$ a pour coordonnées $(3;2;1)$.
    $\quad$
  2. Une équation cartésienne du plan $(EHI)$ est donc de la forme $2x-3z+d=0$.
    Le point $E(0;0;1)$ appartient à ce plan donc $0-3+d=0\ssi d=3$.
    Une équation cartésienne du plan $(EHI)$ est par conséquent $2x-3z+3=0$.
    $\quad$
  3. Le triangle $EIF$ est isocèle en $I$ et $\vect{EF}=\vect{AB}$. Par conséquent l’abscisse de $I$ est $\dfrac{AB}{2}=1,5$.
    Sa côte, $z_I$ vérifie $2\times 1,5-3z_I+3=0 \ssi 3z_I=6 \ssi z_I=2$.
    De plus $I$ appartient au plan $(ABE)$ dont une équation cartésienne est $y=0$.
    Ainsi $I$ a pour coordonnées $(1,5;0;2)$.
    $\quad$
  4. On a $\vect{IE}(-1,5;0;-1)$ et $\vect{IF}(1,5;0;-1)$
    Par conséquent
    $\begin{align*} IE&=\sqrt{(-1,5)^2+(-1)^2} \\
    &=\sqrt{3,25}\end{align*}$
    et $IF=IE=\sqrt{3,25}$.
    D’une part $\vect{IE}.\vect{IF}=-1,5\times 1,5+(-1)\times (-1)=-1,25$
    D’autre part $\vect{IE}.\vect{IF}=IE\times IF\times \cos\widehat{EIF}$
    Ainsi $3,25 \cos\widehat{EIF}=-1,25 \ssi \cos\widehat{EIF}=-\dfrac{5}{13}$
    Donc $\widehat{EIF}\approx 113$°.
    $\quad$
  5. a. La droite $\Delta$ est dirigée par $\vec{u}$ et passe par $R$.
    Une représentation paramétrique de la droite $\Delta$ est donc $$\begin{cases} x=6-3t\\y=-3+4t\\z=-1+t\end{cases}$$
    $\quad$
    b. Le point $K$ appartient au plan $(BFG)$ par conséquent son abscisse est $x_K=3$.
    Le point $K$ appartient à la droite $\Delta$ donc $6-3t=3 \ssi t=1$.
    Ainsi $K$ a pour coordonnées $(3;1;0)$.
    $\quad$
    c. On a $C(3;2;0)$ et $B(3;0;0)$. Donc $K$ est le milieu de $[BC]$ et appartient donc bien à l’arête $[BC]$

 

Ex 4

Exercice 4

  1. On veut calculer
    $\begin{align*} p\left(E_0\cap R_0\right)&=p\left(E_0\right)\times p_{E_0}\left(R_0\right) \\
    &=0,4\times (1-0,01) \\
    &=0,4\times 0,99 \\
    &=0,396\end{align*}$
    Réponse B
    $\quad$
  2. $\left(E_0,E_1\right)$ forme un système complet d’événements fini.
    D’après la formule des probabilités totales on a
    $\begin{align*} p\left(R_0\right)&=p\left(E_0\cap R_0\right)+p\left(E_1\cap R_0\right) \\
    &=0,396+p\left(E_1\right)\times p_{E_1}\left(R_0\right) \\
    &=0,396+0,6\times 0,02 \\
    &=0,408\end{align*}$
    Réponse C
    $\quad$
  3. On a
    $\begin{align*} p_{R_1}\left(E_0\right)&=\dfrac{p\left(R_1\cap E_0\right)}{p\left(R_1\right)} \\
    &=\dfrac{p\left(E_0\right)\times p_{E_0}\left(R_1\right)}{1-p\left(R_0\right)}\\
    &=\dfrac{0,4\times 0,01}{1-0,408} \\
    &\approx 0,006~757\end{align*}$
    Réponse C$\quad$
  4. La probabilité qu’il y ait une erreur de transmission est :
    $\begin{align*} p\left(\left(E_0\cap R_1\right)\cup\left(E_1\cap R_0\right)\right)&=p\left(E_0\cap R_1\right)+p\left(E_1\cap R_0\right) \\
    &=p\left(E_0\right)p_{E_0}\left(R_1\right)+p\left(E_1\right)p_{E_1}\left(R_0\right) \\
    &=0,4\times 0,01+0,6\times 0,02 \\
    &=0,016\end{align*}$
    Réponse B
    $\quad$
  5. On appelle $X$ la variable aléatoire comptant le nombre d’octets transmis sans erreur.
    On effectue indépendamment $10$ expérience de Bernoulli de paramètres $0,88$.
    $X$ suit la loi binomiale de paramètres $n=10$ et $p=0,88$.
    Ainsi
    $\begin{align*} p(X=7)&=\dbinom{10}{7}0,88^7\times 0,12^3 \\
    &\approx 0,085\end{align*}$
    Réponse D
    $\quad$
  6. On reprend la variable aléatoire $X$ définie à la question précédente.
    On veut calculer
    $\begin{align*} p(X\pg 1)&=1-p(X=0) \\
    &=1-0,12^{10} \end{align*}$
    Réponse A
    $\quad$
  7. On appelle $Y$ la variable aléatoire comptant le nombre d’octets transmis sans erreur.
    On effectue indépendamment $N$ expérience de Bernoulli de paramètres $0,88$.
    $Y$ suit la loi binomiale de paramètres $N$ et $p=0,88$.
    On veut déterminer la plus grande valeur de $N$ telle que
    $\begin{align*} P(X=N)\pg 0,1 &\ssi 0,88^N\pg 0,1 \\
    &\ssi N\ln(0,88) \pg \ln(0,1) \\
    &\ssi N\pp \dfrac{\ln(0,1)}{\ln(0,88)} \quad \text{car } \ln(0,88)<0 \end{align*}$
    Or $\dfrac{\ln(0,1)}{\ln(0,88)}\approx 18,01$.
    Par conséquent $N_0=18$.
    Réponse B
    $\quad$

 

 

Énoncé

Le sujet propose 4 exercices.
Le candidat choisit 3 exercices parmi les 4 et ne doit traiter que ces 3 exercices.

Exercice 1     7 points

Principaux domaines abordés : fonctions, fonction logarithme; convexité.

On considère la fonction $f$ définie sur l’intervalle $]0 ; +\infty[$ par $$f(x)=x^2-6x+4\ln(x)$$

On admet que la fonction $f$ est deux fois dérivable sur l’intervalle $]0 ; +\infty[$.
On note $f’$ sa dérivée et $f\dsec$ sa dérivée seconde.
On note $\mathscr{C}_f$ la courbe représentative de la fonction $f$ dans un repère orthogonal.

  1. a. Déterminer $\lim\limits_{x\to 0} f (x)$.
    Interpréter graphiquement ce résultat.
    $\quad$
    b. Déterminer $\lim\limits_{x\to +\infty} f(x)$.
    $\quad$
  2. a. Déterminer $f'(x)$ pour tout réel $x$ appartenant à $]0;+\infty[$.
    $\quad$
    b. Étudier le signe de $f'(x)$ sur l’intervalle $]0;+\infty[$.
    En déduire le tableau de variations de $f$.
    $\quad$
  3. Montrer que l’équation $f (x) = 0$ admet une unique solution dans l’intervalle $[4; 5]$.
    $\quad$
  4. On admet que, pour tout $x$ de $]0 ; +\infty[$, on a : $$f\dsec(x)=\dfrac{2x^2-4}{x^2}$$
    a. Étudier la convexité de la fonction $f$ sur $]0 ; +\infty[$.
    On précisera les valeurs exactes des coordonnées des éventuels points d’inflexion de $\mathscr{C}_f$.
    $\quad$
    b. On note $A$ le point de coordonnées $\left(\sqrt{2};f\left(\sqrt{2}\right)\right)$.
    Soit $t$ un réel strictement positif tel que $t\neq \sqrt{2}$. Soit $M$ le point de coordonnées $\left(t ; f (t)\right)$.
    En utilisant la question 4. a, indiquer, selon la valeur de $t$, les positions relatives du segment $[AM]$ et de la courbe $\mathscr{C}_f$.
    $\quad$

$\quad$

Exercice 2     7 points

Principaux domaines abordés : suites; fonctions, fonction exponentielle

On considère la fonction $f$ définie sur $\R$ par $$f(x) = x^3\e^x$$
On admet que la fonction $f$ est dérivable sur $\R$ et on note $f’$
sa fonction dérivée.

  1. On définit la suite $\left(u_n\right)$ par $u_0 = -1$ et, pour tout entier naturel $n$, $u_{n+1} = f\left(u_n\right)$.
    a. Calculer $u_1$ puis $u_2$.
    On donnera les valeurs exactes, puis les valeurs approchées à $10^{-3}$.
    $\quad$
    b. On considère la fonction $\texttt{fonc}$, écrite en langage Python ci-dessous.
    $$\begin{array}{|l|}
    \hline
    \text{def fonc(n) :}\\
    \quad \text{u =- 1}\\
    \quad \text{for i in range(n) :}\\
    \qquad \text{u=u**3*exp(u)}\\
    \quad \text{return u}\\
    \hline
    \end{array}$$
    On rappelle qu’en langage Python, « $\texttt{i in range (n)}$ » signifie que $\texttt{i}$ varie de $\texttt{0}$ à $\texttt{n-1}$.
    $\quad$
    Déterminer, sans justifier, la valeur renvoyée par $\texttt{fonc(2)}$ arrondie à $10^{-3}$.
    $\quad$
  2. a. Démontrer que, pour tout $x$ réel, on a $f'(x) = x^2\e^x(x+3)$.
    $\quad$
    b. Justifier que le tableau de variations de $f$ sur $\R$ est celui représenté ci-dessous :
    $\quad$

    $\quad$
    c. Démontrer, par récurrence, que pour tout entier naturel $n$, on a : $$-1 \pp u_n \pp u_{n+1} \pp 0$$
    $\quad$
    d. En déduire que la suite $\left(u_n\right)$ est convergente.
    $\quad$
    e. On note $\ell$ la limite de la suite $\left(u_n\right)$.
    On rappelle que $\ell$ est solution de l’équation $f(x) = x$.
    Déterminer $\ell$. $\Big($Pour cela, on admettra que l’équation $x^2\e^x-1 = 0$ possède une
    seule solution dans $\R$ et que celle-ci est strictement supérieure à $\dfrac{1}{2}\Big)$.
    $\quad$

$\quad$

Exercice 3     7 points

Principaux domaines abordés : géométrie dans l’espace.

Une maison est constituée d’un parallélépipède rectangle $ABCDEFGH$ surmonté d’un prisme $EFIHGJ$ dont une base est le triangle $EIF$ isocèle en $I$.
Cette maison est représentée ci-dessous.

On a $AB = 3$, $AD = 2$, $AE = 1$.
On définit les vecteurs $\vec{i}=\dfrac{1}{3}\vect{AB}$, $\vec{j}=\dfrac{1}{2}\vect{AD}$, $\vec{k}=\vect{AE}$.
On munit ainsi l’espace du repère orthonormé $\left(A;~\vec{i},~\vec{j},~\vec{k}\right)$.

  1. Donner les coordonnées du point $G$.
    $\quad$
  2. Le vecteur $\vec{n}$ de coordonnées $(2 ; 0 ; -3)$ est vecteur normal au plan $(EHI)$.
    Déterminer une équation cartésienne du plan $(EHI)$.
    $\quad$
  3. Déterminer les coordonnées du point $I$.
    $\quad$
  4. Déterminer une mesure au degré près de l’angle $\widehat{EIF}$.
    $\quad$
  5. Afin de raccorder la maison au réseau électrique, on souhaite creuser une tranchée rectiligne depuis un relais électrique situé en contrebas de la maison.
    Le relais est représenté par le point $R$ de coordonnées $(6 ; -3 ; -1)$.
    La tranchée est assimilée à un segment d’une droite $\Delta$ passant par $R$ et dirigée par le vecteur $\vec{u}$ de coordonnées $(-3 ; 4 ; 1)$. On souhaite vérifier que la tranchée atteindra la maison au niveau de l’arête $[BC]$.
    a. Donner une représentation paramétrique de la droite $\Delta$.
    $\quad$
    b. On admet qu’une équation du plan $(BFG)$ est $x = 3$.
    Soit $K$ le point d’intersection de la droite $\Delta$ avec le plan $(BFG)$.
    Déterminer les coordonnées du point $K$.
    $\quad$
    c. Le point $K$ appartient-il bien à l’arête $[BC]$ ?
    $\quad$

$\quad$

Exercice 4     7 points

Principaux domaines abordés : probabilités.

Cet exercice est un questionnaire à choix multiples.
Pour chacune des questions suivantes, une seule des quatre réponses proposées est exacte.
Une réponse fausse, une réponse multiple ou l’absence de réponse à une question ne rapporte ni n’enlève de point.

On considère un système de communication binaire transmettant des $0$ et des $1$.
Chaque $0$ ou $1$ est appelé bit.
En raison d’interférences, il peut y avoir des erreurs de transmission :
un $0$ peut être reçu comme un $1$ et, de même, un $1$ peut être reçu comme un $0$.
Pour un bit choisi au hasard dans le message, on note les évènements :

  • $E_0$ : « le bit envoyé est un $0$ »;
  • $E_1$ : « le bit envoyé est un $1$ »;
  • $R_0$ : « le bit reçu est un $0$ »;
  • $R_1$ : « le bit reçu est un $1$ ».

On sait que : $p\left(E_0\right) = 0,4$; $p_{E_0}\left(R_1\right)=0,01$; $p_{E_1}\left(R_0\right)=0,02$.
On rappelle que la probabilité conditionnelle de $A$ sachant $B$ est notée $p_B(A)$.
On peut ainsi représenter la situation par l’arbre de probabilités ci-dessus.

  1. La probabilité que le bit envoyé soit un $0$ et que le bit reçu soit un $0$ est égale à :
    a. $0,99$
    b. $0,396$
    c. $0,01$
    d. $0,4$
    $\quad$
  2. La probabilité $p\left(R_0\right)$ est égale à :
    a. $0,99$
    b. $0,02$
    c. $0,408$
    d. $0,931$
    $\quad$
  3. Une valeur, approchée au millième, de la probabilité $p_{R_1}
    \left(E_0\right)$ est égale
    a. $0,004$
    b. $0,001$
    c. $0,007$
    d. $0,010$
    $\quad$
  4. La probabilité de l’évènement « il y a une erreur de transmission » est égale à :
    a. $0,03$
    b. $0,016$
    c. $0,16$
    d. $0,015$
    $\quad$

Un message de longueur huit bits est appelé un octet.
On admet que la probabilité qu’un octet soit transmis sans erreur est égale à $0,88$.

  1. On transmet successivement $10$ octets de façon indépendante.
    La probabilité, à $10^{-3}$ près, qu’exactement $7$ octets soient transmis sans erreur est égale à :
    a. $0,915$
    b. $0,109$
    c. $0,976$
    d. $0,085$
    $\quad$
  2. On transmet successivement $10$ octets de façon indépendante.
    La probabilité qu’au moins $1$ octet soit transmis sans erreur est égale à :
    a. $1-0,12^{10}$
    b. $0,12^{10}$
    c. $0,88^{10}$
    d. $1-0,88^{10}$
    $\quad$
  3. Soit $N$ un entier naturel. On transmet successivement $N$ octets de façon indépendante.
    Soit $N_0$ la plus grande valeur de $N$ pour laquelle la probabilité que les $N$ octets soient tous transmis sans erreur est supérieure ou égale à $0,1$.
    On peut affirmer que :
    a. $N_0 = 17$
    b. $N_0 = 18$
    c. $N_0 = 19$
    d. $N_0 = 20$
    $\quad$

$\quad$

 

Bac – Spécialité mathématiques – Métropole, Antilles, Guyane – sujet 2 – 9 septembre 2022

Métropole Antilles/Guyane – 9 septembre 2022

Spécialité maths – Sujet 2 – Correction

L’énoncé de ce sujet de bac est disponible ici.

Ex 1

Exercice 1

  1. On obtient l’arbre pondéré suivant :
    $\quad$$\quad$
  2. a. $\left(R,\conj{R}\right)$ forme un système complet d’événements fini. D’après la formule des probabilités totales on a :
    $\begin{align*} p(E)&=p(R)\times p_R(E)+p\left(\conj{R}\right)\times p_{\conj{R}}(E) \\
    &=0,4\alpha+0,7(1-\alpha) \\
    &=0,7-0,3\alpha\end{align*}$
    $\quad$
    b.
    $\begin{align*} p(E)=0,58&\ssi 0,7-0,3\alpha=0,58 \\
    &\ssi -0,12=-0,3\alpha\\
    &\ssi  \alpha=0,4\end{align*}$
    $\quad$
  3. On a
    $\begin{align*}
    p_E\left(\conj{R}\right)&=\dfrac{p\left(E\cap \conj{R}\right)}{p(E)} \\
    &=\dfrac{p\left(\conj{R}\right)\times P_{\conj{R}}(E)}{p(E)} \\
    &=\dfrac{0,7(1-\alpha)}{0,58} \\
    &=\dfrac{0,7\times 0,6}{0,58} \\
    &=\dfrac{21}{29}\\
    &\approx 0,72
    \end{align*}$
    La probabilité que le client ayant loué un vélo électrique ait loué un vélo tout terrain est environ égale à $0,72$.
    $\quad$
  4. On a
    $\begin{align*} p\left(\conj{R}\cap E\right)&=p\left(\conj{R}\right)\times p_{\conj{R}}(E) \\
    &=0,7(1-\alpha)\\
    &=0,7\times 0,6\\
    &=0,42\end{align*}$
    La probabilité que le client loue un vélo tout terrain électrique est égale à $0,42$.
    $\quad$
  5. a. $X(\Omega)=\acco{25,~35,~40,~50}$
    $\begin{align*} p(X=25)&=p\left(R\cap \conj{E}\right) \\
    &= 0,4\times 0,4\\
    &=0,24\end{align*}$
    $\begin{align*} p(X=35)&=p\left(\conj{R}\cap \conj{E}\right) \\
    &= 0,6\times 0,3\\
    &=0,18\end{align*}$
    $\begin{align*} p(X=40)&=p\left(R\cap \conj{E}\right) \\
    &= 0,4\times 0,4\\
    &=0,16\end{align*}$
    $\begin{align*} p(X=50)&=p\left(\conj{R}\cap E\right) \\
    &= 0,6\times 0,7\\
    &=0,42\end{align*}$
    On obtient ainsi le tableau de loi de probabilité de $X$ suivant :
    $\begin{array}{|c|c|c|c|c|}
    \hline
    x&25&35&40&50\\
    \hline
    p(X=x)&0,24&0,18&0,16&0,42\\
    \hline
    \end{array}$
    $\quad$
    b. L’espérance mathématique de $X$ est :
    $\begin{align*} E(X)&=25\times 0,24+35\times 0,18+40\times 0,16+50\times 0,42 \\
    &=39,7\end{align*}$
    En moyenne, une location de vélo coûte $39,70$ euros.
    $\quad$
  6. a. On répète $30$ fois de façon indépendante la même expérience de Bernoulli de paramètre $0,58$.
    $Y$ suit donc la loi binomiale de paramètres $n=30$ et $p=0,58$.
    $\quad$
    b.
    $\begin{align*} p(X=20)&=\dbinom{30}{20} 0,58^{20}\times 0,42^{10} \\
    &\approx 0,095\end{align*}$
    La probabilité qu’un échantillon contienne exactement $20$ clients qui
    louent un vélo électrique est environ égale à $0,095$.
    $\quad$
    c. On veut calculer $P(X\pg 15) \approx 0,858$.
    La probabilité qu’un échantillon contienne au moins $15$ clients qui louent un vélo électrique est environ égale à $0,858$.
    $\quad$

Ex 2

Exercice 2

  1. Soit $n\in \N$
    $\begin{align*} b_{n+1}&=a_{n+1}-2 \\
    &=0,5a_n+1-2 \\
    &=0,5a_n-1 \\
    &=0,5\left(a_n-2\right) \\
    &=0,5b_n\end{align*}$
    La suite $\left(b_n\right)$ est donc géométrique de raison $0,5$.
    Réponse b
    $\quad$
  2. On a donc $u_1=5$, $v_1=3$, $u_2=14$ et $v_2=8$.
    Donc $\dfrac{u_2}{v_2}=1,75$
    Réponse c
    $\quad$
  3. La boucle du programme calcule tous les termes $u_n$ et $v_n$ pour $n$ allant de $1$ à $10$.
    Le programme renvoie donc $u_{10}$ et $v_{10}$.
    Réponse d
    $\quad$
  4. La fonction $f’$ semble croissante sur l’intervalle $[-4;0]$.
    Par conséquent la fonction $f$ semble convexe sur cet intervalle.
    Réponse b
    $\quad$
  5. Le coefficient directeur de la droite $(BC)$ est
    $\begin{align*} f\dsec(1)&=\dfrac{y_C-y_B}{x_C-x_B} \\
    &=5\end{align*}$
    Réponse d
    $\quad$
  6. On considère la fonction $F$ définie sur $\R$ par $F(x)=\left(x^2-2x+3\right)\e^x-2$.
    La fonction $F$ est dérivable sur $\R$ en tant que somme et produit de fonctions dérivables sur $\R$.
    Pour tout réel $x$ on a
    $\begin{align*} F'(x)&=(2x-2)\e^x+\left(x^2-2x+3\right)\e^x \\
    &=\left(2x-2+x^2-2x+3\right)\e^x \\
    &=\left(x^2+1\right)\e^x\end{align*}$
    Ainsi $F$ est une primitive de la fonction $f$ sur $\R$.
    De plus $F(0)=3-2=1$.
    Réponse b
    $\quad$

Ex 3

Exercice 3

Partie A

  1. Par croissances comparées, $\lim\limits_{x\to 0} x\ln(x)=0$ donc $\lim\limits_{x\to 0} f(x)=0$.
    $\quad$
  2. Pour tout réel $x>0$ on a $f(x)=x\left(1-\ln(x)\right)$.
    Or $\lim\limits_{x\to +\infty}\ln(x)=+\infty$ donc $\lim\limits_{x\to +\infty} 1-\ln(x)=-\infty$ ainsi $\lim\limits_{x\to +\infty} f(x)=-\infty$.
    $\quad$
  3. a. Pour tout réel $x>0$ on a
    $\begin{align*} f'(x)&=1-\ln(x)-x\times \dfrac{1}{x} \\
    &=1-\ln(x)+1\\
    &=-\ln(x)\end{align*}$
    $\quad$
    b. $f'(x)=0\ssi -\ln(x)=0 \ssi x=1$
    $f'(x)>0 \ssi -\ln(x)>0 \ssi x\in ]0;1[$.
    La fonction $f$ est donc strictement croissante sur $]0;1]$ et strictement décroissante sur $[1;+\infty[$.
    On obtient ainsi le tableau de variations suivant :
    $\quad$

    $\quad$
  4. $f(x)=x\ssi x-x\ln(x)=x \ssi -x\ln(x)=0 \ssi x=1$ (la valeur $0$ n’est pas solution puisque $f$ n’est pas définie en $0$).
    $\quad$

Partie B

  1. Pour tout $n\in \N$ on pose $P(n):~0,5\pp u_n\pp u_{n+1} \pp 1$.
    Initialisation : $u_0=0,5$ et $u_1\approx 0,85$.
    Par conséquent $P(0)$ est vraie.
    $\quad$
    Hérédité : Soit $n\in \N$. On suppose que $P(n)$ est vraie.
    $0,5\pp u_n \pp u_{n+1} \pp 1$.
    La fonction $f$ est croissante sur l’intervalle $[0,5;1]$.
    Par conséquent $f(0,5) \pp f\left(u_n\right) \pp f\left(u_{n+1}\right) \pp p(1)$ c’est-à-dire $u_1\pp u_{n+1} \pp u_{n+2} \pp 1$.
    Or $u_1\approx 0,85$.
    La propriété $P(n+1)$ est vraie.
    $\quad$
    D’après le principe de récurrence, pour tout $n\in \N$, $0,5\pp u_n\pp u_{n+1} \pp 1$.
    $\quad$
  2. a. La suite $\left(u_n\right)$ est croissante et majorée par $1$. Elle converge donc vers un réel $\ell$.
    $\quad$
    b. La fonction $f$ est continue sur $]0;+\infty[$ et, pour tout $n\in \N$, on a $u_{n+1}=f\left(u_n\right)$.
    Par conséquent $\ell$ est solution de l’équation $f(x)=x$.
    D’après la question A.4. l’unique solution de cette équation est $1$.
    Ainsi $\ell=1$.
    $\quad$

Partie C

  1. La fonction $f_k$ est dérivable sur $]0;+\infty[$ en tant que somme et produit de fonctions dérivables sur cet intervalle.
    Pour tout réel $x>0$ on a :
    $\begin{align*} f_k'(x)&=k-\ln(x)-x\times \dfrac{1}{x} \\
    &=-\ln(x)+k-1\end{align*}$
    $f_k'(x)>0 \ssi -\ln(x)+k-1>0 \ssi \ln(x)<k-1 \ssi x<\e^{k-1}$
    La fonction $f_k$ est donc strictement croissante sur $\left]0;\e^{k-1}\right]$ et strictement décroissante sur $\left[\e^{k-1};+\infty\right[$.
    La fonction $f_k$ admet par conséquent un maximum en $x_k=\e^{k-1}$.
    $\quad$
  2. Soit $k\in \R$.
    $\begin{align*} y_k=f_k\left(x_k\right)\\
    &=k\e^{k-1}-\e^{k-1}\ln\left(\e^{k-1}\right) \\
    &=k\e^{k-1}-(k-1)\e^{k-1} \\
    &=\e^{k-1}\left(k-(k-1)\right) \\
    &=\e^{k-1}\\
    &=x_k\end{align*}$
    $\quad$

Ex 4

Exercice 4

  1. a. Les coordonnées du vecteur $\vec{u}’$ sont $\begin{pmatrix} 0\\1\\1\end{pmatrix}$.
    $\quad$
    b. Les vecteurs $\vec{u}$ et $\vec{u}’$ ne sont pas colinéaires (ils n’ont pas les mêmes coordonnées nulles). Les droites $\mathscr{D}$ et $\mathscr{D}’$ ne sont donc pas parallèles.
    $\quad$
    c. Une représentation paramétrique de la droite $\mathscr{D}$ est $\begin{cases} x=2+k\\y=4+2k\\z=0\end{cases}$.
    $\quad$
  2. $\vec{v}.\vec{u}=2-2+0=0$ et $\vec{v}.\vec{u}’=0-1+1=0$.
    $\vec{v}$ est donc orthogonal aux deux vecteurs, non colinéaires, $\vec{u}$ et $\vec{u}’$.
    $\vec{v}$ est donc un vecteur directeur de la droite perpendiculaire à la fois à $\mathscr{D}$ et $\mathscr{D}’$.
    Ainsi $\vec{v}\begin{pmatrix}2\\-1\\1\end{pmatrix}$ est un vecteur directeur de la droite $\Delta$.
    $\quad$
  3. a. $\vec{n}.\vec{u}=2-2+0=0$ et $\vec{n}.\vec{v}= 4+1-5=0$.
    Ainsi $\vec{n}$ orthogonal à deux vecteurs non colinéaires du plan $\mathscr{P}$.
    $\quad$
    b. Une équation cartésienne du plan $\mathscr{P}$ est donc de la forme $2x-y-5z+d=0$.
    Le point $A(2;4;0)$ appartient au plan $\mathscr{P}$.
    Par conséquent $4-4-0+d=0 \ssi d=0$.
    Une équation cartésienne du plan $\mathscr{P}$ est donc $2x-y-5z=0$.
    $\quad$
    c. $M’$ est un point de $Delta$. Il appartient donc également au plan $\mathscr{P}$ qui contient cette droite.
    $M’$ est un point de $\mathscr{D}’$.
    $M’$ est donc le point d’intersection de la droite $\mathscr{D}’$ avec le plan $\mathscr{P}$.
    $2\times 3-1-5=0$ : le point de coordonnées $(3;1;1)$ appartient donc au plan $\mathscr{P}$.
    En prenant $t=-2$ dans la représentation paramétrique de la droite $\mathscr{D}’$ on obtient le point de coordonnées $(3;1;1)$.
    Ainsi ce point est le point d’intersection de la droite $\mathscr{D}’$ et $\mathscr{P}$.
    Ainsi $M’$ a pour coordonnées $(3;1;1)$.
    $\quad$
  4. a. $\vec{v}$ est un vecteur directeur de $\Delta$ et $M’$ appartient à cette droite.
    Une représentation paramétrique de la droite $\Delta$ est donc $\begin{cases} x= 3+2k’\\y=1-k’\\z=1+k’\end{cases} \qquad k’\in \R$.
    $\quad$
    b. En prenant $k’=-1$ dans la représentation paramétrique de $\Delta$ on obtient le point de coordonnées $(-1;2;0)$.
    En prenant $k=-1$ dans la représentation paramétrique de $\mathscr{D}$ on obtient le point de coordonnées $(-1;2;0)$.
    $M$ est le point d’intersection de ces deux droites. Donc $M$ a pour coordonnées $(1;2;0)$.
    $\quad$
    c. Les coordonnées de $\vect{MM’}$ sont $\begin{pmatrix}2\\-1\\1\end{pmatrix}$.
    Par conséquent
    $\begin{align*} MM’&=\sqrt{2^2+(-1)^2+1^2}\\
    &=\sqrt{4+1+1} \\
    &=\sqrt{6}\end{align*}$.
    $\quad$
  5. a. Un vecteur directeur de la droite $d$ est $\vec{r}\begin{pmatrix} 5\\5\\1\end{pmatrix}$.
    $\vec{n}.\vec{r}=10-5-5=0$. Par conséquent $\vec{n}$ est normal à la droite $d$.
    Ainsi $d$ est parallèle au plan $\mathscr{P}$.
    $\quad$
    b. Les droites $\mathscr{D}$ et $\Delta$ sont perpendiculaires en $M$.
    Le point $A$ appartient à la droite $\mathscr{D}$ et le point $M’$ appartient à la droite $\Delta$.
    Le triangle $AMM’$ est rectangle en $M$.
    Les coordonnées de $\vect{AM}$ sont $\begin{pmatrix} -1\\-2\\0\end{pmatrix}$.
    Par conséquent
    $\begin{align*} AM&=\sqrt{(-1)^2+(-2)^2+0^2} \\
    &=\sqrt{5}\end{align*}$
    Ainsi l’aire du triangle $AMM’$ est
    $\begin{align*} \mathscr{A}&=\dfrac{AM\times MM’}{2} \\
    &=\dfrac{\sqrt{30}}{2}\end{align*}$.
    Le volume du tétraèdre $ANMM’$ est donc $V=\dfrac{\sqrt{30}}{3}\ell$.
    $\quad$
    c. La droite $d$ est parallèle au plan $\mathscr{P}$. La distance d’un point de la droite $d$ à ce plan est donc toujours la même. Ainsi $\ell$ ne dépend pas du point $N$ choisi.
    Par conséquent $AN_1MM’$ et $AN_2MM’$ ont le même volume.
    $\quad$

Énoncé

Le sujet propose 4 exercices
Le candidat choisit 3 exercices parmi les 4 exercices et ne doit traiter que ces 3 exercices.

Exercice 1     7 points
Thème : probabilités

Dans le magasin d’Hugo, les clients peuvent louer deux types de vélos : vélos de route ou bien vélos tout terrain. Chaque type de vélo peut être loué dans sa version électrique ou non.
On choisit un client du magasin au hasard, et on admet que :

  • Si le client loue un vélo de route, la probabilité que ce soit un vélo électrique est de $0,4$ ;
  • Si le client loue un vélo tout terrain, la probabilité que ce soit un vélo électrique est de $0,7$ ;
  • La probabilité que le client loue un vélo électrique est de $0,58$.

On appelle $\alpha$ la probabilité que le client loue un vélo de route, avec $0\pp \alpha\pp 1$.

On considère les événements suivants :

  • $R$ : « le client loue un vélo de route » ;
  • $E$ : « le client loue un vélo électrique » ;
  • $\conj{R}$ et $\conj{E}$ , événements contraires de $R$ et $E$.

On modélise cette situation aléatoire à l’aide de l’arbre reproduit ci-dessous :

Si $F$ désigne un événement quelconque, on notera $p(F)$ la probabilité de $F$.

  1. Recopier cet arbre sur la copie et le compléter.
    $\quad$
  2. a. Montrer que $p(E)=0,7-0,3\alpha$.
    $\quad$
    b. En déduire que : $\alpha = 0,4$.
    $\quad$
  3. On sait que le client a loué un vélo électrique. Déterminer la probabilité qu’il ait loué un vélo tout terrain. On donnera le résultat arrondi au centième.
    $\quad$
  4. Quelle est la probabilité que le client loue un vélo tout terrain électrique ?
    $\quad$
  5. Le prix de la location à la journée d’un vélo de route non électrique est de $25$ euros, celui d’un vélo tout terrain non électrique de $35$ euros. Pour chaque type de vélo, le choix de la version électrique augmente le prix de location à la journée de $15$ euros.
    On appelle $X$ la variable aléatoire modélisant le prix de location d’un vélo à la journée.
    a. Donner la loi de probabilité de $X$. On présentera les résultats sous forme d’un tableau.
    $\quad$
    b. Calculer l’espérance mathématique de $X$ et interpréter ce résultat.
    $\quad$
  6. Lorsqu’on choisit $30$ clients d’Hugo au hasard, on assimile ce choix à un tirage avec remise. On note $Y$ la variable aléatoire associant à un échantillon de $30$ clients choisis au hasard le nombre de clients qui louent un vélo électrique.
    On rappelle que la probabilité de l’événement $E$ est : $p(E) = 0,58$.
    a. Justifier que $Y$ suit une loi binomiale dont on précisera les paramètres.
    $\quad$
    b. Déterminer la probabilité qu’un échantillon contienne exactement $20$ clients qui louent un vélo électrique. On donnera le résultat arrondi au millième.
    $\quad$
    c. Déterminer la probabilité qu’un échantillon contienne au moins $15$ clients qui louent un vélo électrique. On donnera le résultat arrondi au millième.
    $\quad$

$\quad$

Exercice 2     7 points
Thèmes : suites, fonctions

Cet exercice est un questionnaire à choix multiples. Pour chacune des questions suivantes, une seule des quatre réponses proposées est exacte.
Une réponse fausse, une réponse multiple ou l’absence de réponse à une question ne rapporte ni n’enlève de point.
Pour répondre, indiquer sur la copie le numéro de la question et la lettre de la réponse choisie. Aucune justification n’est demandée.

  1. On considère les suites $\left(a_n\right)$ et $\left(b_n\right)$ définie par $a_0=1$ et, pour tout entier naturel $n$, $a_{n+1} = 0,5a_n+1$ et $b_n=a_n-2$.
    On peut affirmer que :
    a. $\left(a_n\right)$ est arithmétique ;
    b. $\left(b_n\right)$ est géométrique ;
    c. $\left(a_n\right)$ est géométrique ;
    d. $\left(b_n\right)$ est arithmétique.
    $\quad$

Dans les questions 2. et 3., on considère les suites $\left(u_n\right)$ et $\left(b_n\right)$ définies par :$$u_0=2,~v_0=1 \text{ et, pour tout entier naturel }n :\begin{cases} u_{n+1}=u_n+3v_n\\v_{n+1}=u_n+v_n\end{cases}$$

  1. On peut affirmer que :
    a. $\begin{cases} u_2=5\\v_2=3\end{cases}$;
    b. $u_2^2-3v_2^2=-2^2$;
    c. $\dfrac{u_2}{v_2}=1,75$;
    d. $5u_1=3v_1$.
    $\quad$
  2. On considère le programme ci-dessous écrit en langage Python :
    $$\begin{array}{|l|}
    \hline
    \text{def valeurs() :}\\
    \quad \text{u = 2}\\
    \quad \text{v = 1}\\
    \quad \text{for k in range(1,11) :}\\
    \qquad \text{c = u}\\
    \qquad \text{u = u+3*v}\\
    \qquad \text{v = c+v}\\
    \quad \text{return (u,v)}\\
    \hline
    \end{array}$$
    Ce programme renvoie :
    a. $u_{11}$ et $v_{11}$;
    b. $u_{10}$ et $v_{11}$;
    c. les valeurs de $u_n$ et $v_n$ pour $n$ allant de $1$ à $10$;
    d. $u_{10}$ et $v_{10}$.
    $\quad$

Pour les questions 4. et 5., on considère une fonction $f$ deux fois dérivable sur l’intervalle $[-4 ; 2]$. On note $f’$ la fonction dérivée de $f$ et $f\dsec$ la dérivée seconde de $f$.
On donne ci-dessous la courbe représentative $\mathcal{C}’$ de la fonction dérivée $f’$ dans un repère du plan. On donne de plus les points $A(-2; 0)$, $B(1; 0)$ et $C(0; 5)$.

  1. La fonction $f$ est :
    a. concave sur $[-2; 1]$;
    b. convexe sur $[-4; 0]$;
    c. convexe sur $[-2; 1]$;
    d. convexe sur $[0; 2]$.
    $\quad$
  2. On admet que la droite $(BC)$ est la tangente à la courbe $\mathcal{C}’$ au point $B$.
    On a :
    a. $f'(1) < 0$;
    b. $f'(1)= 5$;
    c. $f\dsec(1) > 0$;
    d. $f\dsec(1) = -5$.
    $\quad$
  3. Soit $f$ la fonction définie sur $\R$ par $f(x)=\left(x^2+1\right)\e^x$.
    La primitive $F$ de $f$ sur $\R$ telle que $F(0) = 1$ est définie par :
    a. $F(x)=\left(x^2-2x+3\right)\e^x$;
    b. $F(x)=\left(x^2-2x+3\right)\e^x-2$;
    c. $F(x)=\left(\dfrac{1}{3}x^3+x\right)\e^x+1$;
    d. $F(x)=\left(\dfrac{1}{3}x^3+x\right)\e^x$;
    $\quad$

$\quad$

Exercice 3     7 points
Thèmes : fonction logarithme, suites

Les parties B et C sont indépendantes.

On considère la fonction $f$ définie sur $]0; +\infty[$ par $f(x) = x-x\ln(x)$, où $\ln$ désigne la fonction logarithme népérien.

Partie A

  1. Déterminer la limite de $f(x)$ quand $x$ tend vers $0$.
    $\quad$
  2. Déterminer la limite de $f(x)$ quand $x$ tend vers $+\infty$.
    $\quad$
  3. On admet que la fonction $f$ est dérivable sur $]0; +\infty[$ et on note $f’$ sa fonction dérivée.
    a. Démontrer que, pour tout réel $x>0$, on a : $f'(x)=-\ln(x)$.
    $\quad$
    b. En déduire les variations de la fonction $f$ sur $]0; +\infty[$ et dresser son tableau de variation.
    $\quad$
  4. Résoudre l’équation $f(x) = x$ sur $]0; +\infty[$.
    $\quad$

Partie B

Dans cette partie, on pourra utiliser avec profit certains résultats de la partie A.

On considère la suite $\left(u_n\right)$ définie par : $$\begin{cases} u_0=0,5\\\text{pour tout entier naturel }n, u_{n+1}=u_n-u_n\ln\left(u_n\right)\end{cases}$$
Ainsi, pour tout entier naturel $n$, on a : $u_{n+1}=f\left(u_n\right)$.

  1. On rappelle que la fonction $f$ est croissante sur l’intervalle $[0,5; 1]$.
    Démontrer par récurrence que, pour tout entier naturel $n$, on a : $0,5\pp u_n\pp u_{n+1}\pp 1$.
    $\quad$
  2. a. Montrer que la suite $\left(u_n\right)$ est convergente.
    $\quad$
    b. On note $l$ la limite de la suite $\left(u_n\right)$. Déterminer la valeur de $l$.
    $\quad$

Partie C

Pour un nombre réel $k$ quelconque, on considère la fonction $f_k$ définie sur $]0; +\infty[$ par : $$f_k(x)=kx-x\ln(x)$$

  1. Pour tout nombre réel $k$, montrer que $f_k$ admet un maximum $y_k$ atteint en $x_k=\e^{k-1}$.
    $\quad$
  2. Vérifier que, pour tout nombre réel $k$, on a : $x_k=y_k$.
    $\quad$

$\quad$

 

Exercice 4     7 points
Thème : géométrie dans l’espace

Dans l’espace rapporté à un repère orthonormé $\Oijk$, on considère :

  • la droite $\mathscr{D}$ passant par le point $A(2; 4; 0)$ et dont un vecteur directeur est $\vec{u}\begin{pmatrix}1\\2\\0\end{pmatrix}$;
  • la droite $\mathscr{D}’$ dont une représentation paramétrique est : $\begin{cases}x=3\\y=3+t\\z=3+t\end{cases} \quad, t\in \R$.
  1. a. Donner les coordonnées d’un vecteur directeur $\vec{u’}$ de la droite $\mathscr{D}’$.
    $\quad$
    b. Montrer que les droites $\mathscr{D}$ et $\mathscr{D}’$ ne sont pas parallèles.
    $\quad$
    c. Déterminer une représentation paramétrique de la droite $\mathscr{D}$.

On admet dans la suite de cet exercice qu’il existe une unique droite $\Delta$ perpendiculaire aux droites $\mathscr{D}$ et $\mathscr{D}’$. Cette droite $\Delta$ coupe chacune des droites $\mathscr{D}$ et $\mathscr{D}’$. On appellera $M$ le point d’intersection de $\Delta$ et $\mathscr{D}$, et $M’$ le point d’intersection de $\Delta$ et $\mathscr{D}’$.

On se propose de déterminer la distance $MM’$ appelée « distance entre les droites $\mathscr{D}$ et $\mathscr{D}’$ ».

  1. Montrer que le vecteur $\vec{v}\begin{pmatrix}2\\-1\\1\end{pmatrix}$ est un vecteur directeur de la droite $\Delta$.
    $\quad$
  2. On note $\mathscr{P}$ le plan contenant les droites $\mathscr{D}$ et $\Delta$, c’est-à-dire le plan passant par le point $A$ et de vecteurs directeurs $\vec{u}$ et $\vec{v}$.
    a. Montrer que le vecteur $\vec{n}\begin{pmatrix}2\\-1\\-5\end{pmatrix}$ est un vecteur normal au plan $\mathscr{P}$.
    $\quad$
    b. En déduire qu’une équation du plan $\mathscr{P}$ est : $2x-y-5z=0$.
    $\quad$
    c. On rappelle que $M’$ est le point d’intersection des droites $\Delta$ et $\mathscr{D}’$. Justifier que $M’$ est également le point d’intersection de $\mathscr{D}’$ et du plan $\mathscr{P}$.
    En déduire que les coordonnées du point $M’$ sont $(3; 1; 1)$.
    $\quad$
  3. a. Déterminer une représentation paramétrique de la droite $\Delta$.
    $\quad$
    b. Justifier que le point $M$ a pour coordonnées $(1; 2; 0)$.
    $\quad$
    c. Calculer la distance $MM’$.
    $\quad$
  4. On considère la droite $d$ de représentation paramétrique $\begin{cases} x=5t\\y=2+5t\\z=1+t\end{cases} \quad$ avec $t\in \R$.
    a. Montrer que la droite $d$ est parallèle au plan $\mathscr{P}$.
    $\quad$
    b. On note $\ell$ la distance d’un point $N$ de la droite $d$ au plan $\mathscr{P}$. Exprimer le volume du tétraèdre $ANMM’$ en fonction de $\ell$.
    On rappelle que le volume d’un tétraèdre est donné par : $V=\dfrac{1}{3}\times \mathscr{B}\times h$ où $\mathscr{B}$ désigne l’aire d’une base et $h$ la hauteur relative à cette base.
    $\quad$
    c. Justifier que, si $N_1$ et $N_2$ sont deux points quelconques de la droite $d$, les tétraèdres $AN_1MM’$ et $AN_2MM’$ ont le même volume.
    $\quad$

$\quad$

Bac – Spécialité mathématiques – Métropole, Antilles, Guyane – sujet 1 – 8 septembre 2022

Métropole Antilles/Guyane – 8 septembre 2022

Spécialité maths – Sujet 1 – Correction

L’énoncé de ce sujet de bac est disponible ici.

Ex 1

Exercice 1

  1. Pour tout réel $x$
    $\begin{align*} g(x)&=\dfrac{2\e^x}{\e^x+1} \\
    &=\dfrac{2\e^x}{\e^x\left(1+\e^{-x}\right) }\\
    &=\dfrac{2}{1+\e^{-x}}\end{align*}$
    Or $\lim\limits_{x\to +\infty} \e^{-x}=0$ et $\lim\limits_{x\to +\infty} g(x)=2$.
    La droite d’équation $y=2$ est donc asymptote à la courbe représentative de la fonction $g$ en $+\infty$.
    Réponse b
    $\quad$
  2. La fonction $f\dsec$ semble positive sur $]-\infty;-1]$ et sur $[2;+\infty[$.
    Par conséquent $f$ est convexe sur $]-\infty;-1]$ et sur $[2;+\infty[$.
    Réponse c
    $\quad$
  3. Pour tout $n\in \N$ on a :
    $\begin{align*} v_{n+1}&=u_{n+1}-2 \\
    &=\dfrac{1}{2}u_{n+1}+1-2 \\
    &=\dfrac{1}{2}u_{n+1}-1 \\
    &=\dfrac{1}{2}\left(u_n-2\right)\\
    &=\dfrac{1}{2}v_n\end{align*}$
    La suite $\left(v_n\right)$ est donc géométrique de raison $\dfrac{1}{2}$.
    Réponse d
    $\quad$
  4. $0<\dfrac{1}{4}<1$ donc $\lim\limits_{n\to +\infty} \left(\dfrac{1}{4}\right)^n=0$. Par conséquent $\lim\limits_{n\to +\infty} 1+\left(\dfrac{1}{4}\right)^n=1$.
    $\begin{align*}\dfrac{n}{n+1}&=\dfrac{n}{n\left(1+\dfrac{1}{n}\right)}\\
    &=\dfrac{1}{1+\dfrac{1}{n}}\end{align*}$
    Or $\lim\limits_{n\to +\infty} \dfrac{1}{n}=0$.
    Par conséquent $\lim\limits_{n\to +\infty} \dfrac{n}{n+1}=1$ et $\lim\limits_{n\to +\infty} 2-\dfrac{n}{n+1}=1$.
    D’après le théorème des gendarmes, $\lim\limits_{n\to +\infty} u_n=1$.
    Réponse b
    $\quad$
  5. On considère la fonction $F$ définie sur $]0;+\infty[$ par $F(x)=\dfrac{1}{3}x^3\left(\ln(x)-\dfrac{1}{3}\right)$.
    La fonction $F$ est dérivable sur $]0;+\infty[$ en tant que produit de fonctions dérivables sur cet intervalle.
    Pour tout $x>0$ on a :
    $\begin{align*} F'(x)&=\dfrac{1}{3}\times 3x^2\left(\ln(x)-\dfrac{1}{3}\right)+\dfrac{1}{3}x^3\times \dfrac{1}{x}\\
    &=x^2\ln(x)-\dfrac{1}{3}x^2+\dfrac{1}{3}x^2 \\
    &=x^2\ln(x)\end{align*}$
    Ainsi $F$ est une primitive de la fonction $f$.
    Réponse a
    $\quad$
  6. Soit $x\in \R$
    $\begin{align*} 2+\dfrac{3\e^{-x}-5}{\e^{-x}+1}&=\dfrac{2\e^{-x}+2+3\e^{-x}-5}{\e^{-x}+1} \\
    &=\dfrac{5\e^{-x}-3}{\e^{-x}+1} \\
    &=\dfrac{\e^{-x}\left(5-3\e^x\right)}{\e^{-x}\left(1+\e^x\right)} \\
    &=\dfrac{5-3\e^x}{1+\e^x}\end{align*}$
    Réponse a
    $\quad$

Ex 2

Exercice 2

  1. a. On a $p\left(\conj{M}\cap \conj{G}\right)=0,06$ et $p\left(\conj{M}\right)=1-0,7$ c’est-à-dire $p\left(\conj{M}\right)=0,3$.
    Or
    $\begin{align*} P_{\conj{M}}\left(\conj{G}\right)&=\dfrac{p\left(\conj{M}\cap \conj{G}\right)}{p\left(\conj{M}\right)} \\
    &=\dfrac{0,06}{0,3} \\
    &=0,2\end{align*}$
    $\quad$
    b. On obtient l’arbre suivant :
    $\quad$
    $\quad$
    c. On veut calculer
    $\begin{align*} p\left(G\cap \conj{M}\right)&=p\left(\conj{M}\right)\times p_{\conj{M}}(G) \\
    &=0,3\times 0,8\\
    &=0,24\end{align*}$
    La probabilité de l’événement « le client visite la grotte et ne visite pas le musée » est égale à $0,24$.
    $\quad$
    d. $\left(M,\conj{M}\right)$ forme un système complet d’événements fini. D’après la formule des probabilités totales :
    $\begin{align*} p(G)&=p(G\cap M)+p\left(\conj{M}\cap G\right) \\
    &=p(M)\times p_M(G)+p\left(\conj{M}\cap G\right) \\
    &=0,7\times 0,6+0,24 \\
    &=0,66\end{align*}$
    $\quad$
  2. On veut calculer
    $\begin{align*} p_G(M)&=\dfrac{p(G\cap M)}{p(G)} \\
    &=\dfrac{0,7\times 0,6}{0,66} \\
    &=\dfrac{7}{11} \\
    &>\dfrac{1}{2}
    \end{align*}$
    L’affirmation est donc exacte.
    $\quad$
  3. a. On a $T(\Omega)=\acco{0,~5,~12,~17}$
    $\begin{align*} p(T=0)&=p\left(\conj{G}\cap \conj{M}\right) \\
    &=0,06\end{align*}$
    $\begin{align*} p(T=5)&=p\left(G\cap \conj{M}\right) \\
    &=0,24\end{align*}$
    $\begin{align*} p(T=12)&=p\left(\conj{G}\cap M\right) \\
    &=0,28\end{align*}$
    $\begin{align*} p(T=17)&=p\left(G\cap M\right) \\
    &=0,42\end{align*}$
    Ainsi
    $\begin{array}{|c|c|c|c|c|}
    \hline
    t&0&5&12&17\\
    \hline
    p(T=t)&0,06&0,24&0,28&0,42\\
    \hline
    \end{array}$
    $\quad$
    b. L’espérance mathématique de $T$ est donc
    $\begin{align*} E(T)&=0\times 0,06+5\times 0,24+12\times 0,28+17\times 0,42 \\
    &=11,7\end{align*}$
    $\quad$
    c. Un client dépense donc en moyenne $11,70$ €.
    On appelle $N$ le nombre moyen de clients par journée.
    $11,7N\pg 700 \ssi x\pg \dfrac{700}{11,7}$
    Or $\dfrac{700}{11,7}\approx 59,83$.
    Il faut donc, en moyenne, au moins $60$ clients par journée pour atteindre cet objectif.
    $\quad$
  4. On appelle $p$ le prix de la visite de la grotte. On appelle $T’$ la variable aléatoire qui modélise la somme dépensée par un client de l’hôtel pour ces visites. On obtient alors la loi de probabilité suivante
    $\begin{array}{|c|c|c|c|c|}
    \hline
    t&0&x&12&12+x\\
    \hline
    p(T’=t)&0,06&0,24&0,28&0,42\\
    \hline
    \end{array}$
    Son espérance est donc
    $\begin{align*} E(T’)&=0,24x+12\times 0,28+0,42(12+x) \\
    &=0,24x+3,36+5,04+0,42x \\
    &=8,4+0,66x\end{align*}$
    $\begin{align*} E(T’)=15&\ssi 8,4+0,66x=15 \\
    &\ssi 0,66x=6,6 \\
    &\ssi x=10\end{align*}$
    Le prix de la visite de la grotte devrait donc être de $10$ euros pour atteindre l’objectif.
    $\quad$
  5. On appelle $X$ la variable aléatoire comptant le nombre de clients ayant visité la grotte. On répète $100$ fois de façon indépendante la même expérience de Bernoulli de paramètre $0,66$.
    $X$ suit donc la loi binomiale de paramètres $n=100$ et $p=0,66$.
    D’après la calculatrice :
    $\begin{align*} P(X\pg 75)&=1-P(X\pp 74) \\
    &\approx 0,034\end{align*}$
    La probabilité qu’au moins les trois quarts des clients de l’hôtel aient visité la grotte est environ égale à $0,034$.
    $\quad$

Ex 3

Exercice 3

Partie A

  1. Par croissances comparées,$\lim\limits_{x\to +\infty} \dfrac{\ln(x)}{x}=0$.
    $\quad$
  2. a. Pour tout réel $x\pg 1$ on a :
    $\begin{align*} f'(x)&=\dfrac{\dfrac{1}{x}\times x-\ln(x)}{x^2} \\
    &=\dfrac{1-\ln(x)}{x^2}\end{align*}$
    $\quad$
    b. Pour tout réel $x\pg 1$ on a $x^2\pg 1$
    $1-\ln(x)=0\ssi \ln(x)=1\ssi x=\e$ donc $f'(x)=0 \ssi x=\e$
    $1-\ln(x)>0 \ssi \ln(x)<1 \ssi x<\e$ donc $f'(x)>0 \ssi x\in [1;\e]$
    $1-\ln(x)<0 \ssi \ln(x)>1 \ssi x>\e$ donc $f'(x)>0 \ssi x\in [\e;+\infty[$
    $\quad$
    c. On obtient donc le tableau de variations suivant :
    $\quad$

    $\quad$
  3. a. Soit $k$ un réel, $0\pp k \pp \e^{-1}$. La fonction $f$ est continue et strictement croissante sur $[1;\e]$.
    $f(1)=0\pp k$ et $f(\e)=\e^{-1}\pg k$.
    D’après le théorème de la bijection (ou corollaire du théorème des valeurs intermédiaires) l’équation $f(x)=k$ admet une unique solution $\alpha$ sur l’intervalle $[1;\e]$.
    $\quad$
    b. Soit $k$ strictement supérieur à $\dfrac{1}{\e}$.
    Pour tout réel $x\pg 1$ on a $fx)\pp \e^{-1}$.
    Par conséquent l’équation $f(x)=k$ n’admet aucune solution sur $[1;+\infty[$.
    $\quad$

Partie B

  1. La fonction $g$ est dérivable sur $\R$ comme composée de fonctions dérivables sur $\R$.
    Pour tout réel $x$ on a $g'(x)=\dfrac{1}{4}\e^{\frac{x}{4}}>0$ car la fonction exponentielle est strictement positive.
    La fonction $g$ est donc strictement croissante sur $\R$.
    $\quad$
  2. Pour tout $n\in \N$ on pose $P(n):~u_n \pp u_{n+1} \pp \e$.
    Initialisation : $u_0=1$ et $u_1=\e^{\frac{1}{4}}\approx 1,28$
    Par conséquent $u_0\pp u_1 \pp \e$ et $P(0)$ est vraie.
    $\quad$
    Hérédité : Soit $n\in \N$. On suppose $P(n)$ vraie.
    $u_n \pp u_{n+1} \pp \e$. La fonction $g$ est strictement croissante sur $[1;\e]$. Par conséquent :
    $g\left(u_{n+1}\right) \pp g\left(u_{n+1}\right) \pp g(\e)$ soit $u_{n+1} \pp u_{n+2} \pp \e^{-1}\pp \e$.
    Donc $P(n+1)$ est vraie.
    $\quad$
    D’après le principe de récurrence, pour tout entier naturel $n$, on a $u_n\pp u_{n+1} \pp \e$.
    $\quad$
  3. La suite $\left(u_n\right)$ est croissante et majorée par $\e$.
    Par conséquent $\left(u_n\right)$ converge vers un réel $\ell$.
    $\quad$
  4. $\e^{\frac{x}{4}}=x \ssi \dfrac{x}{4}=\ln(x) \ssi \dfrac{1}{4}=\dfrac{\ln(x)}{x} \ssi f(x)=\dfrac{1}{4}$
    $\quad$
  5. D’après la calculatrice une solution de l’équation $f(x)=\dfrac{1}{4}$ est environ égale à $1,43$ qui appartient bien à $[1;\e]$.
    Ainsi $\ell \approx 1,43$.

Ex 4

Exercice 4

  1. a. $\vect{DE}\begin{pmatrix} 12\\-15\\-6\end{pmatrix}$
    Par conséquent $\dfrac{1}{3}\vect{DE}\begin{pmatrix}4\\-5\\-2\end{pmatrix}$ est un vecteur directeur de $\Delta$.
    Ainsi, une représentation paramétrique de $\Delta$ est $\begin{cases} x=-1+4t\\y=6-5t\\z=8-2t\end{cases} \qquad t\in \R$.
    $\quad$
    b. $\Delta$ et $\Delta’$ sont parallèles. Un vecteur directeur de de $\Delta$ est donc également un vecteur directeur de $\Delta’$.
    Une représentation paramétrique de $\Delta’$ est donc $\begin{cases} x=4t\\y=-5t\\z=-2t\end{cases} \qquad t\in \R$.
    $\quad$
    c. $4t=1,36 \ssi t=0,34$
    De plus $-5\times 0,34=-1,7$ et $-2\times 0,34=-0,68 \neq -0,7$.
    Donc $F$ n’appartient pas à la droite $\Delta’$.
    $\quad$
  2. a. $\vect{AB}\begin{pmatrix}2\\2\\-1\end{pmatrix}$ et $\vect{AC}\begin{pmatrix}2\\0\\4\end{pmatrix}$.
    Ces deux vecteurs ne sont clairement pas colinéaires (aucune coordonnée nulle pour le vecteur $\vect{AB}$). Les points $A$, $B$ et $C$ définissent donc bien un plan.
    $\quad$
    b. On note $\vec{n}\begin{pmatrix}4\\-5\\-2\end{pmatrix}$.
    $\vec{n}.\vect{AB}=8-10+2=0$ et $\vec{n}.\vect{AC}=8+0-8=0$.
    Le vecteur $\vec{n}$ est donc orthogonal à deux vecteurs non colinéaires du plan $(ABC)$.
    La droite $\Delta$ est par conséquent perpendiculaire au plan $(ABC)$.
    $\quad$
    c. Une équation du plan $(ABC)$ est donc de la forme $4x-5y-2z+d=0$.
    Le point $A(-1;-1;3)$ appartient au plan $(ABC)$.
    Par conséquent $-4+5-6+d=0 \ssi d=5$.
    Une équation cartésienne du plan $(ABC)$ est donc $4x-5y-2z+5=0$.
    $\quad$
  3. a. Prenons $t=2$ dans la représentation paramétrique de $\Delta$.
    Le point de coordonnées $(7;-4;5)$ appartient donc à la droite $\Delta$.
    Donc $G(7;-4;4)$ appartient à la droite $\Delta$.
    $\quad$
    b. Les coordonnées du point $H$ sont solution du système
    $\begin{align*} \begin{cases} 4x-5y-2z+5=0\\x=-1+4t\\y=6-5t\\z=8-2t\end{cases}&\ssi \begin{cases} x=-1+4t\\y=6-5t\\z=8-2t\\-4+16t-30+25t-16+4t+5=0\end{cases} \\
    &\ssi \begin{cases} x=-1+4t\\y=6-5t\\z=8-2t\\45t=45\end{cases} \\
    &\ssi \begin{cases} t=1\\x=3\\y=1\\z=6\end{cases} \end{align*}$.
    Le point $H$ a donc pour coordonnées $(3;1;6)$.
    $\quad$
    c. La distance du point $G$ au plan $(ABC)$ est par conséquent $HG$.
    Or $\vect{HG}$ a pour coordonnées $\begin{pmatrix} -4\\5\\2\end{pmatrix}$
    Ainsi
    $\begin{align*} HG&=\sqrt{(-4)^2+5^2+2^2} \\
    &=\sqrt{16+25+4} \\
    &=\sqrt{45} \\
    &=\sqrt{9\times 5}\\
    &=3\sqrt{5}\end{align*}$
    $\quad$
  4. a. $\vect{AB}.\vect{AC}=4+0-4=0$.
    Le triangle $ABC$ est donc rectangle en $A$.
    $\quad$
    b. $AB=\sqrt{9}=3$ et $AC=\sqrt{20}=2\sqrt{5}$
    Le volume du tétraèdre $ABCG$ est donc
    $\begin{align*} V&=\dfrac{\dfrac{AB\times AC}{2}\times HG}{3} \\
    &=\dfrac{3\times \sqrt{5}\times 3\sqrt{5}}{3} \\
    &=15\end{align*}$
    $\quad$

Énoncé

Le sujet propose 4 exercices
Le candidat choisit 3 exercices parmi les 4 exercices et ne doit traiter que ces 3 exercices.

Exercice 1     7 points
Thèmes : fonctions, suites

Cet exercice est un questionnaire à choix multiples. Pour chacune des questions suivantes, une seule des quatre réponses proposées est exacte. Une réponse fausse, une réponse multiple ou l’absence de réponse à une question ne rapporte ni n’enlève de point.
Pour répondre, indiquer sur la copie le numéro de la question et la lettre de la réponse choisie. Aucune justification n’est demandée.

  1. On considère la fonction $g$ définie sur $\R$ par : $g(x)=\dfrac{2\e^x}{\e^x+1}$.
    La courbe représentative de la fonction $g$ admet pour asymptote en $+\infty$ la droite d’équation :
    a. $x=2$;
    b. $y=2$;
    c. $y=0$
    d. $x=-1$.
    $\quad$
  2. On considère une fonction $f$ définie et deux fois dérivable sur $\R$.
    On appelle $C$ sa représentation graphique.
    $\quad$
    On désigne par $f\dsec$ la dérivée seconde de $f$.
    $\quad$
    On a représenté sur le graphique ci-dessous la courbe de $f\dsec$, notée $C\dsec$.
    $\quad$

    $\quad$
    a. $C$ admet un unique point d’inflexion;
    b. $f$ est convexe sur l’intervalle $[-1;2]$;
    c. $f$ est convexe sur $]-\infty;-1]$ et sur $[2;+\infty[$;
    d. $f$ est convexe sur $\R$.
    $\quad$
  3. On donne la suite $\left(u_n\right)$ définie par : $u_0= 0$ et pour tout entier naturel $n$, $u_{n+1}=\dfrac{1}{2}u_n+1$.
    La suite $\left(v_n\right)$, définie pour tout entier naturel $n$ par $v_n=u_n-2$, est :
    a. arithmétique de raison $-2$;
    b. géométrique de raison $-2$;
    c. arithmétique de raison $1$;
    d. géométrique de raison $\dfrac{1}{2}$.
    $\quad$
  4. On considère une suite $\left(u_n\right)$ telle que, pour tout entier naturel $n$, on a : $$1+\left(\dfrac{1}{4}\right)^n \pp u_n \pp 2-\dfrac{n}{n+1}$$
    On peut affirmer que la suite $\left(u_n\right)$ :
    a. converge vers $2$;
    b. converge vers $1$;
    c. diverge vers $+\infty$;
    d. n’a pas de limite.
    $\quad$
  5. Soit $f$ la fonction définie sur $]0; +\infty[$ par $f(x)=x^2\ln(x)$.
    Une primitive $F$ de $f$ sur $]0; +\infty[$ est définie par :
    a. $F(x) =\dfrac{1}{3}x^3\left(\ln(x)-\dfrac{1}{3}\right)$;
    b. $F(x) = \dfrac{1}{3}x^3\left(\ln(x)-1\right)$;
    c. $F(x) = \dfrac{1}{3}x^2$;
    d. $F(x) = \dfrac{1}{3}x^2\left(\ln(x)-1\right)$.
    $\quad$
  6. Pour tout réel $x$ , l’expression $2+\dfrac{3\e^{-x}-5}{\e^{-x}+1}$ est égale à :
    a. $\dfrac{5-3\e^x}{1+\e^x}$;
    b. $\dfrac{5+3\e^x}{1-\e^x}$;
    c. $\dfrac{5+3\e^x}{1+\e^x}$;
    d. $\dfrac{5-3\e^x}{1-\e^x}$.
    $\quad$

$\quad$

Exercice 2     7 points
Thème : probabilités

Un hôtel situé à proximité d’un site touristique dédié à la préhistoire propose deux visites dans les environs, celle d’un musée et celle d’une grotte.

Une étude a montré que $70\%$ des clients de l’hôtel visitent le musée. De plus, parmi les clients visitant le musée, $60\%$ visitent la grotte.
Cette étude montre aussi que $6\%$ des clients de l’hôtel ne font aucune visite.
On interroge au hasard un client de l’hôtel et on note :

  • $M$ l’événement : « le client visite le musée » ;
  • $G$ l’événement : « le client visite la grotte ».

On note $\conj{M}$ l’événement contraire de $M$, $\conj{G}$ l’événement contraire de $G$, et pour tout événement $E$, on note $p(E)$ la probabilité de $E$.

Ainsi, d’après l’énoncé, on a : $p\left(\conj{M}\cap \conj{G}\right)= 0,06$

  1. a. Vérifier que $p_{\conj{M}}\left(\conj{G}\right) = 0,2$, où $p_{\conj{M}}\left(\conj{G}\right)$ désigne la probabilité que le client interrogé ne visite pas la grotte sachant qu’il ne visite pas le musée.
    $\quad$
    b. L’arbre pondéré ci-dessous modélise la situation. Recopier et
    compléter cet arbre en indiquant sur chaque branche la probabilité
    associée.
    $\quad$
    $\quad$
    c. Quelle est la probabilité de l’événement « le client visite la grotte et ne visite pas le musée » ?
    $\quad$
    d. Montrer que $p(G) = 0,66$.
    $\quad$
  2. Le responsable de l’hôtel affirme que parmi les clients qui visitent la grotte, plus de la moitié visitent également le musée. Cette affirmation est-elle exacte ?
    $\quad$
  3. Les tarifs pour les visites sont les suivants :
    $\bullet$ visite du musée : $12$ euros ;
    $\bullet$ visite de la grotte : $5$ euros.
    On considère la variable aléatoire $T$ qui modélise la somme dépensée par un client de l’hôtel pour ces visites.
    a. Donner la loi de probabilité de $T$. On présentera les résultats sous la forme d’un tableau.
    $\quad$
    b. Calculer l’espérance mathématique de $T$.
    $\quad$
    c. Pour des questions de rentabilité, le responsable de l’hôtel estime que le montant moyen des recettes des visites doit être supérieur à $700$ euros par jour. Déterminer le nombre moyen de clients par journée permettant d’atteindre cet objectif.
    $\quad$
  4. Pour augmenter les recettes, le responsable souhaite que l’espérance de la variable aléatoire modélisant la somme dépensée par un client de l’hôtel pour ces visites passe à $15$ euros, sans modifier le prix de visite du musée qui demeure à $12$ euros. Quel prix faut-il fixer pour la visite de la grotte afin d’atteindre cet objectif ? (On admettra que l’augmentation du
    prix d’entrée de la grotte ne modifie pas la fréquentation des deux sites).
    $\quad$
  5.  On choisit au hasard $100$ clients de l’hôtel, en assimilant ce choix à un tirage avec remise. Quelle est la probabilité qu’au moins les trois quarts de ces clients aient visité la grotte à l’occasion de leur séjour à l’hôtel ? On donnera une valeur du résultat à $10^{-3}$ près.
    $\quad$

$\quad$

Exercice 3     7 points
Thèmes : fonctions logarithme et exponentielle, suites

Les parties A et B sont, dans une large mesure, indépendantes.

Partie A

On considère la fonction $f$ définie sur l’intervalle $[1 ; +\infty[$ par $f(x)=\dfrac{\ln(x)}{x}$, où $\ln$ désigne la fonction logarithme népérien.

  1. Donner la limite de la fonction $f$ en $+\infty$.
    $\quad$
  2. On admet que la fonction $f$ est dérivable sur l’intervalle $[1 ; +\infty[$ et on note $f’$ sa fonction dérivée.
    a. Montrer que, pour tout nombre réel $x\pg 1$, $f'(x)=\dfrac{1-\ln(x)}{x^2}$.
    $\quad$
    b. Justifier le tableau de signes suivant, donnant le signe de $f'(x)$ suivant les valeurs de $x$.
    $\quad$

    $\quad$
    c. Dresser le tableau de variations complet de la fonction $f$.
    $\quad$
  3. Soit $k$ un nombre réel positif ou nul.
    a. Montrer que, si $0\pp k\pp \dfrac{1}{\e}$, l’équation $f(x)=k$ admet une unique solution sur l’intervalle $[1 ;\e]$.
    $\quad$
    b. Si $k>\dfrac{1}{\e}$, l’équation $f(x) = k$ admet-elle des solutions sur l’intervalle $[1 ; +\infty[$ ?
    Justifier.
    $\quad$

Partie B

Soit $g$ la fonction définie sur $\R$ par : $g(x)=\e^{\frac{x}{4}}$.
On considère la suite $\left(u_n\right)$ définie par $u_0 = 1$ et, pour tout entier naturel $n$ : $$u_{n+1}=\e^{\frac{u_n}{4}} \text{  c’est à dire : } u_{n+1}=g\left(u_n\right)$$

  1. Justifier que la fonction $g$ est croissante sur $\R$.
    $\quad$
  2. Montrer par récurrence que, pour tout entier naturel $n$, on a : $u_n\pp u_{n+1} \pp \e$.
    $\quad$
  3. En déduire que la suite $\left(u_n\right)$ est convergente.
    $\quad$

On note $\ell$ la limite de la suite $\left(u_n\right)$, et on admet que $\ell$ est solution de l’équation : $$\e^{\frac{x}{4}}=x$$

  1. En déduire que $\ell$ est solution de l’équation $f(x)=\dfrac{1}{4}$, où $f$ est la fonction étudiée dans la partie A.
    $\quad$
  2. Donner une valeur approchée à $10^{-2}$ près de la limite $\ell$ de la suite $\left(u_n\right)$.
    $\quad$

$\quad$

Exercice 4     7 points
Thème : géométrie dans l’espace

Dans l’espace rapporté à un repère orthonormé $\Oijk$, on considère les points
$A(-1 ; -1 ; 3)$, $B(1 ; 1 ; 2)$, $C(1 ; -1 ; 7)$.
On considère également la droite ∆ passant par les points $D(-1 ; 6 ; 8)$ et $E(11 ; -9 ; 2)$.

  1. a. Vérifier que la droite $\Delta$ admet pour représentation paramétrique :
    $$\begin{cases}x = -1 + 4t\\y = 6-5t\\z = 8-2t\end{cases} \quad \text{avec }t\in \R$$
    $\quad$
    b. Préciser une représentation paramétrique de la droite $\Delta’$ parallèle à $\Delta$ et passant par l’origine $O$ du repère.
    $\quad$
    c. Le point $F(1,36 ; -1,7 ; -0,7)$ appartient-il à la droite $\Delta’$ ?
    $\quad$
  2. a. Montrer que les points $A$, $B$ et $C$ définissent un plan.
    $\quad$
    b. Montrer que la droite $\Delta$ est perpendiculaire au plan $(ABC)$.
    $\quad$
    c. Montrer qu’une équation cartésienne du plan $(ABC)$ est : $4x-5y-2z+5=0$.
    $\quad$
  3. a. Montrer que le point $G(7; -4; 4)$ appartient à la droite $\Delta$.
    $\quad$
    b. Déterminer les coordonnées du point $H$, projeté orthogonal du point $G$ sur le plan $(ABC)$.
    $\quad$
    c. En déduire que la distance du point $G$ au plan $(ABC)$ est égale à $3\sqrt{5}$.
    $\quad$
  4. a. Montrer que le triangle $ABC$ est rectangle en $A$.
    $\quad$
    b. Calculer le volume $V$ du tétraèdre $ABCG$.
    On rappelle que le volume $V$ d’un tétraèdre est donné par la formule $V=\dfrac{1}{3}\times \mathscr{B}\times h$ où $\mathscr{B}$ est l’aire d’une base et $h$ la hauteur correspondant à cette base.
    $\quad$

$\quad$

Bac – Spécialité mathématiques – Amérique du Nord – sujet 2 – 19 mai 2022

Amérique du nord – 19 mai 2022

Spécialité maths – Sujet 2 – Correction

L’énoncé de ce sujet de bac est disponible ici.

Ex 1

Exercice 1

  1. On obtient l’arbre pondéré suivant :
    $\quad$
    $\quad$
  2. a. $\left(A_1,B_1\right)$ forme un système complet d’événements fini.
    D’après la formule des probabilités totales :
    $\begin{align*} a_2&=P\left(A_2\right) \\
    &=P\left(A_1\cap A_2\right)+P\left(B_1\cap A_2\right) \\
    &=P\left(A_1\right)\times P_{A_1}\left(A_2\right)+P\left(B_1\right)\times P_{B_1}\left(A_2\right) \\
    &=0,5\times 0,84+0,5\times 0,24 \\
    &=0,54\end{align*}$
    $\quad$
    b. On veut calculer :
    $\begin{align*} P_{A_2}\left(B_1\right)&=\dfrac{P\left(A_2\cap B_1\right)}{P\left(A_2\right)} \\
    &=\dfrac{0,5\times 0,24}{0,54}\\
    &=\dfrac{2}{9}\end{align*}$
    La probabilité que le vélo se trouve au point B le premier matin sachant qu’il se trouve au point A le deuxième matin est égale à $\dfrac{2}{9}$ soit environ égale à $0,222$.
    $\quad$
  3. a. On obtient l’arbre suivant :$\quad$
    b. Soit $n\in \N^*$. $\left(A_n,B_n\right)$ forme un système complet d’événements fini.
    D’après la formule des probabilités totales on a
    $\begin{align*} a_{n+1}&=P\left(A_{n+1}\right) \\
    &=P\left(A_n\cap A_{n+1}\right)+P\left(B_n\cap A_{n+1}\right) \\
    &=P\left(A_n\right)\times P_{A_n}\left(A_{n+1}\right)+P\left(B_n\right)\times P_{B_n}\left(A_{n+1}\right) \\
    &=0,84a_n+0,24\left(1-a_n\right) \\
    &=0,6a_n+0,24\end{align*}$
    $\quad$
  4. Pour tout entier naturel $n$ non nul on pose $R(n):~a_n=0,6-0,1\times 0,6^{n-1}$.
    Initialisation : $a_1=0,5$ et $0,6-0,1^1=0,5$ donc $R(1)$ est vraie.
    $\quad$
    Hérédité : Soit $n\in \N$. On suppose $R(n)$ vraie.
    $\begin{align*} a_{n+1}&=0,6a_n+0,24 \\
    &=0,6\left(0,6-0,1\times 0,6^{n-1}\right)+0,24\\
    &=0,36-0,1\times 0,6^n+0,24 \\
    &=0,6-0,1\times 0,6^n\end{align*}$
    Donc $R(n+1)$ est vraie.
    $\quad$
    Conclusion : La propriété est vraie au rang $1$ et est héréditaire.
    Par conséquent, pour tout entier naturel $n$ on a $a_n=0,6-0,1\times 0,6^{n-1}$.
    $\quad$
  5. $-1<0,6<1$ donc $\lim\limits_{n\to +\infty} 0,6^n=0$ et $\lim\limits_{n\to +\infty} a_n=0,6$.
    Sur le long terme, la probabilité que le vélo se trouve au point $A$ est égale à $0,6$.
    $\quad$
  6. $\quad$
    $\begin{align*} a_n\pg 0,599&\ssi 0,6-0,1\times 0,6^{n-1}\pg 0,599 \\
    &\ssi -0,1\times 0,6^{n-1} \pg -0,001 \\
    &\ssi 0,6^{n-1} \pp 0,01 \\
    &\ssi (n-1)\ln(0,6)\pp \ln(0,01) \\
    &\ssi n-1\pg \dfrac{\ln(0,01)}{\ln(0,6)} \quad \text{car } \ln(0,6)<0\\
    &\ssi n\pg \dfrac{\ln(0,01)}{\ln(0,6)}+1\end{align*}$
    Or $\dfrac{\ln(0,01)}{\ln(0,6)}+1\approx 10,02$
    Le plus petit entier naturel $n$ tel que $a_n\pg 0,599$ est donc $11$.
    La probabilité que le vélo se trouve au point $A$ est supérieure à $0,599$ à partir du $11$-ième jour.
    $\quad$

 

Ex 2

Exercice 2

Partie A

  1. La fonction $p$ est dérivable sur $[-3;4]$ en tant que fonction polynôme.
    Pour tout réel $x\in [-3;4]$ on a $p'(x)=3x^2-6x+5$.
    Il s’agit d’un polynôme du second degré dont le discriminant est $\Delta=-24<0$
    Ainsi $p'(x)$ est du signe du coefficient principal $a=3>0$.
    Par conséquent $p$ est strictement croissante sur $[-3;4]$.
    $\quad$
  2. La fonction $p$ est continue (car dérivable) et strictement croissante sur $[-3;4]$.
    $p(-3)=-68<0$ et $p(4)=37>0$.
    D’après le théorème de la bijection (ou corollaire du théorème des valeurs intermédiaires), l’équation $p(x)=0$ admet une unique solution $\alpha$ sur $[-3;4]$.
    $\quad$
  3. D’après la calculatrice $\alpha\approx -0,2$.
    $\quad$
  4. La fonction $p$ est strictement croissante sur $[-3;4]$ et s’annule en $\alpha$. On obtient alors le tableau de signes suivant :
    $\quad$

Partie B

  1. a. La fonction $f$ est dérivable sur $[-3;4]$ en tant que quotient de fonctions dérivables dont le dénominateur ne s’annule pas.
    Pour tout réel $x\in [-3;4]$ on a :
    $\begin{align*} f'(x)&=\dfrac{\e^x\left(1+x^2\right)-2x\e^x}{\left(1+x^2\right)^2} \\
    &=\dfrac{\left(x^2-2x+1\right)\e^x}{\left(1+x^2\right)^2} \\
    &=\dfrac{(x-1)^2\e^x}{\left(1+x^2\right)^2} \end{align*}$
    $\quad$
    b. On a fonc $f'(1)=0$.
    La courbe $\mathscr{C}_f$ admet une tangente horizontale au point d’abscisse $1$.
    $\quad$
  2. a. Il semblerait que la fonction change de convexité (et donc $\mathscr{C}_f$ possède un point d’inflexion) environ en $0$ et en $1$.
    Le toboggan semble dont assurer de bonnes sensations.
    $\quad$
    b. La fonction exponentielle est strictement positive sur $\R$ et pour tout réel $x\in [-3;4]$ on a $\left(1+x^2\right)^3>0$.
    Le signe de $f\dsec(x)$ ne dépend donc que de celui de $p(x)(x-1)$.
    $x-1=0 \ssi x=1$ et $x-1>0\ssi x>1$.
    D’après le tableau de signes obtenu à la question A.4. on obtient le tableau de signes de $f\dsec(x)$.
    La fonction $f$ est donc convexe sur $[-3;\alpha]$ et $[1;4]$ et concave sur $[\alpha;1]$. $f\dsec(x)$ s’annule en $\alpha$ et $1$.
    Donc $\mathscr{C}_f$ possède deux points d’inflexion et le toboggan assurera de bonnes sensations.
    $\quad$

 

Ex 3

Exercice 3

  1. a. $\vect{AR}\begin{pmatrix}0\\3\\2\end{pmatrix}$ et $\vect{AT}\begin{pmatrix}-3\\0\\2\end{pmatrix}$
    Par conséquent
    $\begin{align*} AR&=\sqrt{0^2+3^2+2^2} \\
    &=\sqrt{13}\end{align*}$
    $\begin{align*} AT&=\sqrt{(-3)^2+0^2+2^2} \\
    &=\sqrt{13}\end{align*}$
    Ainsi $AR=AT$. Le triangle $ART$ est isocèle en $A$.
    b. $\quad$
    $\begin{align*} \vect{AR}.\vect{AT}&=0\times -(-3)+3\times 0+2\times 2\\
    &=4\end{align*}$
    $\quad$
    c. On a également $\vect{AR}.\vect{AT}=AR\times AT\times \cos \widehat{RAT}$.
    Par conséquent
    $\begin{align*} \cos \widehat{RAT}&=\dfrac{\vect{AR}.\vect{AT}}{AR\times AT} \\
    &=\dfrac{4}{13} \end{align*}$
    Donc $\widehat{RAT}\approx 72,1$°
    $\quad$
  2. a. D’une part
    $\begin{align*} \vec{n}.\vect{AR}&=2\times 0+(-2)\times 3+3\times 2\\
    &=0\end{align*}$
    D’autre part
    $\begin{align*} \vec{n}.\vect{AT}&=2\times (-3)+(-2)\times 0+3\times 2\\
    &=0\end{align*}$
    Ainsi $\vec{n}$ est orthogonal à deux vecteurs non colinéaires (l’angle $\widehat{RAT}$ n’est ni plat ni nul) du plan $(ART)$.
    $\vec{n}$ est donc un vecteur normal au plan $(ART)$.
    $\quad$
    b. Une équation du plan $(ART)$ est par conséquent de la forme $2x-2y+3z+d=0$.
    Or $A(6;0;2)$ appartient à ce plan.
    Donc $12-0+6+d=0 \ssi d=-18$
    Une équation cartésienne du plan $(ART)$ est $2x-2y+3z-18=0$.
    $\quad$
  3. a. $\vec{n}\begin{pmatrix}2\\-2\\3\end{pmatrix}$ est un vecteur directeur de la droite $\Delta$ et le point $S\left(3;\dfrac{5}{2};0\right)$ appartient à cette droite.
    Une représentation paramétrique de la droite $\Delta$ est bien $\begin{cases} x=3+2k\\y=\dfrac{5}{2}-2k\\z=3k\end{cases} \quad k\in \R$.
    $\quad$
    b. Prenons $k=1$ dans la représentation paramétrique précédente. Le point de coordonnées $\left(5;\dfrac{1}{2};3\right)$ appartient à la droite $\Delta$.
    $2\times 5-2\times \dfrac{1}{2}+3\times 3-18=10-1+9-18=0$. Le point de coordonnées $\left(5;\dfrac{1}{2};3\right)$ appartient au plan $(ART)$.
    Ainsi $L$ a pour coordonnées $\left(5;\dfrac{1}{2};3\right)$.
    $\quad$
  4. a. On a $D(0,8,0)$ et $K(0;4;4)$ donc $\vect{DK}\begin{pmatrix}0\\-4\\4\end{pmatrix}$ et $\vect{DN}\begin{pmatrix} 0\\-4t\\4t\end{pmatrix}$
    Par conséquent $\vect{DN}=t\vect{DK}$.
    Les points $D$, $N$ et $K$ sont alignés.
    $T\in[0;1]$ donc $N$ appartient au segment $[DK]$.
    $\quad$
    b. On a $\vect{SL}\begin{pmatrix}2\\-2\\3\end{pmatrix}$ et $\vect{SN}\begin{pmatrix} -3\\\dfrac{11}{2}-4t\\4t\end{pmatrix}$.$\begin{align*} &(SL) \text{ et }(SN)\text{ sont perpendiculaires}\\
    &\ssi\vect{SL}.\vect{SN}=0 \\
    &\ssi 2\times (-3)+(-2)\times  \left(\dfrac{11}{2}-4t\right)+3\times 4t=0 \\
    &\ssi -6-11+8t+12t=0 \\
    &\ssi 20t=17 \\
    &\ssi t=0,85\end{align*}$
    Le point $N$ doit donc avoir pour coordonnées $(0;4,6;3,4)$ pour que les deux rayons lasers soient perpendiculaires.
    $\quad$

 

Ex 4

Exercice 4

  1. $\quad$
    $\begin{align*}a&=\ln(9)+\ln\left(\dfrac{\sqrt{3}}{3}\right)+\ln\left(\dfrac{1}{9}\right) \\
    &=\ln(9)+\ln\left(\sqrt{3}\right)-\ln(3)-\ln(9)\\
    &=\dfrac{1}{2}\ln(3)-\ln(3) \\
    &=-\dfrac{1}{2}\ln(3)\end{align*}$
    Réponse d
    $\quad$
  2. $x-10>0\ssi x>10$ : l’équation est définie sur $]10;+\infty[$
    Sur $]10;+\infty[$
    $\begin{align*} &\ln(x)+\ln(x-10)=\ln(3)+\ln(7) \\
    &\ssi \ln\left(x(x-10)\right)=\ln(21) \\
    &\ssi x(x-10)=21 \\
    &\ssi x^2-10x-21=0\end{align*}$
    Le discriminant de $x^2-10x-21$ est $\Delta=184>0$.
    Les racines de ce polynômes sont $x_1=\dfrac{10-\sqrt{184}}{2}<0$ et $x_2=\dfrac{10+\sqrt{184}}{2}>10$
    Donc l’unique solution de $(E)$ est $\dfrac{10+\sqrt{184}}{2}$.
    Réponse c
    $\quad$
  3. La fonction $f$ est dérivable sur $]0;+\infty[$ en tant que somme et produit de fonctions dérivables.
    Pour tout réel $x\in ]0;+\infty[$,
    $\begin{align*} f'(x)&=2x\left(-1+\ln(x)\right)+x^2\times \dfrac{1}{x} \\
    &=-2x+2x\ln(x)+x \\
    &=x\left(2\ln(x)-1\right)\end{align*}$
    $\ln\left(\sqrt{e}\right)=\dfrac{1}{2}$
    Par conséquent $f’\left(\sqrt{e}\right)=0$.
    Une équation de la tangente au point d’abscisse $\sqrt{e}$ est donc $y=f\left(\sqrt{e}\right)$ soit $y=-\dfrac{1}{2}\e$.
    Réponse d
    $\quad$
  4. On répète $5$ fois de façon indépendante la même expérience de Bernoulli. On appelle $X$ la variable aléatoire qui compte le nombre de jetons jaunes tirés.
    $X$ suit donc la loi binomiale de paramètres $n=5$ et $p=\dfrac{2}{5}$
    Ainsi
    $\begin{align*} P(X=2)&=\dbinom{5}{2}\left(\dfrac{2}{5}\right)^2\left(\dfrac{3}{5}\right)^3\\
    &\approx 0,346\end{align*}$
    Réponse b
    $\quad$
  5. On reprend la variable aléatoire $X$ définie à la question précédente.
    $\begin{align*} P(X\pg 1)&=1-P(X=0) \\
    &=1-\left(\dfrac{3}{5}\right)^5\\
    &\approx 0,922\end{align*}$
    Réponse d
    $\quad$
  6. On reprend la variable aléatoire $X$ définie à la question 4..
    Son espérance mathématiques est :
    $\begin{align*} E(X)&=np\\
    &=5\times \dfrac{2}{5} \\
    &=2\end{align*}$
    Réponse c
    $\quad$

Énoncé

Exercice 1     7 points

Thème : probabilités, suites

Dans une région touristique, une société propose un service de location de vélos pour la journée.
La société dispose de deux points de location distinctes, le point A et le point B. Les vélos peuvent être empruntés et restitués indifféremment dans l’un où l’autre des deux points de location.
On admettra que le nombre total de vélos est constant et que tous les matins, à l’ouverture du service, chaque vélo se trouve au point A ou au point B.

D’après une étude statistique :

  • Si un vélo se trouve au point A un matin, la probabilité qu’il se trouve au point A le matin suivant est égale à $0,84$;
  • Si un vélo se trouve au point B un matin la probabilité qu’il se trouve au point B le matin suivant est égale à $0,76$.

À l’ouverture du service le premier matin, la société a disposé la moitié de ses vélos au point A, l’autre moitié au point B.

On considère un vélo de la société pris au hasard.

Pour tout entier naturel non nul n, on définit les évènements suivants :

  • $A_n$ : « le vélo se trouve au point A le $n$-ième matin »
  • $B_n$ : « le vélo se trouve au point B le $n$-ième matin ».

Pour tout entier naturel non nul $n$, on note $a_n$ la probabilité de l’évènement $A_n$ et $b_n$ la probabilité de l’évènement $B_n$. Ainsi $a_1 = 0,5$ et $b_1 = 0,5$.

  1. Recopier et compléter l’arbre pondéré ci-dessous qui modélise la situation pour les deux premiers matins :$\quad$
  2. a. Calculer $a_2$.
    $\quad$
    b. Le vélo se trouve au point A le deuxième matin. Calculer la probabilité qu’il se soit trouvé au point B le premier matin. La probabilité sera arrondie au millième.
    $\quad$
  3. a. Recopier et compléter l’arbre pondéré ci-dessous qui modélise la situation pour les $n$-ième et $n +1$-ième matins.
    $\quad$
    b. Justifier que pour tout entier naturel non nul $n$, $a_{n+1} = 0,6a_n +0,24$.
    $\quad$
  4. Montrer par récurrence que, pour tout entier naturel non nul $n$, $a_n = 0,6−0,1×0,6^{n−1}$.
    $\quad$
  5. Déterminer la limite de la suite $\left(a_n\right)$ et interpréter cette limite dans le contexte de l’exercice.
    $\quad$
  6. Déterminer le plus petit entier naturel $n$ tel que $a_n > 0,599$ et interpréter le résultat obtenu dans le contexte de l’exercice.
    $\quad$

$\quad$

Exercice 2     7 points

Thème : fonctions, fonction exponentielle

Partie A

Soit p la fonction définie sur l’intervalle $[-3 ; 4]$ par : $$p(x)=x^3-3x^2+5x+1$$

  1. Déterminer les variations de la fonction $p$ sur l’intervalle $[-3 ; 4]$.
    $\quad$
  2. Justifier que l’équation $p(x) = 0$ admet dans l’intervalle $[-3 ; 4]$ une unique solution qui sera notée $\alpha$.
    $\quad$
  3. Déterminer une valeur approchée du réel $\alpha$ au dixième près.
    $\quad$
  4. Donner le tableau de signes de la fonction $p$ sur l’intervalle $[-3 ; 4]$.
    $\quad$

Partie B

Soit $f$ la fonction définie sur l’intervalle $[-3 ; 4]$ par :$$f(x)=\dfrac{\e^x}{1+x^2}$$
On note $\mathscr{C}_f$ sa courbe représentative dans un repère orthogonal.

  1. a. Déterminer la dérivée de la fonction $f$ sur l’intervalle $[-3 ; 4]$.
    $\quad$
    b. Justifier que la courbe $\mathscr{C}_f$ admet une tangente horizontale au point d’abscisse $1$.
    $\quad$
  2. Les concepteurs d’un toboggan utilisent la courbe $\mathscr{C}_f$ comme profil d’un toboggan. Ils estiment que le toboggan assure de bonnes sensations si le profil possède au moins deux points d’inflexion.
    $\quad$
    a. D’après le graphique ci-dessus, le toboggan semble-t-il assurer de bonnes sensations ?
    Argumenter.
    b. On admet que la fonction $f\dsec$, dérivée seconde de la fonction $f$ , a pour expression pour tout réel $x$ de l’intervalle $[-3 ; 4]$ :
    $$f\dsec(x)=\dfrac{p(x)(x-1)\e^x}{\left(1+x^2\right)^3}$$
    où $p$ est la fonction définie dans la partie A.
    En utilisant l’expression précédente de $f\dsec$, répondre à la question : « le toboggan assure-t-il de bonnes sensations ? ». Justifier.
    $\quad$

$\quad$

Exercice 3     7 points

Thème : géométrie dans l’espace

Une exposition d’art contemporain a lieu dans une salle en forme de pavé droit de largeur $6$ m, de longueur $8$ m et de hauteur $4$ m.
Elle est représentée par le parallélépipède rectangle $OBCDEFGH$ où $OB = 6$ m, $OD = 8$ m et $OE = 4$ m.
On utilise le repère orthonormé $\Oijk$ tel que $\vec{i}=\dfrac{1}{6}\vect{OB}$, $\vec{j}=\dfrac{1}{8}\vect{OD}$ et $\vec{k}=\dfrac{1}{4}\vect{OE}$.

 

Dans ce repère on a, en particulier $C(6; 8; 0)$, $F(6; 0; 4)$ et $G(6; 8; 4)$.
Une des œuvres exposées est un triangle de verre représenté par le triangle $ART$ qui a pour sommets $A(6; 0; 2)$, $R(6; 3; 4)$ et $T(3; 0; 4)$, Enfin, $S$ est le point de coordonnées $\left(3;\dfrac{5}{2};0\right)$.

  1. a. Vérifier que le triangle $ART$ est isocèle en $A$.
    $\quad$
    b. Calculer le produit scalaire $\vect{AR}.\vect{AT}$.
    $\quad$
    c. En déduire une valeur approchée à $0,1$ degré près de l’angle $\widehat{RAT}$.
    $\quad$
  2. a. Justifier que le vecteur $\vec{n}\begin{pmatrix}2\\-2\\3\end{pmatrix}$ est un vecteur normal au plan $(ART)$.
    $\quad$
    b. En déduire une équation cartésienne du plan $(ART)$.
    $\quad$
  3. Un rayon laser dirigé vers le triangle $ART$ est émis du plancher à partir du point $S$. On admet que ce rayon est orthogonal au plan $(ART)$.
    a. Soit $\Delta$ la droite orthogonale au plan $(ART)$ et passant par le point $S$.
    Justifier que le système ci-dessous est une représentation paramétrique de la droite $\Delta$ : $$\begin{cases} x=3+2k\\[3pt]y=\dfrac{5}{2}-2k\\[3pt]z=3k\end{cases} \quad, \text{avec } k\in \R$$
    $\quad$
    b. Soit $L$ le point d’intersection de la droite $\Delta$, avec le plan $(ART)$.
    Démontrer que $L$ a pour coordonnées $\left(5;\dfrac{1}{2};3\right)$.
    $\quad$
  4. L’artiste installe un rail représenté par le segment $[DK]$ ou $K$ est le milieu du segment $[EH]$.
    Sur ce rail, il positionne une source lumineuse laser en un point $N$ du segment $[DK]$ et il oriente ce second rayon laser vers le point $S$.
    $\quad$
    $\quad$
    a. Montrer que, pour tout réel $t$ de l’intervalle $[0; 1]$, le point $N$ de coordonnées $(0 ; 8−4t ; 4t)$ est un point du segment $[DK]$.
    $\quad$
    b. Calculer les coordonnées exactes du point $N$ tel que les deux rayons laser représentés par les segments $[SL]$ et $[SN]$ soient perpendiculaires.
    $\quad$

$\quad$

Exercice 4     7 points

Thème : : fonction logarithme népérien, probabilités

Cet exercice est un questionnaire à choix multiples (QCM) qui comprend six questions. Les six questions sont indépendantes. Pour chacune des questions, une seule des quatre réponses est exacte. Le candidat indiquera sur sa copie le numéro de la question suivi de la lettre correspondant à la réponse exacte.
Aucune justification n’est demandée.
Une réponse fausse, une réponse multiple ou une absence de réponse ne rapporte ni n’enlève aucun point.

Question 1

Le réel $a$ est définie par $a = \ln(9)+\ln\left(\dfrac{\sqrt{3}}{3}\right)+\ln\left(\dfrac{1}{9}\right)$ est égal à :
a. $1-\dfrac{1}{2}\ln(3)$
b. $\dfrac{1}{2}\ln(3)$
c. $3\ln(3)-\dfrac{1}{2}$
d. $-\dfrac{1}{2}\ln(3)$
$\quad$

Question 2

On note $(E)$ l’équation suivante $\ln(x) +\ln(x −10) = ln (3)+ln (7)$ d’inconnue le réel $x$.
a. $3$ est solution de $(E)$.
b. $5-\sqrt{46}$ est solution de $(E)$.
c. L’équation $(E)$ admet une unique solution réelle.
d. L’équation $(E)$ admet deux solutions réelles.
$\quad$

Question 3

La fonction $f$ est définie sur l’intervalle $]0 ; +\infty[$ par l’expression $f(x)=x^2\left(-1+\ln(x)\right)$.
On note $\mathscr{C}_f$ sa courbe représentative dans le plan muni d’un repère.
a. Pour tout réel $x$ de l’intervalle $]0 ; +\infty[$, $f'(x)=2x+\dfrac{1}{x}$.
b. La fonction $f$ est croissante sur l’intervalle $]0 ; +\infty[$.
c. $f’\left(\sqrt{\e}\right)$ est différent de $0$.
d. La droite d’équation $y=-\dfrac{1}{2}\e$ est tangente à la courbe $\mathscr{C}_f$ au point d’abscisse $\sqrt{\e}$.
$\quad$

Question 4

Un sac contient $20$ jetons jaunes et $30$ jetons bleus. On tire successivement et avec remise $5$ jetons du sac.
La probabilité de tirer exactement $2$ jetons jaunes, arrondie au millième, est :
a. $0,683$
b. $0,346$
c. $0,230$
d. $0,165$
$\quad$

Question 5

Un sac contient $20$ jetons jaunes et $30$ jetons bleus. On tire successivement et avec remise $5$ jetons du sac.
La probabilité de tirer au moins un jeton jaune, arrondie au millième, est :
a. $0,078$
b. $0,259$
c. $0,337$
d. $0,922$
$\quad$

Question 6

Un sac contient $20$ jetons jaunes et $30$ jetons bleus.
On réalise l’expérience aléatoire suivante : on tire successivement et avec remise cinq jetons du sac.
On note le nombre de jetons jaunes obtenus après ces cinq tirages.
Si on répète cette expérience aléatoire un très grand nombre de fois alors, en moyenne, le nombre de jetons jaunes est égal à:
a. $0,4$
b. $1,2$
c. $2$
d. $2,5$
$\quad$

$\quad$

 

Bac – Spécialité mathématiques – Amérique du Nord – sujet 1 – 18 mai 2022

Amérique du nord – 18 mai 2022

Spécialité maths – Sujet 1- Correction

L’énoncé de ce sujet de bac est disponible ici.

Ex 1

Exercice 1

  1. a. On obtient l’arbre pondéré suivant:
    $\quad$
    $\quad$
    b. $\left(V,\conj{V}\right)$ forme un système complet d’événements fini.
    D’après la formule des probabilités totales on a :
    $\begin{align*} P(R)&=P(R\cap V)+P\left(R\cap \conj{V}\right) \\
    &=P(V)\times P_V(R)+P\left(\conj{V}\right)\times P_{\conj{V}}(R)\\
    &=\dfrac{2}{3}\times \dfrac{1}{50}+\dfrac{1}{3}\times \dfrac{1}{10} \\
    &=\dfrac{7}{150}\end{align*}$
    La probabilité que Paul rate son train est égale à $\dfrac{7}{150}$.
    $\quad$
    c. On veut calculer :
    $\begin{align*} P_R(V)&=\dfrac{P(R\cap V)}{P(R)} \\
    &=\dfrac{~\dfrac{2}{150}~}{\dfrac{7}{150}} \\
    &=\dfrac{2}{7}\end{align*}$
    La probabilité que Paul ait pris son vélo pour rejoindre la gare sachant qu’il a raté son train est égale à $\dfrac{2}{7}$.
    $\quad$
  2. a. On répète $20$ fois de façon indépendante la même expérience de Bernoulli. $X$ compte le nombre de jours où Paul prend son vélo sur ces $20$ jours.
    $X$ suit donc la loi binomiale de paramètres $n=20$ et $p=\dfrac{2}{3}$.
    $\quad$
    b.
    $\begin{align*} P(X=10)&=\dbinom{20}{10}\times \left(\dfrac{2}{3}\right)^{10}\times \left(\dfrac{1}{3}\right)^{10} \\
    &\approx 0,054\end{align*}$
    La probabilité que Paul prenne son vélo exactement $10$ jours sur ces $20$ jours pour se rendre à la gare est environ égale à $0,054$.
    $\quad$
    c.
    $\begin{align*} P(X\pg 10)&=1-P(X\pp 9) \\
    &\approx 0,962\end{align*}$
    La probabilité que Paul prenne son vélo au moins $10$ jours sur ces $20$ jours pour se rendre à la gare est environ égale à $0,962$.
    d. L’espérance mathématique de $X$ est
    $\begin{align*} E(X)&=np\\
    &=20\times \dfrac{2}{3}\\
    &=\dfrac{40}{3}\end{align*}$
    En moyenne, sur un période choisie au hasard de $20$ jours, Paul prend son vélo environ $13$ jours pour se rendre à la gare.
    $\quad$
  3. L’espérance mathématique de $T$ est :
    $\begin{align*} E(T)&=10\times P(T=10)+11\times P(T=11)+\ldots+18\times P(T=18) \\
    &=13,5\end{align*}$
    En moyenne Paul met $13,5$ minutes pour se rendre à la gare en voiture.
    $\quad$

Ex 2

Exercice 2

  1. a. Pour tout entier naturel $n$ on note $P(n):~T_n\pg 20$.
    Initialisation : $T_0=180\pg 20$ donc $P(0)$ est vraie.
    $\quad$
    Hérédité : Soit $n\in \N$. On suppose $P(n)$ vraie.
    $\begin{align*} T_n\pg 20&\ssi 0,955T_n \pg 19,1 \\
    &\ssi 0,955T_n+0,9 \pg 20 \\
    &\ssi T_{n+1} \pg 20\end{align*}$
    Donc $P(n+1)$ est vraie.
    $\quad$
    Conclusion : La propriété est vraie au rang $0$ et est héréditaire.
    Pour tout entier naturel $n$, $T_n\pg 20$.
    $\quad$
    b. Soit $n\in \N$.
    $\begin{align*} T_{n+1}-T_n&=0,955T_n+0,9 -T_n \\
    &=-0,045T_n+0,9 \\
    &=-0,045\left(T_n-20\right)\end{align*}$
    $\quad$
    Pour tout $n\in \N$ on a $T_n\pg 20 \ssi T_n-20\pg 0$.
    Ainsi $T_{n+1}-T_n\pp 0$.
    Par conséquent $\left(T_n\right)$ est décroissante.
    $\quad$
    c. La suite $\left(T_n\right)$ est décroissante et minorée par $20$. Elle converge donc vers un réel $\ell$.
    $\quad$
  2. a. Soit $n\in \N$. $u_n=T_n-20\ssi T_n=u_n+20$
    $\begin{align*} u_{n+1}&=T_{n+1}-20 \\
    &=0,955T_n+0,9-20 \\
    &=0,955T_n-19,1\\
    &=0,955\left(u_n+20\right)-19,1 \\
    &=0,955u_n+19,1-19,1\\
    &=0,955u_n\end{align*}$
    Par conséquent, la suite $\left(u_n\right)$ est géométrique de raison $0,955$ et de premier terme $u_0=160$.
    $\quad$
    b. Pour tout entier naturel $n$ on a $u_n=160\times 0,955^n$.
    Donc $T_n=u_n+20=20+160\times 0,955^n$.
    $\quad$
    c. $-1<0,955<1$ donc $\lim\limits_{n\to +\infty} 0,955^n=0$.
    Par conséquent $\lim\limits_{n\to +\infty} T_n=20$.
    $\quad$
    d.
    $\begin{align*} T_n\pp 120&\ssi 20+160\times 0,955^n \pp 120 \\
    &\ssi 160\times 0,955^n \pp 100 \\
    &\ssi 0,955^n \pp 0,625 \\
    &\ssi n\ln(0,955) \pp \ln(0,625) \\
    &\ssi n\pg \dfrac{\ln(0,625)}{\ln(0,955)} \end{align*}$
    Or $\dfrac{\ln(0,625)}{\ln(0,955)} \approx 10,2$.
    L’ensemble solution de $T_n\pp 120$ est l’ensemble des entiers naturels supérieurs ou égaux à $11$.
    $\quad$
  3. a. La température au centre du gâteau va décroitre jusqu’à atteindre la température ambiante.
    Il était donc prévisible que $\lim\limits_{n\to +\infty} T_n=20$.
    $\quad$
    b. La fonction $\texttt{temp}$ renvoie le plus petit entier naturel $n$ tel que $T_n\pp x$.
    D’après la question 2.d. la commande $\texttt{temp(120)}$ renvoie $11$.
    Cela signifie que la température au centre du gâteau devient inférieure à $120$ degré Celsius au bout de $11$ minutes.
    $\quad$

Ex 3

Exercice 3

  1. a. $\vect{JK}\begin{pmatrix} -1\\2\\0\end{pmatrix}$ et $\vect{JL}\begin{pmatrix} -4\\-2\\-3\end{pmatrix}$
    Donc
    $\begin{align*} \vect{JK}.\vect{JL}&=-1\times (-4)+2\times (-2)+0\times (-3) \\
    &=4-4\\
    &=0\end{align*}$
    Les vecteurs $\vect{JK}$ et $\vect{JL}$ sont donc orthogonaux. Le triangle $JKL$ est par conséquent rectangle en $J$.
    $\quad$
    b.
    $\begin{align*} JK&=\sqrt{(-1)^2+2^2+0^2} \\
    &=\sqrt{5}\end{align*}$
    $\begin{align*} JL&=\sqrt{(-4)^2+(-2)^2+(-3)^2} \\
    &=\sqrt{29}\end{align*}$
    L’aire du triangle $JKL$ est donc :
    $\begin{align*} \mathscr{A}&=\dfrac{JK\times JL}{2} \\
    &=\dfrac{\sqrt{5}\times \sqrt{29}}{2} \\
    &=\dfrac{\sqrt{145}}{2} \text{ cm}^2\end{align*}$
    c. Dans le triangle $JKL$ rectangle en $J$ on a $\tan \widehat{JKL}=\dfrac{JL}{JK}$
    Soit $\tan \widehat{JKL}=\dfrac{\sqrt{29}}{\sqrt{5}}$
    Par conséquent $\widehat{JKL} \approx 67,5$°
    $\quad$
  2. a. D’une part :
    $\begin{align*}\vect{JK}.\vec{n}&=6\times (-1)+3\times 2+0\times (-10) \\
    &=-6+6\\
    &=0\end{align*}$
    D’autre part
    $\begin{align*}\vect{JL}.\vec{n}&=6\times (-4)+3\times (-2)+(-10)\times (-3)
    &=-24-6+30\\
    &=0\end{align*}$
    Le vecteur $\vec{n}$ est donc orthogonal à deux vecteur non colinéaires (puisqu’orthogonaux) du plan $(JKL)$.
    $\vec{n}$ est par conséquent un vecteur normal au plan $(JKL)$.
    $\quad$
    b. Une équation cartésienne du plan $(JKL)$ est donc de la forme $6x+3y-10z+d=0$.
    Le point $J(2;0;1)$ appartient au plan $(JKL)$.
    Par conséquent $12+0-10+d=0 \ssi d=-2$
    Une équation cartésienne du plan $(JKL)$ est $6x+3y-10z-2=0$.
    $\quad$
  3. a. $\vec{n}$ est un vecteur directeur de $\Delta$.
    Une représentation paramétrique de $\Delta$ est donc $\begin{cases} x=10+6t\\y=9+3t\\z=-6-10t\end{cases} \qquad ,t\in \R$.
    $\quad$
    b. On résout le système :
    $\begin{align*} \begin{cases} x=10+6t\\y=9+3t\\z=-6-10t\\6x+3y-10z-2=0\end{cases}&\ssi \begin{cases} x=10+6t\\y=9+3t\\z=-6-10t\\6(10+6t)+3(9+3t)-10(-6-10t)-2=0\end{cases}\\
    &\ssi \begin{cases} x=10+6t\\y=9+3t\\z=-6-10t\\60+36t+27+9t+60+100t-2=0\end{cases}\\
    &\ssi \begin{cases} x=10+6t\\y=9+3t\\z=-6-10t\\145t+145=0\end{cases}\\
    &\ssi \begin{cases}t=-1\\ x=4\\y=6\\z=4\end{cases}\end{align*}$
    Ainsi $H$ a pour coordonnées $(4;6;4)$.
    $\quad$
    c. $\vect{HT}\begin{pmatrix} 6\\3\\-10\end{pmatrix}$
    Donc
    $\begin{align*} HT&=\sqrt{6^2+3^2+(-10)^2} \\
    &=\sqrt{145}\end{align*}$
    Par conséquent :
    $\begin{align*} V&=\dfrac{1}{3}\mathscr{A}\times HT \\
    &=\dfrac{1}{3}\times \dfrac{\sqrt{145}}{2}\times \sqrt{145}\\
    &=\dfrac{145}{6} \text{ cm}^3\end{align*}$
    $\quad$

Ex 4

Exercice 4

  1. Soit $x\in \R$
    $\begin{align*} 1-\dfrac{1-\e^x}{1+\e^x}&=\dfrac{1+\e^x-1+\e^x}{1+\e^x} \\
    &=\dfrac{2\e^x}{1+\e^x} \\
    &=\dfrac{2\e^x}{\e^x\left(\e^{-x}+1\right)} \\
    &=\dfrac{2}{1+\e^{-x}}\end{align*}$
    Affirmation 1 vraie
    $\quad$
  2. $\quad$
    $\begin{align*} g(x)=\dfrac{1}{2} &\ssi \dfrac{\e^x}{\e^x+1}=\dfrac{1}{2} \\
    &\ssi 2\e^x=\e^x+1 \\
    &\ssi \e^x=1 \\
    &\ssi x=0\end{align*}$
    L’équation $g(x)=\dfrac{1}{2}$ admet une unique solution : $0$.
    Affirmation 2 vraie
    $\quad$
  3. La fonction $f$ est dérivable sur $\R$ en tant que produit de fonctions dérivables.
    Pour tout réel $x$ on a
    $\begin{align*} f'(x)&=2x\e^{-x}-x^2\e^{-x} \\
    &=\left(2x-x^2\right)\e^{-x} \\
    &=x(2-x)\e^{-x}\end{align*}$
    Si l’axe des abscisses est tangent à la courbe $C$ en $A\left(x_A;y_A\right)$ alors $f’\left(x_A\right)=0$.
    Or $f'(x)=0 \ssi x(2-x)=0$ car la fonction exponentielle est strictement positive.
    $x(2-x)=0 \ssi x=0$ ou $x=2$.
    Cependant $f(0)=0$ et $f(2)=4\e^{-2}\neq 0$. Le point de coordonnées $(0;0)$ appartient à l’axe des abscisses mais le point de coordonnées $\left(2;4\e^{-2}\right)$ n’appartient pas à cet axe.
    L’axe des abscisses est tangent à la courbe $C$ qu’en l’origine du repère.
    Affirmation 3 vraie
    $\quad$
  4. La fonction $h$ est deux fois dérivable sur $\R$ en tant que produit de fonctions deux fois dérivables.
    Soit $x\in \R$
    $\begin{align*} h'(x)&=\e^x\left(1-x^2\right)-2x\e^x \\
    &=\e^x\left(-x^2-2x+1\right)\end{align*}$
    $\begin{align*} h\dsec(x)&=\e^x\left(-x^2-2x+1\right)+\e^{-x}(-2x-2)\\
    &=\e^x\left(-x^2-2x+1-2x-2\right) \\
    &=\e^x\left(-x^2-4x-1\right)\end{align*}$
    La fonction exponentielle est strictement positive sur $\R$. Le signe de $h\dsec(x)$ ne dépend donc que de celui de $-x^2-4x-1$.
    Le discriminant de ce polynôme du second degré est $\Delta=12>0$.
    Ainsi l’équation $-x^2-4x-1=0$ admet $2$ solutions distinctes et $h\dsec(x)$ change deux fois de signe en s’annulant.
    La courbe représentative de la fonction $h$ admet donc deux points d’inflexion.
    Affirmation 4 fausse
    $\quad$
  5. Soit $x>0$. $\dfrac{\e^x}{\e^x+x}=\dfrac{1}{1+\dfrac{x}{\e^x}}$
    Or, par croissances comparées $\lim\limits_{x\to +\infty} \dfrac{\e^x}{x}=+\infty$ donc $\lim\limits_{x\to +\infty} \dfrac{x}{\e^x}=0$
    Par conséquent $\lim\limits_{x\to +\infty} \dfrac{1}{1+\dfrac{x}{\e^x}}=1$
    Affirmation 5 fausse
    $\quad$
  6. $\quad$
    $\begin{align*} 1+\e^{2x}\pg 2\e^x &\ssi \left(\e^x\right)^2-2\e^x+1 \pg 0\\
    &\ssi \left(\e^x-1\right)^2 \pg 0\end{align*}$
    Cette dernière inégalité est vraie pour tout réel $x$.
    Affirmation 6 vraie
    $\quad$

 

Énoncé

Exercice 1     7 points

Thème : probabilités

Chaque chaque jour où il travaille, Paul doit se rendre à la gare pour rejoindre son lieu de travail en train. Pour cela, il prend son vélo deux fois sur trois et, si il ne prend pas son vélo, il prend sa voiture.

  1. lorsqu’il prend son vélo pour rejoindre la gare, Paul ne rate le train qu’une fois sur $50$ alors que, lorsqu’il prend sa voiture pour rejoindre la gare Paul rate son train une fois sur $10$.
    On considère une journée au hasard lors de laquelle Paul sera à la gare pour prendre le train qui le conduira au travail.
    On note :
    • $V$ l’évènement « Paul prend son vélo pour rejoindre la gare »;
    • $R$ l’évènement « Paul rate son train ».
    a. Faire un arbre pondéré résumant la situation.
    $\quad$
    b. Montrer que la probabilité que Paul rate son train est égale à $\dfrac{7}{150}$.
    $\quad$
    c. Paul a raté son train. Déterminer la valeur exacte de la probabilité qu’il ait pris son vélo pour rejoindre la gare.
    $\quad$
  2. On choisit au hasard un mois pendant lequel Paul s’est rendu $20$ jours à la gare pour rejoindre son lieu de travail selon les modalités décrites en préambule.
    On suppose que, pour chacun de ces $20$ jours, le choix entre le vélo et la voiture est indépendant des choix des autres jours.
    On note $X$ la variable aléatoire donnant le nombre de jours où Paul prend son vélo sur ces $20$ jours.
    a. Déterminer la loi suivie par la variable aléatoire $X$. Préciser ses paramètres.
    $\quad$
    b. Quelle est la probabilité que Paul prenne son vélo exactement 10 jours sur ces $20$ jours pour se rendre à la gare ? On arrondira la probabilité cherchée à $10^{-3}$.
    $\quad$
    c. Quelle est la probabilité que Paul prenne son vélo au moins $10$ jours sur ces $20$ jours pour se rendre à la gare ? On arrondira la probabilité cherchée à $10^{-3}$.
    $\quad$
    d. En moyenne, combien de jours sur une période choisie au hasard de $20$ jours pour se rendre à la gare, Paul prend-il son vélo ? On arrondira la réponse à l’entier.
    $\quad$
  3. Dans le cas où Paul se rend à la gare en voiture, on note $T$ la variable aléatoire donnant le temps de trajet nécessaire pour se rendre à la gare. La durée du trajet est donnée en minutes, arrondie à la minute. La loi de probabilité de $T$ est donnée par le tableau ci-dessous :
    $\begin{array}{|l|c|c|c|c|c|c|c|c|c|}
    \hline
    k\text{ (en minutes)}&10&11&12&13&14&15&16&17&18\\
    \hline
    P(T=k)&0,14&0,13&0,13&0,12&0,12&0,11&0,10&0,08&0,07\\
    \hline
    \end{array}$
    Déterminer l’espérance de la variable aléatoire $T$ et interpréter cette valeur dans le contexte de l’exercice.
    $\quad$

$\quad$

Exercice 2     7 points

Thème : suites

Dans cet exercice, on considère la suite $\left(T_n\right)$ définie par :
$$T_0 = 180 \text{ et, pour tout entier naturel }n,~ T_{n+1} = 0,955T_n +0,9$$

  1. a. Démontrer par récurrence que, pour tout entier naturel $n$, $T_n \pg 20$.
    $\quad$
    b. Vérifier que pour tout entier naturel $n$, $T_{n+1}-T_n = −0,045\left(T_n-20\right))$. En déduire le sens de
    variation de la suite $\left(T_n\right)$.
    $\quad$
    c. Conclure de ce qui précède que la suite $\left(T_n\right)$ est convergente. Justifier.
    $\quad$
  2. Pour tout entier naturel $n$, on pose : $u_n = T_n-20$.
    a. Montrer que la suite $\left(u_n\right)$ est une suite géométrique dont on précisera la raison.
    $\quad$
    b. En déduire que pour tout entier naturel $n$, $T_n = 20+160\times 0,955^n$.
    $\quad$
    c. Calculer la limite de la suite $\left(T_n\right)$.
    $\quad$
    d. Résoudre l’inéquation $T_n\pp 120$ d’inconnue $n$ entier naturel.
    $\quad$
  3. Dans cette partie, on s’intéresse à l’évolution de la température au centre d’un gâteau après sa sortie du four.
    On considère qu’à la sortie du four, la température au centre du gâteau est de $180$° C et celle de l’air ambiant de $20$° C.
    La loi de refroidissement de Newton permet de modéliser la température au centre du gâteau par la suite précédente $\left(T_n\right)$. Plus précisément, $T_n$ représente la température au centre du gâteau, exprimée en degré Celsius, n minutes après sa sortie du four.
    a. Expliquer pourquoi la limite de la suite $\left(T_n\right)$ déterminée à la question 2. c. était prévisible dans le contexte de l’exercice.
    $\quad$
    b. On considère la fonction Python ci-dessous :
    $$\begin{array}{|l|}
    \hline
    \text{def temp(x):}\\
    \quad \text{T = 180} \\
    \quad \text{n = 0}\\
    \quad \text{while T > x:} \\
    \qquad \text{T = 0.955 * T + 0.9}\\
    \qquad \text{n = n + 1}\\
    \quad \text{return n}\\
    \hline
    \end{array}$$
    Donner le résultat obtenu en exécutant la commande $\texttt{temp(120)}$.
    Interpréter le résultat dans le contexte de l’exercice.
    $\quad$

$\quad$

Exercice 3     7 points

Thème : géométrie dans l’espace

Dans l’espace muni d’un repère orthonormé $\Oijk$ d’unité $1$ cm, on considère les points suivants :
$$J(2 ; 0 ; 1),~~ K(1 ; 2 ; 1) \text{ et } L(−2 ; −2 ; −2)$$

  1. a. Montrer que le triangle $JKL$ est rectangle en $J$.
    $\quad$
    b. Calculer la valeur exacte de l’aire du triangle $JKL$ en cm$^2$.
    $\quad$
    c. Déterminer une valeur approchée au dixième près de l’angle géométrique $\widehat{JKL}$.
    $\quad$
  2. a. Démontrer que le vecteur $\vec{n}$ de coordonnées$\begin{pmatrix}6\\3\\-10\end{pmatrix}$ est un vecteur normal au plan $(JKL)$.
    $\quad$
    b. En déduire une équation cartésienne du plan $(JKL)$.

$\quad$
Dans la suite, $T$ désigne le point de coordonnées $(10 ; 9 ; −6)$.

  1. a. Déterminer une représentation paramétrique de la droite $\Delta$ orthogonale au plan $(JKL)$ et passant par $T$.
    $\quad$
    b. Déterminer les coordonnées du point $H$, projeté orthogonal du point $T$ sur le plan $(JKL)$.
    $\quad$
    c. On rappelle que le volume $V$ d’un tétraèdre est donné par la formule : $$V=\dfrac{1}{3}\mathscr{B}\times h \text{où $\mathscr{B}$ désigne l’aire d’une base et $h$ la hauteur correspondante}$$
    Calculer la valeur exacte du volume du tétraèdre $JKLT$ en cm$^3$.
    $\quad$

$\quad$

Exercice 4     7 points

Thème : fonction exponentielle

Pour chacune des affirmations suivantes, indiquer si elle est vraie ou fausse. Justifier chaque réponse.

  1. Affirmation 1 : Pour tout réel $x$ : $1-\dfrac{1-\e^x}{1+\e^x}=\dfrac{2}{1+\e^{-x}}$.
    $\quad$
  2. On considère la fonction $g$ définie sur $\R$ par $g(x) =
    \dfrac{\e^x}{\e^x+1}$.
    Affirmation 2 : L’équation $g(x) = \dfrac{1}{2}$ admet une unique solution dans $\R$.
    $\quad$
  3. On considère la fonction $f$ définie sur $\R$ par $f(x) = x^2\e^{-x}$ et on note $\mathscr{C}$ sa courbe dans un repère orthonormé.
    Affirmation 3 : L’axe des abscisses est tangent à la courbe $\mathscr{C}$ en un seul point.
    $\quad$
  4. On considère la fonction $h$ définie sur $\R$ par $h(x)=\e^x\left(1-x^2\right)$.
    Affirmation 4 : Dans le plan muni d’un repère orthonormé, la courbe représentative de la fonction $h$ n’admet pas de point d’inflexion.
    $\quad$
  5. Affirmation 5 : $\lim\limits_{x\to +\infty} \dfrac{\e^x}{\e^x+x}=0$.
    $\quad$
  6. Affirmation 6 : Pour tout réel $x$, $1+\e^{2x}\pg 2\e^x$.
    $\quad$

$\quad$

 

Bac – Spécialité mathématiques – Centres étrangers – sujet 1 – 18 mai 2022

Centres étrangers – Liban – 18 mai 2022

Spécialité maths – Sujet 1- Correction

L’énoncé de ce sujet de bac est disponible ici.

Ex 1

Exercice 1

Partie A

  1. On obtient l’arbre pondéré suivant :
    $\quad$
    $\quad$
  2. On a
    $\begin{align*} P(J\cap C)&=P(J)\times P_J(C)\\
    &=0,2\times 0,06 \\
    &=0,012\end{align*}$
    $\quad$
  3. $\left(J,\conj{J}\right)$ forme un système complet d’événements.
    D’après la formule des probabilités totales :
    $\begin{align*} P(C)&=P(J\cap C)+P\left(\conj{J}\cap C\right) \\
    &=0,012+P\left(\conj{J}\right)P_{\conj{J}}(C)\\
    &=0,012+0,8\times 0,125 \\
    &=0,112\end{align*}$
    $\quad$
  4. On veut calculer :
    $\begin{align*} P_C\left(\conj{J}\right)&=\dfrac{P\left(C\cap \conj{J}\right)}{P(C)} \\
    &=\dfrac{0,8\times 0,125}{0,112} \\
    &\approx 0,893\end{align*}$
    La probabilité que le skieur ait un forfait SÉNIOR sachant qu’il a choisi l’option coupe-file est environ égale à $0,893$.
    $\quad$
  5. Un skieur ayant choisi l’option coupe-file a moins de vingt-cinq ans ou plus de vingt-cinq ans.
    Ainsi :
    $\begin{align*} P_C(J)&=1-P_C\left(\conj{J}\right) \\
    &\approx 0,107\\
    &<0,15\end{align*}$
    L’affirmation est donc vraie.
    $\quad$

Partie B

  1. $X$ suit la loi binomiale de paramètres $n=30$ et $p=0,112$.
    $\quad$
  2. On veut calculer :
    $\begin{align*} P(X\pg 1)&=1-P(X=0) \\
    &=1-(1-0,112)^{30} \\
    &=1-0,888^{30} \\
    &\approx 0,972\end{align*}$
    La probabilité qu’au moins un des $30$ skieurs ait choisi l’option coupe-file est environ égale à $0,972$.
    $\quad$
  3. On veut calculer :
    $\begin{align*} P(X\pp 1)&=P(X=0)+P(X=1) \\
    &=0,888^{30}+\dbinom{30}{1}0,112^1\times 0,888^{29} \\
    &\approx 0,136\end{align*}$
    La probabilité qu’au plus un des $30$ skieurs ait choisi l’option coupe-file est environ égale à $0,136$.
    $\quad$
  4. L’espérance mathématique de $X$ est :
    $\begin{align*} E(X)&=np\\
    &=30\times 0,112 \\
    &=3,36\end{align*}$
    $\quad$

Ex 2

Exercice 2

  1. On appelle $v_n$ le volume d’eau, en litres, contenu dans la bouteille au bout de $n$ heures.
    On a donc, pour tout entier naturel $n$, $v_{n+1}=(1-0,15)v_n$ soit $v_{n+1}=0,85 v_n$.
    $\left(v_n\right)$ est donc une suite géométrique de raison $0,85$ et de premier terme $1$.
    Par conséquent, pour tout entier naturel $n$, $v_n=0,85^n$.
    $\begin{align*} u_n \pp 0,25&\ssi 0,85^n \pp 0,25 \\
    &\ssi n\ln(0,85)\pp \ln(0,25) \\
    &\ssi n\pg \dfrac{\ln(0,25)}{\ln(0,85)} \qquad \text{car } \ln(0,85)<0 \end{align*}$
    Or $\dfrac{\ln(0,25)}{\ln(0,85)}\approx 8,53$.
    C’est donc au bout de $9$ heures que le volume d’eau devient inférieur à un quart de litre.
    Réponse c
    $\quad$
  2. Pour tout $n\in \N$, on pose $P(n):~u_n=6$.
    Initialisation : $u_0=6$ donc $P(0)$ est vraie.
    $\quad$
    Hérédité : Soit $n\in \N$. On suppose $P(n)$ vraie.
    $\begin{align*} u_{n+1}&=\dfrac{1}{2} u_n+3 \\
    &=\dfrac{1}{2}\times 6+3 \\
    &=6\end{align*}$
    Donc $P(n+1)$ est vraie.
    $\quad$
    Conclusion : La propriété est vraie au rang $0$ et est héréditaire.
    Donc, pour tout entier naturel $n,~ u_n=6$.
    Réponse d
    $\quad$
  3. Soit $x\in ]0;+\infty[$
    $\begin{align*} f(2x)&=4\ln(3\times 2x) \\
    &=4\left(\ln(2)+\ln(3x)\right) \\
    &=4\ln(2)+4\ln(3x)\\
    &=\ln\left(2^4\right)+f(x)\\
    &=\ln(16)+f(x)\end{align*}$
    Réponse b
    $\quad$
  4. Pour tout réel $x>1$ on a $g(x)=\dfrac{\ln(x)}{x}\times \dfrac{x}{x-1}$.
    Par croissances comparées, $\lim\limits_{x\to +\infty} \dfrac{\ln(x)}{x}=0$
    D’après la limite du quotient des termes de plus haut degré $\lim\limits_{x\to +\infty} \dfrac{x}{x-1}=\lim\limits_{x\to +\infty} \dfrac{x}{x}=1$.
    Ainsi $\lim\limits_{x\to +\infty} g(x)=0$ : $C_g$ admet une asymptote horizontale d’équation $y=0$.
    $\quad$
    $C_g$ ne peut avoir d’asymptote verticale qu’en $1$.
    Pour tout réel $x\in ]1;+\infty[$ on a $g(x)=\dfrac{\ln(x)-\ln(1)}{x-1}$.
    Ainsi $g(x)$ est le taux d’accroissement de la fonction $\ln$ entre $1$ et $x$.
    Donc $\lim\limits_{x\to 1^+} g(x)=\ln'(1)=\dfrac{1}{1}$.
    $C_g$ n’a pas d’asymptote verticale.
    Réponse c
    $\quad$
  5. $h$ est définie sur $]0;2]$. Par conséquent :
    $\begin{align*} h(x)=0&\ssi 1+2\ln(x)=0 \\
    &\ssi 2\ln(x)=-1 \\
    &\ssi \ln(x)=-0,5 \\
    &\ssi x=\e^{-0,5}\end{align*}$
    Or $\e^{-0,5}\in \left[\dfrac{1}{\e};2\right]$.
    Réponse b
    $\quad$
  6. D’une part
    $\begin{align*} h\left(\sqrt{\e}\right)&=\left(\sqrt{\e}\right)^2\left(1+2\ln\left(\sqrt{\e}\right)\right) \\
    &=\e\left(1+2\times \dfrac{1}{2}\ln(\e)\right) \\
    &=2\e\end{align*}$
    D’autre part
    $\begin{align*} h’\left(\sqrt{\e}\right)&=4\left(\sqrt{\e}\right)\left(1+\ln\left(\sqrt{\e}\right)\right) \\
    &=4\sqrt{e}\left(1+\dfrac{1}{2}\right)\\
    &=6\sqrt{\e}\end{align*}$
    Une équation de la tangente à $C_h$ au point d’abscisse $\sqrt{\e}$ est donc $y=6\sqrt{\e}\left(x-\sqrt{\e}\right)+2\e$
    Or
    $\begin{align*} 6\sqrt{\e}\left(x-\sqrt{\e}\right)+2\e&=6\sqrt{\e}x-6\e+2\e \\
    &=6\sqrt{\e}x-4\e \\
    &=\left(6\e^{1/2}\right).x-4\e\end{align*}$
    Réponse d
    $\quad$
  7. Pour tout réel $x\in ]0;2]$ on a
    $\begin{align*} h\dsec(x)&=4\left(1+\ln(x)\right)+4x\times \dfrac{1}{x} \\
    &=4+4\ln(x)+4 \\
    &=8+4\ln(x)\end{align*}$
    $\begin{align*} h\dsec(x)>0&\ssi 8+4\ln(x)>0 \\
    &\ssi 4\ln(x)>-8 \\
    &\ssi \ln(x)>-2 \\
    &\ssi x>\e^{-2}\end{align*}$.
    On a, de même, $h\dsec(x)=0 \ssi x=\e^{-2}$.
    $\e^{-2}\in ]0;2]$.
    La courbe $C_h$ possède donc un unique point d’inflexion sur $]0;2]$.
    Réponse b
    $\quad$

 

Ex 3

Exercice 3

Partie A

  1. a. $\lim\limits_{x\to -\infty} 0,5x-2=-\infty$ et $\lim\limits_{X\to -\infty} \e^X=0$ donc $\lim\limits_{x\to -\infty} \e^{0,5x-2}=0$.
    Ainsi $\lim\limits_{x\to -\infty} f(x)=-\infty$.
    $\quad$
    b. Pour tout réel $x$ non nul on a
    $\begin{align*} 1+0,5x\left(2-\dfrac{\e^{0,5x}}{0,5x}\times \e^{-2}\right) &=1+x-\e^{-0,5x}\times \e^{-2} \\
    &=f(x)\end{align*}$
    $\lim\limits_{x\to +\infty} 0,5x=+\infty$ et, par croissances comparées, $\lim\limits_{X\to +\infty} \dfrac{\e^X}{X}=+\infty$.
    Donc $\lim\limits_{x\to +\infty} \dfrac{\e^{0,5x}}{0,5x}=+\infty$.
    Par produit des limites, $\lim\limits_{x\to +\infty} f(x)=-\infty$.
    $\quad$
  2. a. Pour tout réel $x$ on a $f'(x)=1-0,5\e^{0,5x-2}$
    $\quad$
    b.
    $\begin{align*} f'(x)<0&\ssi 1-0,5\e^{0,5x-2}<0 \\
    &\ssi -0,5\e^{0,5x-2}<-1 \\
    &\ssi \e^{0,5x-2}>2 \\
    &\ssi 0,5x-2>\ln(2) \\
    &\ssi 0,5x>2+\ln(2) \\
    &\ssi x>4+2\ln(2)\end{align*}$
    Ainsi l’ensemble des solutions de l’inéquation $f'(x)<0$ est bien $\left]4+2\ln(2);+\infty\right[$.
    $\quad$
  3. En raisonnant de la même façon on obtient $f'(x)=0 \ssi x=4+2\ln(2)$.
    On obtient donc le tableau de variations suivant :

    $\begin{align*} f\left(4+2\ln(2)\right)&=1+4+2\ln(2)-\e^{2+\ln(2)-2} \\
    &=5+2\ln(2)-2\\
    &=3+2\ln(2)\end{align*}$
    $\quad$
  4. $4+2\ln(2)>0$.
    La fonction $f$ est donc continue (car dérivable) et strictement croissante sur $[-1;0]$.
    $f(-1)=-\e^{-2,5}<0$ et $f(0)=1-\e^{-2}>0$
    D’après le théorème de la bijection (ou corollaire du théorème des valeurs intermédiaires) l’équation $f(x)=0$ admet donc une unique solution sur l’intervalle $[-1;0]$.
    $\quad$

Partie B

  1. a. Pour tout entier naturel $n$ on pose $P(n):~u_n\pp u_{n+1} \pp 4$
    Initialisation : $u_0=0$ et $u_1=1-\e^{-2}\approx 0,86$
    Donc $u_0\pp u_1\pp 4$ et $P(0)$ est vraie.
    $\quad$
    Hérédité : Soit $n\in \N$. On suppose $P(n)$ vraie.
    La fonction $f$ est strictement croissante sur $\left]-\infty;4+2\ln(2)\right]$ donc sur $[0;4]$.
    $\begin{align*} u_n\pp u_{n+1} \pp 4&\Rightarrow f\left(u_n\right) \pp f\left(u_{n+1}\right) \pp f(4) \\
    &\Rightarrow u_{n+1}\pp u_{n+2}\pp 5-1\end{align*}$
    Par conséquent $P(n+1)$ est vraie.
    $\quad$
    Conclusion : La propriété est vraie au rang $0$ et est héréditaire.
    Pour tout entier naturel $n$ on a $u_n \pp u_{n+1} \pp 4$.
    $\quad$
    b. La suite $\left(u_n\right)$ est croissante et majorée par $4$; elle converge donc vers un réel $\ell$.
    $\quad$
  2. a. $\ell$ est solution de l’équation $x=f(x)$
    $\begin{align*} x=f(x)&\ssi 1+x-\e^{0,5x-2}=x \\
    &\ssi 1-\e^{0,5x-2}=0 \\
    &\ssi \e^{0,5x-2}=1 \\
    &\ssi 0,5x-2=0 \\
    &\ssi 0,5x=2 \\
    &\ssi x=4\end{align*}$
    Ainsi $\ell =4$.
    $\quad$
    b. La fonction $\texttt{valeur}$ renvoie le plus petit entier naturel $n$ tel que $u_n>a$.
    Cela signifie donc le plus petit entier naturel $n$ tel que $u_n>3,99$ est $12$.
    $\quad$

 

Ex 4

Exercice 4

  1. a. On a $R(3;2;-1)$ et $\vect{AB}\begin{pmatrix} -4\\4\\0\end{pmatrix}$
    $\quad$
    b. Une équation du plan $\mathscr{P}_1$ est donc de la forme $-4x+4y+d=0$.
    $R(3;2;-1)$ appartient au plan $\mathscr{P}_1$ donc $-12+8+d=0 \ssi d=4$.
    Une équation de $\mathscr{P}_1$ est donc $-4x+4y+4=0$ soit $x-y-1=0$.
    $\quad$
    c. $10-9-1=0$ donc $E(10;9;8)$ appartient à $\mathscr{P}_1$.
    $\vect{EA}\begin{pmatrix} -5\\-9\\-9\end{pmatrix}$ et $\vect{EB}\begin{pmatrix} -9\\-5\\-9\end{pmatrix}$
    $\begin{align*} EA&=\sqrt{(-5)^2+(-9)^2+(-9)^2}\\
    &=\sqrt{25+81+81} \\
    &=\sqrt{187}\end{align*}$
    $\begin{align*} EB&=\sqrt{(-9)^2+(-5)^2+(-9)^2}\\
    &=\sqrt{187}\end{align*}$
    On a donc $EA=EB$.
    $\quad$
  2. a. Un vecteur normal au plan $\mathscr{P}_2$ est $\vec{n}\begin{pmatrix}1\\0\\-1\end{pmatrix}$
    $\vect{AB}$ et $\vec{n}$ ne sont pas colinéaires.
    Les plans $\mathscr{P}_1$ et $\mathscr{P}_2$ sont par conséquent sécants.
    $\quad$
    b. Soit $t\in \R$.
    $\begin{align*} (2+t)-(1+t)-1&=2+t-1-t-1 \\
    &=0\end{align*}$
    La droite dont une représentation paramétrique est $\begin{cases} x=2+t\\y=1+t\\z=t\end{cases},~(t\in \R)$ est incluses dans le plan $\mathscr{P}_1$.
    $\begin{align*} (2+t)-t-2&=2+t-t-2 \\
    &=0\end{align*}$
    La droite dont une représentation paramétrique est $\begin{cases} x=2+t\\y=1+t\\z=t\end{cases},~(t\in \R)$ est incluses dans le plan $\mathscr{P}_2$.
    L’intersection de deux plans est une droite.
    Ainsi une représentation paramétrique de $\Delta$ est $\begin{cases} x=2+t\\y=1+t\\z=t\end{cases},~(t\in \R)$.
    $\quad$
  3. $\quad$
    $\begin{align*} \begin{cases} x=2+t\\y=1+t\\z=t\\y+z-3=0\end{cases} &\ssi  \begin{cases} x=2+t\\y=1+t\\z=t\\1+t+t-3=0\end{cases} \\
    &\ssi \begin{cases} x=2+t\\y=1+t\\z=t\\t=1\end{cases} \\
    &\ssi \begin{cases} t=1\\x=3\\y=2\\z=1\end{cases}\end{align*}$
    La droite $\Delta$ est sécante au plan $\mathscr{P}_3$ en $\Omega(3;2;1)$.
    $\quad$
  4. a. $\Omega$ appartient au plan médiateur de $[AB]$ donc $\Omega A=\Omega B$.
    $\Omega$ appartient au plan médiateur de $[AC]$ donc $\Omega A=\Omega C$.
    $\Omega$ appartient au plan médiateur de $[AD]$ donc $\Omega A=\Omega D$.
    Ainsi $\Omega A=\Omega B=\Omega C=\Omega D$.
    $\quad$
    b. Les points $A$, $B$, $C$ et $D$ appartiennent donc à la sphère de centre $\Omega$ et de rayon $\Omega A$.
    Or
    $\begin{align*} \Omega A&=\sqrt{(5-3)^2+(0-2)^2+(-1-1)^2} \\
    &=\sqrt{4+4+4} \\
    &=2\sqrt{3}\end{align*}$
    $\quad$

 

Énoncé

Exercice 1     7 points

Thème : probabilités

Dans une station de ski, il existe deux types de forfait selon l’âge du skieur :

  • un forfait JUNIOR pour les personnes de moins de vingt-cinq ans ;
  • un forfait SÉNIOR pour les autres.

Par ailleurs, un usager peut choisir, en plus du forfait correspondant à son âge,
l’option coupe-file qui permet d’écourter le temps d’attente aux remontées
mécaniques.

On admet que :

  • $20\%$ des skieurs ont un forfait JUNIOR ;
  • $80\%$ des skieurs ont un forfait SÉNIOR ;
  • parmi les skieurs ayant un forfait JUNIOR, $6\%$ choisissent l’option coupe-file ;
  • parmi les skieurs ayant un forfait SÉNIOR, $12,5\%$ choisissent l’option coupe-file.

On interroge un skieur au hasard et on considère les événements :

  • $J$ : « le skieur a un forfait JUNIOR » ;
  • $C$ : « le skieur choisit l’option coupe-file ».

Les deux parties peuvent être traitées de manière indépendante.

Partie A

  1. Traduire la situation par un arbre pondéré.
    $\quad$
  2. Calculer la probabilité $P(J\cap C)$.
    $\quad$
  3. Démontrer que la probabilité que le skieur choisisse l’option coupe-file
    est égale à $0,112$.
    $\quad$
  4. Le skieur a choisi l’option coupe-file. Quelle est la probabilité qu’il s’agisse d’un skieur ayant un forfait SÉNIOR ? Arrondir le résultat à $10^{-3}$.
    $\quad$
  5. Est-il vrai que les personnes de moins de vingt-cinq ans représentent moins de $15\%$ des skieurs ayant choisi l’option coupe-file ? Expliquer.
    $\quad$

Partie B
On rappelle que la probabilité qu’un skieur choisisse l’option coupe-file est
égale à $0,112$.

On considère un échantillon de $30$ skieurs choisis au hasard.

Soit $X$ la variable aléatoire qui compte le nombre des skieurs de l’échantillon ayant choisi l’option coupe-file.

  1. On admet que la variable aléatoire $X$ suit une loi binomiale.
    Donner les paramètres de cette loi.
    $\quad$
  2. Calculer la probabilité qu’au moins un des $30$ skieurs ait choisi l’option coupe-file.
    Arrondir le résultat à $10^{-3}$.
    $\quad$
  3. Calculer la probabilité qu’au plus un des $30$ skieurs ait choisi l’option coupe-file.
    Arrondir le résultat à $10^{-3}$.
    $\quad$
  4. Calculer l’espérance mathématique de la variable aléatoire $X$.
    $\quad$

$\quad$

Exercice 2     7 points

Thème : suites, fonctions, fonction logarithme

Cet exercice est un questionnaire à choix multiples.
Pour chacune des questions suivantes, une seule des quatre réponses proposées est exacte.
Une réponse fausse, une réponse multiple ou l’absence de réponse à une question ne rapporte ni n’enlève de point.
Pour répondre, indiquer sur la copie le numéro de la question et la lettre de la
réponse choisie. Aucune justification n’est demandée.

  1. Un récipient contenant initialement $1$ litre d’eau est laissé au soleil.
    Toutes les heures, le volume d’eau diminue de $15\%$.
    Au bout de quel nombre entier d’heures le volume d’eau devient-il inférieur à un quart de litre ?
    a. $2$ heures
    b. $8$ heures
    c. $9$ heures
    d. $13$ heures
    $\quad$
  2. On considère la suite $\left(u_n\right)$ définie pour tout entier naturel $n$ par $u_{n+1}=\dfrac{1}{2}u_n+3$ et $u_0=6$. On peut affirmer que :
    a. la suite $\left(u_n\right)$ est strictement croissante.
    b. la suite $\left(u_n\right)$ est strictement décroissante.
    c. la suite $\left(u_n\right)$ n’est pas monotone.
    d. la suite $\left(u_n\right)$ est constante.
    $\quad$
  3. On considère la fonction $f$ définie sur l’intervalle $]0;+\infty[$ par $f(x)=4\ln(3x)$
    Pour tout réel $x$ de l’intervalle $]0;+\infty[$ , on a :
    a. $f(2x)=f(x)+\ln(24)$
    b. $f(2x)=f(x)+\ln(16)$
    c. $f(2x)=\ln(2)+f(x)$
    d. $f(2x)=2f(x)$
    $\quad$
  4. On considère la fonction $g$ définie sur l’intervalle $]1;+\infty[$ par :
    $$g(x)\dfrac{\ln(x)}{x-1}$$
    On note $\mathcal{C}_g$ la courbe représentative de la fonction $g$ dans un repère orthogonal.
    La courbe $\mathcal{C}_g$ admet :
    a. une asymptote verticale et une asymptote horizontale.
    b. une asymptote verticale et aucune asymptote horizontale.
    c. aucune asymptote verticale et une asymptote horizontale.
    d. aucune asymptote verticale et aucune asymptote horizontale.
    $\quad$

Dans la suite de l’exercice, on considère la fonction $h$ définie sur l’intervalle $]0 ; 2]$ par : $$h(x) = x^2\left(1 + 2 \ln(x)\right)$$
On note $\mathcal{C}_h$ la courbe représentative de $h$ dans un repère du plan.
On admet que $h$ est deux fois dérivable sur l’intervalle $]0 ; 2]$.
On note $h’$ sa dérivée et $h\dsec$ sa dérivée seconde.

On admet que, pour tout réel $x$ de l’intervalle $]0 ; 2]$, on a :$$h'(x)=4x\left(1+\ln(x)\right)$$

  1. Sur l’intervalle $\left[\dfrac{1}{\e};2\right]$, la fonction $h$ s’annule :
    a. exactement $0$ fois.
    b. exactement $1$ fois.
    c. exactement $2$ fois.
    d. exactement $3$ fois.
    $\quad$
  2. Une équation de la tangente à $\mathcal{C}_h$ au point d’abscisse $\sqrt{\e}$ est :
    a. $y=\left(6\e^{\frac{1}{2}}\right).x$
    b. $y=\left(6\sqrt{\e}\right).x+2\e$
    c. $y=6\e^{\frac{x}{2}}$
    d. $y=\left(6\e^{\frac{1}{2}}\right).x-4\e$
    $\quad$
  3. Sur l’intervalle $]0 ; 2]$, le nombre de points d’inflexion de la courbe $\mathcal{C}_h$ est égal à :
    a. $0$
    b. $1$
    c. $2$
    d. $3$
    $\quad$

$\quad$

Exercice 3     7 points

Thème : suites, fonctions, fonction exponentielle

Partie A

On considère la fonction $f$ définie pour tout réel $x$ par : $$f(x)=1+x-\e^{0,5x-2}$$
On admet que la fonction $f$ est dérivable sur $\R$. On note $f’$ sa dérivée.

  1. a. Déterminer la limite de la fonction $f$ en $-\infty$.
    $\quad$
    b. Démontrer que, pour tout réel $x$ non nul, $f(x) = 1 + 0,5x\left(2-\dfrac{\e^{0,5x}}{0,5x}\times \e^{-2}\right)$.
    En déduire la limite de la fonction $f$ en $+\infty$.
    $\quad$
  2. a. Déterminer $f'(x)$ pour tout réel $x$.
    $\quad$
    b. Démontrer que l’ensemble des solutions de l’inéquation $f'(x)<0$ est
    l’intervalle $]4 + 2\ln(2) ; +\infty[$.
    $\quad$
  3. Déduire des questions précédentes le tableau de variation de la fonction $f$ sur $\R$.
    On fera figurer la valeur exacte de l’image de $4 + 2\ln(2)$ par $f$.
    $\quad$
  4. Montrer que l’équation $f(x) = 0$ admet une unique solution sur l’intervalle $[-1; 0]$.
    $\quad$

Partie B

On considère la suite $\left(u_n\right)$ définie par $u_0=0$ et, pour tout entier naturel $n$ ,
$$u_{n+1}=f\left(u_n\right) \text{ où } f \text{ est la fonction définie à la }\textbf{ partie A.}$$

  1. a. Démontrer par récurrence que, pour tout entier naturel $n$ , on a : $$u_n\pp u_{n+1}\pp 4$$
    $\quad$
    b. En déduire que la suite $\left(u_n\right)$ converge. On notera $\ell$ la limite.
    $\quad$
  2. a. On rappelle que $\ell$ vérifie la relation $\ell=f(\ell)$.
    Démontrer que $\ell = 4$.
    $\quad$
    b. On considère la fonction $\texttt{valeur}$ écrite ci-dessous dans le langage Python :
    $\begin{array}{|l|}
    \hline
    \text{def valeur(a):}\\
    \quad\text{u=0}\\
    \quad\text{n=0}\\
    \quad\text{while u<=a:}\\
    \qquad\text{u=1+u-exp(0.5*u-2)}\\
    \qquad\text{n=n+1}\\
    \quad\text{return n}\\
    \hline
    \end{array}$
    L’instruction $\texttt{valeur(3.99)}$ renvoie la valeur $12$.
    Interpréter ce résultat dans le contexte de l’exercice.
    $\quad$

$\quad$

Exercice 4     7 points

Thème : Géométrie dans l’espace

L’espace est muni d’un repère orthonormé $\Oijk$.
On considère les points $A(5 ; 0 ; -1)$, $B(1 ; 4 ; -1)$, $C(1 ; 0 ; 3)$, $D(5 ; 4 ; 3)$ et $E(10 ; 9 ; 8)$

  1. a. Soit $R$ le milieu du segment $[AB]$.
    Calculer les coordonnées du point $R$ ainsi que les coordonnées du vecteur $\vect{AB}$.
    $\quad$
    b. Soit $\mathcal{P}_1$ le plan passant par le point $R$ et dont $\vect{AB}$ est un vecteur normal.
    Démontrer qu’une équation cartésienne du plan $\mathcal{P}_1$ est :
    $$x-y-1=0$$
    $\quad$
    c. Démontrer que le point $E$ appartient au plan $\mathcal{P}_1$ et que $EA = EB$.
    $\quad$
  2. On considère le plan $\mathcal{P}_2$ d’équation cartésienne $x-z-2=0$.
    a. Justifier que les plans $\mathcal{P}_1$ et $\mathcal{P}_2$ sont sécants.
    $\quad$
    b. On note $\Delta$ la droite d’intersection de $\mathcal{P}_1$ et $\mathcal{P}_2$ .
    Démontrer qu’une représentation paramétrique de la droite $\Delta$ est :$$\begin{cases} x=2+t\\y=1+t\\z=t\end{cases} \quad (t\in \R)$$
    $\quad$
  3. On considère le plan $\mathcal{P}_3$ d’équation cartésienne $y+z-3=0$.
    Justifier que la droite $\Delta$ est sécante au plan $\mathcal{P}_3$ en un point $\Omega$ dont on déterminera les coordonnées.

Si $S$ et $T$ sont deux points distincts de l’espace, on rappelle que l’ensemble des points $M$ de l’espace tels que $MS = MT$ est un plan, appelé plan médiateur du segment $[ST]$.
On admet que les plans $\mathcal{P}_1$, $\mathcal{P}_2$ et $\mathcal{P}_3$ sont les plans médiateurs respectifs des segments $[AB]$, $[AC]$ et $[AD]$.

  1. a. Justifier que $\Omega A = \Omega B = \Omega C = \Omega D$.
    $\quad$
    b. En déduire que les points $A$, $B$, $C$ et $D$ appartiennent à une même sphère dont on précisera le centre et le rayon.
    $\quad$

$\quad$

 

 

 

Bac – Spécialité mathématiques – Centres étrangers – sujet 2 – 12 mai 2022

Centres étrangers – 12 mai 2022

Spécialité maths – Sujet 2- Correction

L’énoncé de ce sujet de bac est disponible ici.

Ex 1

Exercice 1

  1. La fonction $f$ est dérivable sur $\R$ en tant que quotient de fonctions dérivables dont le dénominateur ne s’annule pas.
    Pour tout réel $x$ on a :
    $\begin{align*} f'(x)&=\dfrac{1\times\e^x-x\e^x}{\left(\e^x\right)^2} \\
    &=\dfrac{(1-x)\e^x}{\e^{2x}} \\
    &=\dfrac{1-x}{\e^x} \\
    &=(1-x)\e^{-x}\end{align*}$
    Réponse C
    $\quad$
  2. La fonction $f\dsec$ semble donc strictement positive sur $]-3;-1[$ et strictement négative sur $]-1;1[$.
    La fonction $f’$ semble donc croissante sur $[-3;-1]$ et strictement décroissante sur $[-1;1]$.
    Ainsi $f’$ admet un maximum en $x=-1$.
    Réponse D
    $\quad$
  3. On considère la fonction $F$ définie sur $\R$ par $F(x)=-\dfrac{1}{2}\left(x^2+1\right)\e^{-x^2}$.
    Elle est dérivable sur $\R$ en tant que produit et composée de fonctions dérivables sur $\R$.
    Pour tout réel $x$
    $\begin{align*} F'(x)&=-\dfrac{1}{2}\left(2x\e^{-x^2}+\left(x^2+1\right)\times (-2x)\e^{-x^2}\right)\\
    &=-\dfrac{1}{2}\left(2x\e^{-x^2}-2x^3\e^{-x^2}-2x\e^{-x^2}\right) \\
    &=-\dfrac{1}{2}\times \left(-2x^3\right) \e^{-x^2}\\
    &=f(x)\end{align*}$
    Réponse C
    $\quad$
  4. Pour tout réel $x$ on a
    $\begin{align*} \dfrac{\e^x+1}{\e^x-1}&=\dfrac{\e^x\left(1+\e^{-x}\right)}{\e^x\left(1-\e^{-x}\right)} \\
    &=\dfrac{1+\e^{-x}}{1-\e^{-x}}\end{align*}$
    Or $\lim\limits_{x\to +\infty} \e^{-x}=0$.
    Donc $\lim\limits_{x\to +\infty} \dfrac{\e^x+1}{\e^x-1}=1$.
    Réponse B
    $\quad$
  5. Une primitive de la fonction $f$ est la fonction $F$ définie sur $\R$ par $F(x)=\dfrac{1}{2}\e^{2x+1}+K$
    $\begin{align*} F(0)=1&\ssi \dfrac{1}{2}\e+K=1 \\
    &\ssi K=1-\dfrac{1}{2}\e\end{align*}$
    Donc, pour tout réel $x$, $F(x)=\dfrac{1}{2}\e^{2x+1}+1-\dfrac{1}{2}\e$.
    Réponse C
    $\quad$
  6. La fonction $f$ semble concave sur $[-2;1]$ et convexe sur $[1;4]$.
    Par conséquent $f\dsec(x)$ est négatif sur $[-2;1]$ et positif sur $[1;4]$ en ne s’annulant qu’en $1$.
    Réponse A
    $\quad$

 

Ex 2

Exercice 2

  1. Par croissances comparées, $\lim\limits_{x\to 0} x\ln(x)=0$ donc $\lim\limits_{x\to 0} f(x)=1$.
    $\quad$
    $\lim\limits_{x\to +\infty} \ln(x)=+\infty$ donc $\lim\limits_{x\to +\infty} x\ln(x)=+\infty$ et $\lim\limits_{x\to +\infty} f(x)=+\infty$
    $\quad$
  2. a. Pour tout réel $x$ strictement positif,
    $\begin{align*} f'(x)&=1\times \ln(x)+x\times \dfrac{1}{x} \\
    &=\ln(x)+1\end{align*}$
    $\quad$
    b. $\ln(x)+1=0 \ssi \ln(x)=-1 \ssi x=\e^{-1}$
    $\ln(x)+1>0\ssi \ln(x)>-1 \ssi x>\e^{-1}$
    On obtient donc le tableau de variations suivant :

    $\quad$
    c. La fonction $f$ est strictement décroissante sur $\left]0;\e^{-1}\right[$
    Donc pour tout $x\in \left]0;\e^{-1}\right]$ on a $0<1-\e^{-1}\pp f(x) <1$.
    $f(1)=1$.
    La fonction $f$ est strictement croissante sur $\left]0;\e^{-1}\right[$
    Donc pour tout $x\in \left[\e^{-1};1\right[$ on a $0<1-\e^{-1}\pp f(x) < 1$.
    Ainsi, pour tout $x\in [0;1]$ on a $0<f(x)<1$.
    $\quad$
  3. a. $f'(1)=1$ et $f(1)=1$
    Une équation de $(T)$ est donc $y=1\times (x-1)+1$ soit $y=x$.
    $\quad$
    b. La fonction $f’$ est dérivable sur $]0;+\infty[$ en tant que somme de fonctions dérivables.
    Pour tout réel $x\in ]0;+\infty[$ on a $f\dsec(x)=\dfrac{1}{x}>0$.
    La fonction $f$ est donc convexe sur $]0;+\infty[$.
    $\quad$
    c. La courbe $C_f$ est donc au-dessus de toutes ses tangentes en particulier au-dessus de $T$.
    Donc, pour tout réel $x$ strictement positif, $f(x)\pg x$.
    $\quad$
  4. a. Pour tout entier naturel $n$ on pose $P(n):~0<u_n<1$.
    Initialisation : $u_0\in ]0;1[$ par définition. Donc $P(0)$ est vraie.
    $\quad$
    Hérédité : Soit $n\in \N$. On suppose $P(n)$ vraie.
    On a $0<u_n<1$ donc, d’après la question 2.c., $0<f\left(u_n\right)<1$ soit $0<u_{n+1}<1$.
    $P(n+1)$ est donc vraie.
    $\quad$
    Conclusion : La propriété est vraie au rang $0$ et est héréditaire.
    Par conséquent, pour tout entier naturel $n$ on a $0<u_n<1$.
    $\quad$
    b. Soit $n\in \N$
    D’après la question 3.c. on a $f\left(u_n\right)\pg u_n$ soit $u_{n+1}\pg u_n$.
    La suite $\left(u_n\right)$ est donc croissante.
    $\quad$
    c. La suite $\left(un\right)$ est croissante et majorée par $1$. Elle est donc convergente.
    $\quad$

 

 

 

Ex 3

Exercice 3

  1. $\vect{AB}\begin{pmatrix}3\\3\\3\end{pmatrix}$, $\vect{AC}\begin{pmatrix}3\\0\\-3\end{pmatrix}$ et $\vect{AD}\begin{pmatrix}-3\\6\\-3\end{pmatrix}$
    $A$, $B$, $C$ et $D$ sont coplanaires si, et seulement si, il existe deux réels $x$ et $y$ tels que :
    $\begin{align*} \vect{AD}=x\vect{AB}+y\vect{AC}&\ssi \begin{cases} 3x+3y&=-3\\3x&=6\\3x-3y&=-3\end{cases} \\
    &\ssi \begin{cases} x+y&=-3\\x&=2\\x-y&=-1\end{cases} \\
    &\ssi \begin{cases} x=2\\y=-5\\y=3\end{cases}\end{align*}$
    Les deux dernières lignes du système sont incompatibles.
    Les points $A$, $B$, $C$ et $D$ ne sont donc pas coplanaires.
    $\quad$
  2. a.
    $\begin{align*} \vect{AB}.\vect{AC}&=3\times 3+3\times 0+3\times (-3) \\
    &=9+0-9\\
    &=0\end{align*}$
    Ces deux vecteurs sont orthogonaux.
    Le triangle $ABC$ est donc rectangle en $A$.
    $\quad$
    b.
    $\begin{align*} \vect{AD}.\vect{AB}&=-3\times 3+6\times 3+(-3)\times 3 \\
    &=-9+18-9 \\
    &=0\end{align*}$
    $\begin{align*} \vect{AD}.\vect{AC}&=-3\times 3+6\times 0+(-3)\times (-3) \\
    &=-9+0+9 \\
    &=0\end{align*}$
    Le vecteur $\vect{AD}$ est donc orthogonal à deux vecteurs non colinéaires (ils sont orthogonaux d’après la question précédente) du plan $(ABC)$.
    La droite $(AD)$ est par conséquent perpendiculaire au plan $(ABC)$.
    $\quad$
    c.
    $\begin{align*} AD&=\sqrt{(-3)^2+6^2+(-3)^2} \\
    &=\sqrt{9+36+9} \\
    &=\sqrt{54}\end{align*}$
    $\begin{align*} AB&=\sqrt{3^2+3^2+3^2} \\
    &=\sqrt{9+9+9} \\
    &=\sqrt{27}\end{align*}$
    $\begin{align*} AC&=\sqrt{3^2+0^2+(-3)^2} \\
    &=\sqrt{9+0+9} \\
    &=\sqrt{18}\end{align*}$
    L’aire du triangle $ABC$ est
    $\begin{align*} \mathscr{A}&=\dfrac{AB\times AC}{2} \\
    &=\dfrac{\sqrt{27}\times \sqrt{18}}{2}\\
    &=\dfrac{9\sqrt{6}}{2}\end{align*}$
    Par conséquent le volume du tétraèdre $ABCD$ est
    $\begin{align*} \mathscr{V}&=\dfrac{1}{3}\times AD\times \mathscr{A} \\
    &=\dfrac{1}{3}\times \sqrt{54}\times \dfrac{9\sqrt{6}}{2}\\
    &=27\end{align*}$
    $\quad$
  3. a. $\vect{BH}\begin{pmatrix}-1\\-1\\-4\end{pmatrix}$, $\vect{BC}\begin{pmatrix}0\\-3\\-6\end{pmatrix}$ et $\vect{BD}\begin{pmatrix}-6\\3\\-6\end{pmatrix}$
    $\begin{align*} \vect{BH}=\alpha\vect{BC}+\beta\vect{BD}&\ssi \begin{cases} -6\beta&=-1 \\
    -3\alpha+3\beta&=-1\\
    -6\alpha-6\beta&=-4\end{cases} \\
    &\ssi \begin{cases} \beta=\dfrac{1}{6}\\\alpha=\dfrac{1}{2}\end{cases}\end{align*}$
    Par conséquent $\vect{BH}=\dfrac{1}{2}\vect{BC}+\dfrac{1}{6}\vect{BD}$.
    $\quad$
    b. $\vect{AH}\begin{pmatrix}2\\2\\-1\end{pmatrix}$
    $\begin{align*} \vect{AH}.\vect{BC}&=2\times 0+2\times (-3)+(-1)\times (-6) \\
    &=0\end{align*}$
    $\begin{align*} \vect{AH}.\vect{BD}&=2\times (-6)+2\times 3+(-1)\times (-6) \\
    &=0\end{align*}$
    Le vecteur $\vect{AH}$ est donc orthogonal à deux vecteurs non colinéaires du plan $(BCD)$.
    C’est donc un vecteur normal à ce plan.
    D’après la question précédente, $H$ appartient au plan $(BCD)$.
    Donc $H$ est le projeté orthogonal du point $A$ sur le plan $(BCD)$.
    $\quad$
    c.
    $\begin{align*} AH&=\sqrt{2^2+2^2+(-1)^2} \\
    &=\sqrt{9} \\
    &=3\end{align*}$
    La distance du point $A$ au plan $(BCD)$ est égale à $3$ unités de longueur.
    $\quad$
  4. On note $\mathscr{A}$ l’aire du triangle $BCD$.
    $\begin{align*} V=\dfrac{1}{3}\mathscr{A}\times h&\ssi 27=\dfrac{1}{3}\mathscr{A}\times 3\\
    &\ssi \mathscr{A}=27\end{align*}$
    $\quad$

Ex 4

Exercice 4

  1. a. On appelle :
    $\bullet ~~N_i$ l’événement « le jeton tiré lors du $i$-ème tirage est noir » ;
    $\bullet ~~B_i$ l’événement « le jeton tiré lors du $i$-ème tirage est blanc ».
    On obtient donc l’arbre pondéré suivant :
    $ \quad$
    $\quad$
    b. La probabilité de perdre $9$ € sur une partie est égale à :
    $\begin{align*} P\left(B_1\cap B_2\right)&=P\left(B_1\right)\times P_{B_1}\left(B_2\right) \\
    &=\dfrac{3}{5}\times \dfrac{3}{5} \\
    &=\dfrac{9}{25}\end{align*}$
    $\quad$
  2. a. À chaque tirage, la probabilité de tirer un jeton noir vaut $\dfrac{N}{N+3}$ et la probabilité de tirer un jeton blanc vaut $\dfrac{3}{N+3}$.
    La probabilité de tirer deux jetons blancs est égale à $\left(\dfrac{3}{N+3}\right)^2$.
    La probabilité de tirer deux jetons noirs est égale à $\left(\dfrac{N}{N+3}\right)^2$.
    Par conséquent la probabilité de tirer deux jetons de couleurs différentes est égale à :
    $\begin{align*} p&=1-\left(\dfrac{3}{N+3}\right)^2-\left(\dfrac{N}{N+3}\right)^2 \\
    &=1-\dfrac{9}{(N+3)^2}-\dfrac{N^2}{(N+3)^2} \\
    &=\dfrac{(N+3)^2-9-N^2}{(N+3)^2} \\
    &=\dfrac{N^2+6N+9-9-N^2}{(N+3)^2} \\
    &=\dfrac{6N}{(N+3)^2}\end{align*}$
    On obtient ainsi la loi de probabilité suivante :
    $\begin{array}{|c|c|c|c|}
    \hline
    x&-9&-1&5\\
    \hline
    P(X=x)&\dfrac{9}{(N+3)^2}&\dfrac{N^2}{(N+3)^2}&\dfrac{6N}{(N+3)^2}\\
    \hline
    \end{array}$
    $\quad$
    b. Le discriminant de $-x^2+30x-81$ est $\Delta=576>0$.
    Les racines de $-x^2+3x-81$ sont donc $x_1=\dfrac{-30-\sqrt{576}}{-2}=27$ et $x_1=\dfrac{-30+\sqrt{576}}{-2}=3$.
    Le coefficient principal du polynôme du second degré est $a=-1<0$.
    Par conséquent l’ensemble solution de $-x^2+3x-81>0$ est $]3;27[$.
    $\quad$
    c. Le jeu est favorable au joueur si, et seulement si, l’espérance de $X$ est strictement positive.
    $\begin{align*} E(X)>0&\ssi -9\times \dfrac{9}{(N+3)^2}-1\times \dfrac{N^2}{(N+3)^2}+5\times \dfrac{6N}{(N+3)^2}>0 \\
    &\ssi -81-N^2+30N>0\end{align*}$
    D’après la question précédente, le jeu est favorable au joueur si, et seulement si, $N$ est un entier naturel compris entre $4$ et $26$, tous les deux inclus.
    $\quad$
    d. On considère la fonction $g$ définie sur $[0;+\infty[$ par $g(x)=\dfrac{-x^2+30x-81}{(x+3)^2}$.
    Elle est dérivable sur $[0;+\infty[$ en tant que quotient de fonctions dérivables dont le dénominateur ne s’annule pas.
    Pour tout réel $x\in[0;+\infty[$ on a
    $\begin{align*} g'(x)&=\dfrac{(-2x+30)(x+3)^2-2(x+3)\left(-x^2+30x-81\right)}{(x+3)^4} \\
    &=\dfrac{(-2x+30)(x+3)-2\left(-x^2+30x-81\right)}{(x+3)^3} \\
    &=\dfrac{-2x^2-6x+30x+90+2x^2-60x+162}{(x+3)^3}\\
    &=\dfrac{-36x+252}{(x+3)^3}\end{align*}$
    $g'(x)$ est donc du signe de $-36x+252$ sur $[0;+\infty[$.
    Or $-36x+252>0\ssi -36x>-252 \ssi x<7$.
    Par conséquent $g$ est strictement croissante sur $[0;7]$ et strictement décroissante sur $[7;+\infty[$.
    Elle atteint donc son maximum pour $x=7$.
    Or $E(X)=g(N)$ et $7\in [4;26]$.
    Le gain moyen est donc maximal s’il y a $7$ jetons noirs.
    $\quad$
  3. $N=7$ donc $P(X=5)=0,42$.
    On répète $10$ fois de façons indépendantes la même expérience de Bernoulli. On appelle $Y$ la variable aléatoire comptant le nombre de joueur ayant gagné $5$ euros.
    $Y$ suit donc la loi binomiale de paramètres $n=10$ et $p=0,42$.
    $\begin{align*} P(Y\pg 1)&=1-P(Y=0) \\
    &=1-(1-0,42)^{10} \\
    &=1-0,58^{10}\\
    &\approx 0,996\end{align*}$
    La probabilité d’avoir au moins un joueur gagnant $5$ euros est environ égale à $0,996$.
    $\quad$

 

Énoncé

Exercice 1     7 points

Thème : Fonction exponentielle

Cet exercice est un questionnaire à choix multiples. Pour chacune des questions suivantes, une seule des quatre réponses proposées est exacte.

Une réponse incorrecte, une réponse multiple ou l’absence de réponse à une question en rapporte ni n’enlève de point. Pour répondre, indiquer sur la copie le numéro de la question et la lettre de la réponse choisie. Aucune justification n’est demandée.

  1. Soit $f$ la fonction définie sur $\R$ par $$f(x)=\dfrac{x}{\e^x}$$
    On suppose que $f$ est dérivable sur $\R$ et on note $f’$ sa fonction dérivée.
    a. $f'(x)=\e^{-x}$
    b. $f'(x)=x\e^{-x}$
    c. $f'(x)=(1-x)\e^{-x}$
    d. $f'(x)=(1+x)\e^{-x}$
    $\quad$
  2. Soit $f $ une fonction deux fois dérivable sur l’intervalle $[-3;1]$. On donne ci-dessous la représentation graphique de sa fonction dérivée seconde $f\dsec$.
    On peut alors affirmer que :
    a. La fonction $f$ est convexe sur l’intervalle $[-1;1]$
    b. La fonction $f$ est concave sur l’intervalle $[-2;0]$
    c. La fonction $f’$ est décroissante sur l’intervalle $[-2;0]$
    d. La fonction $f’$ admet un maximum en $x=-1$
    $\quad$
  3. On considère la fonction $f$ définie sur $\R$ par : $$f(x)=x^3\e^{-x^2}$$
    Une primitive $F$ de la fonction $f$ est définie sur $\R$ par :
    a. $F(x)=-\dfrac{1}{6}\left(x^3+1\right)\e^{-x^2}$
    b. $F(x)=-\dfrac{1}{4}x^4\e^{-x^2}$
    c. $F(x)=-\dfrac{1}{2}\left(x^2+1\right)\e^{-x^2}$
    d. $F(x)=x^2\left(3-2x^2\right)\e^{-x^2}$
    $\quad$
  4. Que vaut  $$\lim\limits_{x\to +\infty} \dfrac{\e^x+1}{\e^x-1}$$
    a $-1$
    b. $1$
    c. $+\infty$
    d. N’existe pas
    $\quad$
  5. On considère la fonction $f$ définie sur $\R$ par $f(x)=\e^{2x+1}$.
    La seule primitive de $F$ sur $\R$ de la fonction $f$ telle que $F(0)=1$ est la fonction :
    a. $x\mapsto 2\e^{2x+1}-2\e+1$
    b. $x\mapsto \e^{2x+1}-\e$
    c. $x\mapsto \dfrac{1}{2}\e^{2x+1}-\dfrac{1}{2}\e+1$
    d. $x\mapsto \e^{x^2+x}$
    $\quad$
  6. Dans un repère, on a tracé ci-dessous la courbe représentative d’une fonction $f$ définie et deux fois dérivable sur $[-2;4]$.
    Parmi les courbes suivantes, laquelle représente la fonction $f\dsec$, dérivée seconde de $f$?
    a.
    b.
    c. d. $\quad$

$\quad$

Exercice 2     7 points

Thème : Fonction logarithme et suite

Soit $f$ la fonction définie sur l’intervalle $]0;+\infty[$ par $$f(x)=x\ln(x)+1$$

On note $C_f$ sa courbe représentative dans un repère du plan.

  1. Déterminer la limite de la fonction $f$ en $0$ ainsi que sa limite en $+\infty$.
    $\quad$
  2. a. On admet que $f$ est dérivable sur $]0;+\infty[$ et on notera $f’$ sa fonction dérivée.
    Montrer que pour tout réel $x$ strictement positif : $$f'(x)=1+\ln(x)$$
    $\quad$
    b. En déduire le tableau de variation de la fonction $f$ sur $]0;+\infty[$.
    On y fera figurer la valeur exacte de l’extremum de $f$ sur $]0;+\infty[$ et les limites.
    $\quad$
    c. Justifier que pour tout $x\in ]0;1[$, $f(x)\in ]0;1[$.
    $\quad$
  3. a. Déterminer une équation de la tangente $(T)$ à la courbe $C_f$ au point d’abscisse $1$.
    $\quad$
    b. Étudier la convexité de la fonction $f$ sur $]0;+\infty[$.
    $\quad$
    c. En déduire que pour tout réel $x$ strictement positif $$f(x)\pg x$$
    $\quad$
  4. On définit la suite $\left(u_n\right)$ par son premier terme $u_0$ élément de l’intervalle $]0;1[$ et pour tout entier naturel $n$ : $$u_{n+1}=f\left(u_n\right)$$
    a. Démontrer par récurrence que pour tout entier naturel $n$, on a $0<u_n<1$.
    $\quad$
    b. Déduire de la question 3c la croissance de la suite $\left(u_n\right)$.
    $\quad$
    c. En déduire que la suite $\left(u_n\right)$ est convergente.
    $\quad$

$\quad$

Exercice 3     7 points

Thème : Géométrie dans l’espace

L’espace est muni d’un repère orthonormé $Oijk$.

On considère les points $A(3;-2;2)$, $B(6;1;5)$, $C(6;-2;-1)$ et $D(0;4;-1)$.

On rappelle que le volume d’un tétraèdre est donné par la formule : $$V=\dfrac{1}{3}\mathscr{A}\times h$$
où $\mathscr{A}$ est l’aire de la base et $h$ la hauteur correspondante.

  1. Démontrer que les points $A$, $B$, $C$ et $D$ ne sont pas coplanaires.
    $\quad$
  2. a. Montrer que le triangle $ABC$ est rectangle.
    $\quad$
    b. Montrer que la droite $(AD)$ est perpendiculaire au plan $(ABC)$.
    $\quad$
    c. En déduire le volume du tétraèdre $ABCD$.
    $\quad$
  3. On considère le point $H(5;0;1)$.
    a. Montrer qu’il existe des réels $\alpha$ et $\beta$ tels que $\vect{BH}=\alpha \vect{BC}+\beta\vect{BD}$.
    $\quad$
    b. Démontrer que $H$ est le projeté orthogonal du point $A$ sur le plan $(BCD)$.
    $\quad$
    c. En déduire la distance du point $A$ au plan $(BCD)$.
    $\quad$
  4. Déduire des questions précédentes l’aire du triangle $BCD$.
    $\quad$

$\quad$

Exercice 4     7 points

Thème : Probabilités

Une urne contient des jetons blancs et noirs tous indiscernables au toucher.

Une partie consiste à prélever au hasard successivement et avec remise deux jetons de cette urne.

On établit la règle de jeu suivante :

  • un joueur perd $9$ euros si les deux jetons tirés sont de couleur blanche;
  • un joueur perd $1$ euro si les deux jetons tirés sont de couleur noire;
  • un joueur gagne $5$ euros si les deux jetons tirés sont de couleurs différentes.
  1. On considère que l’urne contient $2$ jetons noirs et $3$ jetons blancs.
    a. Modéliser la situation à l’aide d’un arbre pondéré.
    $\quad$
    b. Calculer la probabilité de perdre $9$ € sur une partie.
    $\quad$
  2. On considère maintenant que l’urne contient $3$ jetons blancs et au moins deux jetons noirs mais on ne connait pas le nombre exact de jetons noirs. On appellera $N$ le nombre de jetons noirs.
    a. Soit $X$ la variable aléatoire donnant le gain du jeu pour une partie.
    Déterminer la loi de probabilité de cette variable aléatoire.
    $\quad$
    b. Résoudre l’inéquation pour $x$ réel : $$-x^2+30x-81>0$$
    $\quad$
    c. En utilisant le résultat de la question précédente, déterminer le nombre de jetons noirs que l’une doit contenir afin que ce jeu soit favorable au joueur.
    $\quad$
    d. Combien de jetons noirs le joueur doit-il demander afin d’obtenir un gain moyen maximal?
    $\quad$
  3. On observe $10$ joueurs qui tentent leur chance en effectuant une partie de ce jeu, indépendamment les uns des autres. On suppose que $7$ jetons noirs ont été placés dans l’urne (avec $3$ jetons blancs). Quelle est la probabilité d’avoir au moins $1$ joueur gagnant $5$ euros?
    $\quad$

$\quad$